You are on page 1of 66

Contact us : info@onlyias.

com

OnlyIAS Nothing Else Visit : dpp.onlyias.in


Contact : +91-7007 931 912

Q.1) सच
ू ी 1 को सच
ू ी II के साथ सम
ु ेलित करें और सचू चयों के a) शील्ड ज्वालामख
ु ी
नीचे दिए गए कूट का प्रयोग कर सही उत्तर का चयन करें : b) ममचित ज्वालामख
ु ी
सच
ू ी1 सूची II c) काल्डेरा ज्वालामुखी
(तह/विन के प्रकार) (विन के उिहारण) d) ववदर/दरार प्रकार का ज्वालामख
ु ी
A. समममत वलन 1) स्ववट्जरलैंड के जुरा
पववत Q.4) अम्िीय और क्षारीय चट्टानों के बीच अंतर के संिर्ा में,

B. उथल पथ ननम्नलिखित में से कौन सा/से कथन सही है/हैं?


ु ल वलन 2) आवरे मलया की ग्रेट
डडवाइडडिंग रें ज 1. अम्लीय ट्टानों में क्षारीय ट्टानों की तल
ु ना में

C. मोनोक्लीनल वलन 3) पाककवतान में कालच त


िं ा मसमलका की उच् मात्रा होती है ।

पववत 2. अम्लीय ट्टानों में क्षारीय ट्टानों की तुलना में भारी

D. इसोक्लीनल 4) भारत मे पीर पिंजाल रें ज धातओ


ु िं की मात्रा अचधक होती है ।
Code: 3. अम्लीय ममट्टी ,क्षारीय ममट्टी की तुलना में अपक्षय के
A B C D मलए कम प्रनतरोधी होती है ।
a) 4 3 2 1
b) 1 4 2 3 नीचे दिए गए कूट का प्रयोग कर सही उत्तर चुननए.
c) 1 3 4 2 a) 1 केवल
d) 4 2 3 1 b) 1 तथा 2 केवल
c) 2 तथा 3 केवल
Q.2) ननम्नलिखित में से कौन-सा/से कथन अवसािी
d) 3 केवल
चट्टानों की ववशेषता है/हैं?
1. पथ्
ृ वी की सतह का तीन ौथाई भाग इनहीिं ट्टानों से
Q.5) ज्वािामुिीय गनतववचि के रचनात्मक प्रर्ावों के संिर्ा
ढका हुआ है ।
में , ननम्नलिखित कथनों पर ववचार करें ::
2. ये ट्टानें खननज सिंसाधनों से रहहत हैं।
1. ज्वालामुखीय ट्टानें अपक्षय और अपघटन पर
3. ये ट्टानें कुछ समद्
ृ ध ममट्टी का स्रोत हैं। उपजाऊ ममट्टी उत्पनन करती हैं।
नीचे दिए गए कूट का प्रयोग कर सही उत्तर चनु नए:
2. ज्वालामुखी वववफोट से ननकलने वाली राख और धूल
a) 1 तथा 2 केवल
खेतों और बगी ों के मलए अनप
ु जाऊ होती है ।
b) 1 तथा 3 केवल
3. ज्वालामख
ु ी मैग्मा का सिंबिंध गीजर के ननमावण से है ।
c) 2 तथा 3 केवल
ऊपर दिए गए कथनों में से कौन-सा/से सही है /हैं?
d) 1,2 तथा 3
a) 1 तथा 3 केवल
b) 1, 2 तथा 3
Q.3) इन ज्वािामुखियों में बेसाल्ट की तुिना में अचिक ठं डा
c) 2 तथा 3 केवल
और अचिक चचपचचपे िावा के ववस्फोट की ववशेषता होती है ।
d) 1 तथा 2 केवल
इन ज्वािामुखियों के पररणामस्वरूप अक्सर ववस्फोटक
ववस्फोट होते हैं। िावा के साथ, बडी मात्रा में
Q.6) P और S र्ू गलर्ाक तरं गों के बीच अंतर के बारे में
पाइरोक्िास्स्टक सामग्री और राि जमीन पर अपना रास्ता
ननम्नलिखित में से कौन सा/से कथन सही है/हैं?
बना िेती है । यह सामग्री वेंट के उद्घाटन के आसपास जमा
1. P-तरिं गें ध्वनन तरिं गों के अनुरूप अनुदैध्यव तरिं गें हैं जबकक
हो जाती है स्जससे परतों का ननमााण होता है । उपरोक्त
S-तरिं गें प्रकाश तरिं गों के अनरू
ु प अनप्र
ु वथ तरिं गें हैं।
कथन में ननम्नलिखित में से ककस ज्वािामुिीय रूप का
2. S-तरिं गों का छाया क्षेत्र P-तरिं गों की अपेक्षा अचधक होता
वणान ककया गया है ?
है ।

DPP 2023 DAY 48 1


Contact us : info@onlyias.com

OnlyIAS Nothing Else Visit : dpp.onlyias.in


Contact : +91-7007 931 912

3. P-तरिं गें तीनों माध्यमों अथावत ठोस, द्रव और गैस से c) 3 4 1 2


d) 3 4 2 1
यात्रा कर सकती हैं जबकक S-तरिं गें केवल ठोस माध्यम से
यात्रा कर सकती हैं। Q.9) ननम्नलिखित कथनो पर ववचार करें :
नीचे दिए गए कूट का प्रयोग कर सही उत्तर चुननए: 1. बहहमुख
व ी आग्नेय ट्टानें महीन दानेदार होती हैं,
a) 1 तथा 2 केवल जबकक अनतवेधी आग्नेय ट्टानें मोटे (खुरदरु े ) दाने
b) 2 केवल वाली होती हैं।
c) 3 केवल 2. बहहमुख
व ी आग्नेय ट्टानों को ज्वालामुखीय ट्टानें भी
d) 1, 2 तथा 3 कहा जाता है जबकक अनतवेधी आग्नेय ट्टानों को
प्लूटोननक ट्टानें भी कहा जाता है ।
Q.7) र्ारत में र्ूकंप मानचचत्रण के संिर्ा में, ननम्नलिखित
3. आग्नेय ट्टानें जीवाश्मों से मुक्त होती हैं लेककन
कथनों पर ववचार करें :
अचधक आचथवक मल्
ू य के खननजों का स्रोत होती हैं।
1. भारतीय मानक ब्यूरो (बीआईएस) द्वारा हदए गए दे श
ऊपर दिए गए कथनों में से कौन-सा/से सही है /हैं?
के भूकिंपीय क्षेत्र मानच त्र के अनुसार भारत को ार
a) 1 तथा 2 केवल
भूकिंपीय क्षेत्रों में वगीकृत ककया गया है ।
b) 2 केवल
2. जोन V भूकिंपीय रूप से सबसे सकिय क्षेत्र है , जबकक जोन
c) 3 केवल
II सबसे कम है ।
d) 1, 2 तथा 3
3. ववज्ञान और प्रौद्योचगकी मिंत्रालय के तहत राष्ट्रीय
भूकिंप ववज्ञान केंद्र दे श में और उसके आसपास भक
ू िं प की Q.10) स्थिीय और जोववयन ग्रहों के बीच अंतर के संबंि में
ननगरानी के मलए भारत सरकार (GoI) की नोडल एजेंसी ननम्नलिखित में से कौन सा/से कथन सही है/हैं?
है । 1. वथलीय ग्रहों का ननमावण मूल तारे के ननकट के क्षेत्र में
ऊपर दिए गए कथनों में से कौन-सा/से सही है /हैं? हुआ था जबकक जोववयन ग्रहों का ननमावण काफी दरू के
a) 1 तथा 2 केवल वथान पर हुआ था।
b) 2 तथा 3 केवल 2. पाचथवव/वथलीय ग्रहों पर जोववयन ग्रहों की तुलना में
c) 1 तथा 3 केवल सौर वायु अचधक तीव्र थी।
d) 1,2 तथा 3 3. वथलीय ग्रह जोववयन ग्रहों से छोटे होते हैं।
नीचे दिए गए कूट का प्रयोग कर सही उत्तर चनु नए:
Q.8) सूची 1 को सच
ू ी II के साथ सुमेलित करें और सचू चयों के
a) 1 तथा 2 केवल
नीचे दिए गए कूट का प्रयोग कर सही उत्तर का चयन करें :
b) 2 तथा 3 केवल

सच c) 1,2 तथा 3
ू ी1 सच
ू ी II
(सकिय ज्वािामि d) 1 तथा 3 केवल
ु ी) (िे श का नाम)
A. ताल ज्वालामख
ु ी 1. इिंडोनेमशया
Q.11) अन्तवेिी आग्नेय चट्टानी वपंडो के संिर्ा में
B. सािंगे ज्वालामख
ु ी 2. नयज़
ू ीलैंड
ननम्नलिखित कथनों पर ववचार करें :
C. सफेद द्वीप 3. कफलीपीिंस
1. बाथोमलथ बडे गुम्बद के आकार के नमलका अनतवेदही
D. माउिं ट मेरापी 4. इक्वेडोर
वपिंड होते हैं स्जनका एक समतल आधार होता है और
Code:
A B C D नी े से एक नमलका से जड
ु ा होता है ।
a) 1 3 2 4
b) 4 3 1 2

DPP 2023 DAY 48 2


Contact us : info@onlyias.com

OnlyIAS Nothing Else Visit : dpp.onlyias.in


Contact : +91-7007 931 912

2. डाइक क्षैनतज वपिंड होते हैं, जो तब ववकमसत होते हैं जब 3. महासागरीय िवट ट्टानें, महाद्वीपीय ट्टानों की
लावा दरारों और भूमम में ववकमसत दरारों के माध्यम से तुलना में बहुत छोटी होती हैं।
अपना रावता बनाता है । ऊपर दिए गए कथनों में से कौन-सा/से सही है /हैं?
ऊपर दिए गए कथनों में से कौन-सा/से सही है /हैं? a) केवल 1 और 2
a) 1 केवल b) केवल 2 और 3
b) 2 केवल c) 1,2 और 3
c) 1 तथा 2दोनों d) केवल 1 और 3
d) कोई नही
Q.15) ननम्नलिखित कथनों पर ववचार करें :
Q.12) कायांतररत चट्टानों के संिर्ा में , ननम्नलिखित 1. दो सामानय भ्रिंशों के बी िवटल भागों के वववथापन
कथनों पर ववचार करें : और मध्य भाग के अवतलन के कारण भ्रिंश घाहटयों का
1. गनतशील कायािंतरण तापमान के दबाव में होता है । ननमावण होता है ।
2. गनतशील कायािंतरण के मामले में , शेल वलेट में 2. खिंड पववत तब बनते हैं जब पथ्
ृ वी की पपवटी समानािंतर
पररवनतवत हो जाता है । भ्रिंश के दोनों ओर डूब जाती है और मध्य भाग अपने
ऊपर दिए गए कथनों में से कौन-सा/से सही है /हैं? वथान पर रहता है ।
a) 1 केवल ऊपर दिए गए कथनों में से कौन-सा/से सही है /हैं?
b) 2 केवल a) केवल 1
c) 1 तथा 2 दोनों b) केवल 2
d) कोई नही c) 1 और 2 दोनों
d) ना ही 1 ना 2
Q.13) डायस्रोकफक बिों (अंतजाात बिों) के संिर्ा में ,
ननम्नलिखित पर ववचार करें : Q.16) महाद्वीपीय शेल्फ के संिर्ा में ननम्नलिखित कथनों
1. लिंबवत डायवरोकफक सिं लनो को ऑरोजेननक सिं लनो पर ववचार करें :
यानी पववत ननमावण गनत के रूप में भी जाना जा सकता 1. महाद्वीपीय समतल की ढाल प्रवणता, 1° या उससे भी
है । कम है ।
2. क्षैनतज डायवरोकफक बलों को सिंपीडन और तनाव बलों 2. महाद्वीपीय शेल्फ की ौडाई और गहराई ववमभनन तटों
में ववभास्जत ककया जा सकता है । पर मभनन होती है ।
ऊपर दिए गए कथनों में से कौन-सा/से सही है /हैं? 3. महाद्वीपीय शेल्फ आचथवक खननजों का एक सिंभाववत
a) केवल 1 स्रोत हैं।
b) केवल 2 ऊपर दिए गए कथनों में से कौन-सा/से सही है /हैं?
c) 1 और 2 दोनों a) 1 और 2
d) ना ही 1 ना 2 b) 2 और 3
c) 1 और 3
Q.14) समद्र
ु ननति प्रसारण पररकल्पना के संबंि में d) 1,2 और 3
ननम्नलिखित कथनों पर ववचार करें :
1. यह पररकल्पना आथवर होम्स द्वारा गढी गई थी। Q.17) समद्र
ु ननति प्रसारण पररकल्पना के संबंि में
2. मध्य महासागरीय कटक के ननकट की ट्टानों में ननम्नलिखित कथनों पर ववचार करें :
सामानय ध्रुवता होती है और ये सबसे छोटी होती हैं। 1. यह पररकल्पना आथवर होम्स द्वारा गढी गई थी।

DPP 2023 DAY 48 3


Contact us : info@onlyias.com

OnlyIAS Nothing Else Visit : dpp.onlyias.in


Contact : +91-7007 931 912

2. मध्य महासागरीय कटक के ननकट की ट्टानों में 1. न


ू ा पत्थर में घोल ,सबसे शस्क्तशाली अपक्षय प्रकिया
सामानय ध्रुवता होती है और ये सबसे छोटी होती हैं। है ।
3. महासागरीय िवट ट्टानें, महाद्वीपीय ट्टानों की 2. नाइरे ट, सल्फेट और पोटे मशयम जैसे खननज ववलयन
तुलना में बहुत छोटी होती हैं। प्रकिया से प्रभाववत नहीिं होते हैं।
ऊपर दिए गए कथनों में से कौन-सा/से सही है /हैं? 3. सोडडयम क्लोराइड एक ट्टान बनाने वाला खननज है ।
a) केवल 1 और 2 ऊपर दिए गए कथनों में से कौन-सा/से सही है /हैं?
b) केवल 2 और 3 a) केवल 1
c) 1,2 और 3 b) केवल 2
d) केवल 1 और 3 c) केवल 1 और 3
d) इनमे से कोई भी नहीिं
Q.18) ननम्नलिखित कथनों पर ववचार करें :
1. दो सामानय भ्रिंशों के बी िवटल भागों के वववथापन Q.21) ‘हाइड्रेशन’ की रासायननक अपक्षय प्रकिया के संिर्ा में
और मध्य भाग के अवतलन के कारण भ्रिंश घाहटयों का ननम्नलिखित कथनों पर ववचार करें :
ननमावण होता है । 1. जलयोजन एक अपररवतवनीय और लिंबी प्रकिया है ।
2. ब्लॉक पववत तब बनते हैं जब पथ्
ृ वी की पपवटी, समानािंतर 2. इस प्रकिया की ननरिं तर पुनराववृ ि से ट्टानों में थकान
भ्रिंश के दोनों ओर डूब जाती है और मध्य भाग अपने होती है और ववघटन हो सकता है ।
वथान पर रहता है । 3. कई मद
ृ ा के खननज गीले और सूखने के दौरान सूज जाते
ऊपर दिए गए कथनों में से कौन-सा/से सही है /हैं? हैं और मसकुड जाते हैं।
a) केवल 1 ऊपर दिए गए कथनों में से कौन-सा/से सही है /हैं?
b) केवल 2 a) केवल 1
c) 1 और 2 दोनों b) केवल 1 और 3
d) ना ही 1 ना 2 c) केवल 2 और 3
d) उपरोक्त सभी
Q.19) महाद्वीपीय अिमाररयों के संिर्ा में ननम्नलिखित
कथनों पर ववचार करें : Q.22) चट्टानों की र्ौनतक अपक्षय प्रकियाओं के संिर्ा में
1. महाद्वीपीय समतल का ढाल 1° या उससे भी कम है । ननम्नलिखित कथनों पर ववचार करें :
2. महाद्वीपीय शेल्फ की ौडाई और गहराई ववमभनन तटों 1. भौनतक अपक्षय प्रकियाएिं थमवल वववतार और दबाव
पर मभनन होती है । मुक्त होने के कारण होती हैं।
3. महाद्वीपीय अलमाररयािं आचथवक खननजों का एक 2. भौनतक अपक्षय प्रकियाएिं गरु
ु त्वाकर्वण बलों और पानी
सिंभाववत स्रोत हैं। के दबावों पर भी ननभवर करती हैं।
ऊपर दिए गए कथनों में से कौन-सा/से सही है /हैं? 3. फ्रॉवट वेस्जिंग भौनतक अपक्षय प्रकियाओिं का एक
a) 1 और 2 उदाहरण है ।
b) 2 और 3 ऊपर दिए गए कथनों में से कौन-सा/से सही है /हैं?
c) 1 और 3 a) केवल 1 और 2
d) 1,2 और 3 b) केवल 2 और 3
c) केवल 1, 3 और 4
Q.20) ‘ववियन’ की रासायननक अपक्षय प्रकिया के संिर्ा में d) उपरोक्त
ननम्नलिखित कथनों पर ववचार करें :

DPP 2023 DAY 48 4


Contact us : info@onlyias.com

OnlyIAS Nothing Else Visit : dpp.onlyias.in


Contact : +91-7007 931 912

Q.23) ‘वह
ृ त ् संचिन' की बदहजाात र्ू-आकृनत प्रकिया के c) केवल 1 और 2
संिर्ा में ननम्नलिखित कथनों पर ववचार करें : d) 1, 2 और 3
1. वह
ृ त ् सिं लन', गुरुत्वाकर्वण के प्रभाव से ववतिंत्र है ।
2. प्राकृनतक वनवपनत का अिंधाधध Q.26) मि
ृ ा की उत्पवत्त के संिर्ा में ननम्नलिखित कथनों पर
ुिं ननष्ट्कासन, वह
ृ त्
सिं लन' का एक सकिय कारण है । ववचार करें :

3. वह 1. मद
ृ ा में ह्यूमस की मात्रा इसकी उववरता में बहुत
ृ त ् सिं लन' की प्रकिया अपरदन के अिंतगवत आती है ।
4. वह महत्वपूणव भूममका ननभाती है ।
ृ त ् सिं लन' के मलए अपक्षय एक पूवावपक्ष
े ा है ।
ऊपर दिए गए कथनों में से कौन-सा/से गित है/हैं? 2. मद
ृ ा एक मल
ू भत
ू अिंतरफलक के रूप में कायव करती है

a) केवल 1 और 3 जहााँ वायुमिंडल, वथलमिंडल, जलमिंडल और जीवमिंडल

b) केवल 1, 2 और 4 ममलते हैं।

c) केवल 1, 3 और 4 3. हजारों वर्ों से पहाडों के ननरिं तर अपक्षय से मद


ृ ा का

d) उपरोक्त सभी ननमावण होता है ।


ऊपर दिए गए कथनों में से कौन-सा/से सही है /हैं?
Q.24) र्ू-आकृनत प्रकियाओं के वह
ृ त ् संचिन' के संिर्ा में a) केवल 1 और 2
ननम्नलिखित कथनों पर ववचार करें : b) केवल 2
1. भू-प्रवाह (Earth Flow) , तीव्र वह
ृ त ् सिं लन' के प्रकार हैं। c) केवल 1 और 3
2. मलबा ,हहमवखलन बहाव की तुलना में बहुत तेज होता d) उपरोक्त सभी
है ।
3. अपक्षनयत पदाथों की मोटी परतें पानी से सिंतप्ृ त हो Q.27) लमट्टी के ननमााण के संबंि में ननम्नलिखित में से

जाती हैं और ननस्श् त ैनलों के साथ नी े बहती हैं जैसे कौन सा कारक सही नहीं है ?

की ड की धारा को मडफ्लो कहा जाता है । a) मूल सामग्री

ऊपर दिए गए कथनों में से कौन-सा/से गित है/हैं? b) वथलाकृनत और अपक्षय

a) केवल 1 c) जलवायु

b) केवल 2 d) मद
ृ ा सिंर ना

c) केवल 2 और 3
Q.28) अपरिन अनाच्छािात्मक प्रकियाओं के संिर्ा में ,
d) उपरोक्त सभी
ननम्नलिखित कथनों पर ववचार करें :

Q.25) र्ू-आकृनतक गनतववचि के रूप में र्ूस्ििन के संिर्ा 1. यह भौनतक रूप से कायव करता है लेककन रासायननक रूप

में , ननम्नलिखित कथनों पर ववचार करें : से नहीिं।

1. भूवखलन एक धीमी और बोधगम्य गनत है । 2. कटाव एजेंटों में बहता पानी, हवा, लहरें और धाराएिं,

2. पथ् ग्लेमशयर और बफव शाममल हैं।


ृ वी के मलबे का तेजी से लढ
ु कना या पीछे की ओर
घूमने के साथ खखसकना मलबा-वखलन कहलाता है । 3. इन प्रकियाओिं के मलए सौर ववककरण ऊजाव का अिंनतम

3. ढलान, रॉकवलाइड और रॉकफॉल सभी भूवखलन के स्रोत है ।

प्रकार हैं। 4. ववश्व वतर पर आने वाले ववककरण का लगभग 33%

ऊपर दिए गए कथनों में से कौन-सा/से सही है /हैं? हाइड्रोलॉस्जकल ि लाता है।

a) केवल 1 ऊपर दिए गए कथनों में से कौन-सा/से सही है /हैं?

b) केवल 3 a) केवल 1
b) केवल 1 और 2

DPP 2023 DAY 48 5


Contact us : info@onlyias.com

OnlyIAS Nothing Else Visit : dpp.onlyias.in


Contact : +91-7007 931 912

c) केवल 2 और 3 4. कृवर् उववरकों में फॉवफेट रॉक, पोटाश और न


ू े जैसे
d) केवल 2, 3 और 4 खननजों का उपयोग ककया जाता है ।
ऊपर दिए गए कथनों में से कौन-सा/से सही है /हैं?
Q.29) दहमािय और पस्चचमी घाट के संिर्ा में ननम्नलिखित a) केवल 1 और 3
कथनों पर ववचार करें : b) केवल 2 और 3
1. हहमालय पववत बेल्ट में गिंभीर भूकिंपीय गनतववचध के c) केवल 1 और 2
अधीन वववतवननक रूप से अस्वथर युवा भूवज्ञ
ै ाननक d) उपरोक्त सभी।
सिंर नाएिं शाममल हैं।
2. पस्श् मी घाट और नीलचगरी भग
ू भीय रूप से स्वथर हैं, Q.32) जिोढ़ पंिे के संिर्ा में ननम्नलिखित कथनों पर
लेककन नव-वववतवननक गनतववचध से प्रभाववत पठारी ववचार करें :
मास्जवन को ऊपर उठा क
ु े हैं। 1. जलोढ पिंखे तब बनते हैं जब उच् वतर की ओर बहने
3. पस्श् मी घाट क्षेत्र की तुलना में , हहमालय क्षेत्र में वखलन वाली धाराएाँ ननम्न प्रवणता वाले मैदानों में चगरती हैं।
ववशाल हैं। 2. पहाडी ढलानों पर बहने वाली धाराओिं द्वारा बहुत मोटे
ऊपर दिए गए कथनों में से कौन-सा/से सही है /हैं? नद्यभार का वहन ककया जाता है ।
a) केवल 1 3. शुष्ट्क क्षेत्रों में जलोढ पिंखे सामानय रूप से कम ढलान
b) केवल 1 और 2 वाले शिंकु हदखाते हैं जबकक आद्रव क्षेत्रों में वे खडी ढलान के
c) केवल 2 और 3 रूप में हदखाई दे ते हैं।
d) उपरोक्त सभी। ऊपर दिए गए कथनों में से कौन-सा/से गित है/हैं?
a) केवल 3
Q.30) 'डायस्रोकफज्म' की अन्तजाात र्ू-आकृनत प्रकियाओं b) केवल 1 और 2
के संिर्ा में ननम्नलिखित कथनों पर ववचार करें : c) केवल 1 और 3
1. ऑरोजेननक प्रकियाओिं में , प्लेट वववतवननकी होती है d) केवल 2 और 3
स्जसमें िवटल प्लेटों के क्षैनतज सिं लन शाममल होते हैं।
2. डायवरोकफज्म में पथ्
ृ वी की पपवटी में फॉल्ट और फ्रैक् र Q.33) ववसपा/मीएंडसा के संिर्ा में ननम्नलिखित कथनों पर
हो सकता है । ववचार करें :
3. एवपरोजेननक एक महाद्वीपीय ननमावण प्रकिया है । 1. ववसपव लप
ू जैसे पैटनव हैं जो बाढ और डेल्टा मैदानों पर
ऊपर दिए गए कथनों में से कौन-सा/से सही है /हैं? ववकमसत होते हैं।
a) केवल 1 और 2 2. इनमें उिल तट के साथ सकिय ननक्षेपण तथा अवतल
b) केवल 1 और 3 तट के साथ अपरदन होता है ।
c) केवल 2 और 3 3. मुडवा बबिंदओ
ु िं पर कट-ऑफ लप
ू को गोखुर झील कहा
d) उपरोक्त सभी। जाता है ।
ऊपर दिए गए कथनों में से कौन-सा/से सही है /हैं?
Q.31) िननजों के संिर्ा में ननम्नलिखित कथनों पर ववचार a) केवल 1
कीस्जएः b) केवल 1 और 3
1. इसकी एक व्यवस्वथत परमास्ववक सिंर ना है । c) केवल 2 और 3
2. इसकी एक ननस्श् त रासायननक सिंर ना और भौनतक d) उपरोक्त सभी
गुण हैं।
3. एक खननज एकल तत्व खननज हो सकता है ।

DPP 2023 DAY 48 6


Contact us : info@onlyias.com

OnlyIAS Nothing Else Visit : dpp.onlyias.in


Contact : +91-7007 931 912

Q.34) दहमनि अपरिन स्थिाकृनत सका के संिर्ा में , a) केवल 1


ननम्नलिखित में से कौन सा/से कथन गित है /हैं? b) केवल 2
1. वे गहरे , लिंबे और ौडे किंु ड या बेमसन हैं स्जनमें बहुत खडी c) केवल 1 और 2
अवतल हैं, जो इसके मसर के साथ-साथ ककनारों पर ऊिं ी d) उपरोक्त सभी।
दीवारों को लिंबवत रूप से चगराती हैं।
2. एक सकव मूल रूप से एक वी-आकार का अवसाद है जो Q.37) र्-ू आकृनतयों के ववकास में पवन के संिर्ा में

एक ग्लेमशयर की क्षरण गनतववचध से बनता है । ननम्नलिखित कथनों पर ववचार करें :


1. एडी, बविंडर, अपड्राफ्ट और डॉविंड्राफ्ट इसकी ववशेर्ताएिं
3. ूिंकक हहमनद केवल हहमरे खा के ऊपर ही उत्पनन हो
सकते हैं, वतवमान सकों के वथान का अध्ययन करने से हैं।

वपछले हहमनदी पैटनव और जलवायु पररवतवन की 2. ट्टानों की सतह से धूल और छोटे कणों का उठना,

जानकारी ममलती है । घर्वण है ।

नीचे दिए गए कूट का प्रयोग कर सही उत्तर चुननए- 3. मूसलाधार बाररश ,अपक्षनयत मलबे को हटाने में मदद

a) केवल 1 करती है ।

b) केवल 2 ऊपर दिए गए कथनों में से कौन-सा/से सही है /हैं?

c) केवल 1 और 3 a) केवल 1 और 2

d) केवल 2 और 3 b) केवल 2 और 3
c) केवल 1 और 3
Q.35) दहमनि अपरिन से ननम्नलिखित में से कौन-सी र्ू- d) ऊपर के सभी
आकृनतयााँ बनती हैं?
1. सकव Q.38) पेडडमें ट के संिर्ा में ननम्नलिखित कथनों पर ववचार

2. एरे त करें :

3. वपराममड ोहटयााँ 1. पेडडमें ट, आकार में अवतल होते हैं।

4. टानव 2. पेडडमें ट ,शुष्ट्क क्षेत्रों तक सीममत हैं।

नीचे दिए गए कूट का प्रयोग कर सही उत्तर चनु नए: 3. मुख्य वववतवननक िों के अिंत में पेडडमें ट्स का ननमावण

a) केवल 1 पथ्
ृ वी की पपवटी के ववकास के ऑरोजेननक रण से

b) केवल 1 और 3 प्लेटफॉमव रण तक सिंिमण के दौरान होता है ।

c) केवल 2 और 4 ऊपर दिए गए कथनों में से कौन-सा/से सही है /हैं?

d) उपरोक्त सभी। a) केवल 1 और 2


b) केवल 2 और 3
Q.36) चूना पत्थर की कंिराओं के ननमााण में र्ज
ू ि की c) केवल 1 और 3
र्लू मका के संिर्ा में ननम्नलिखित कथनों पर ववचार करें : d) ऊपर के सभी
1. ूना पत्थर के ववघटन से किंदराओिं का ननमावण होता है ।
2. भूजल रासायननक अपक्षय के माध्यम से क्षरण का Q.39) चट्टानों या िि
ु ी लमट्टी के ऊपर से अपक्षनयत मेंटि,

कारण बन सकता है । एक दिशा में हवा की िाराओं के िगातार चिने से फट जाता

3. वटै लेग्माइट्स और वटै लक्


े टाइट्स कुछ सबसे प्रमसद्ध है , क्या कहिाता है ?

किंदरा सिंर नाएिं हैं। a) अपवफीनत खोखले

ऊपर दिए गए कथनों में से कौन-सा/से सही है /हैं? b) पेडवटल ट्टानें


c) पेडडप्लेनस

DPP 2023 DAY 48 7


Contact us : info@onlyias.com

OnlyIAS Nothing Else Visit : dpp.onlyias.in


Contact : +91-7007 931 912

d) पेडडमें ट्स 2. डेल्टा


3. एवकसव
Q.40) मरुस्थिीय क्षेत्र में ननक्षेवपत र्ू-आकृनतयों के संिर्ा में 4. समुद्र तट
ननम्नलिखित कथनों पर ववचार करें : नीचे दिए गए कूट का प्रयोग कर सही उत्तर चुननए।
1. अनुप्रवथ टीले हवा की हदशा के समानािंतर सिंरेखखत होते a) केवल 1 और 2
हैं। b) केवल 1, 3 और 4
2. परवलनयक हटब्बा बर नों की तुलना में तुलनात्मक रूप c) केवल 2, 4
से बहुत व्यापक हैं। d) ऊपर के सभी
3. रे त की आपूनतव समद्
ृ ध होने पर अनुदैध्यव टीले बनते हैं।
ऊपर दिए गए कथनों में से कौन-सा/से गित है/हैं? Q.44) र्-ू आकृनत ववकास के चरणों के संिर्ा में
a) केवल 1 और 2 ननम्नलिखित कथनों पर ववचार करें :
b) केवल 2 और 3 1. लैंडफॉमव डेवलपमें ट के प्रारिं मभक रण में डाउनवडव
c) केवल 1 और 3 कहटिंग अचधक होती है ।
d) ऊपर के सभी 2. क्षैनतज अपरदन अचधक प्रभावी हो जाता है और भू-
आकृनत की पररपक्व अववथा में ऊध्वावधर अपरदन की
Q.41) घादटयों के संिर्ा में ननम्नलिखित कथनों पर ववचार तीव्रता कम हो जाती है ।
करें : 3. भू-आकृनत की पुरानी अववथा में जल-ववभाजन के ऊपर
1. गहरी, ौडी, लिंबी , पहाडडयािं एकजट
ु होकर घाहटयााँ तलछट और अपरदन अवशेर्ों की मोटी परतें बबखरी हुई
बनती हैं। हैं।
2. एक घाटी में खडे ढाल होते हैं। ऊपर दिए गए कथनों में से कौन-सा/से गित है/हैं?
3. घाटी क्षैनतज तलीय अवसादी ट्टानों में तथा घाहटयााँ a) केवल 1
कठोर ट्टानों में बनती हैं। b) केवल 1 और 2
ऊपर दिए गए कथनों में से कौन-सा/से सही है /हैं? c) केवल 2 और 3
a) केवल 2 d) इनमें से कोई भी नहीिं
b) केवल 1 और 2
c) केवल 2 और 3 Q.45) र्-ू आकृनतयों के बारे में ननम्नलिखित में से कौन
d) ऊपर के सभी सा/से कथन सही है/हैं?
1. वटै लेग्माइट और वटै लक्
े टाइट अिंततः अलग-अलग
Q.42) प्राकृनतक िेवे और पॉइंट बार ननम्नलिखित में से व्यास के वतिंभों का ननमावण करने के मलए सिंलग्न हो
ककस र्ू-आकृनतक शासन के एक ननक्षेपण स्थि हैं? जाते हैं।
a) हवाओिं 2. हहमाच्छाहदत पहाडों में सकवस सबसे आम भ-ू आकृनतयााँ
b) लहर की हैं।
c) नहदयों नीचे दिए गए कूट का प्रयोग कर सही उत्तर चुननए।
d) ग्लेमशयरों a) केवल 1
b) केवल 2
Q.43) ननम्नलिखित में से कौन-सा/से निी की छतों का
c) 1 और 2 दोनों
ननक्षेपण स्थिरूप है /हैं?
d) उपरोक्त में से कोई नहीिं
1. जलोढ पिंखे

DPP 2023 DAY 48 8


Contact us : info@onlyias.com

OnlyIAS Nothing Else Visit : dpp.onlyias.in


Contact : +91-7007 931 912

Q.46) र्ौनतक या यांत्रत्रक अपक्षय प्रकियाएं कुछ बिों पर a) ग्रेनाइट


ननर्ार करती हैं। ये बि हो सकते हैं – b) ममट्टी
1. गुरुत्वाकर्वण बल जैसे ओवरबडवन प्रेशर, लोड और c) क्वाट्वज
शीयररिंग वरे स। d) लवण
2. तापमान पररवतवन, किवटल वद्
ृ चध या पशु गनतववचध के
कारण वववतार बल। Q.50) ननम्नलिखित में से कौन सा/से कथन सही है/हैं?

3. पानी के दाब, को गीला करने और सुखाने के िों द्वारा 1. अपक्षय जन आिंदोलन के मलए एक पूवावपेक्षा है और जन

ननयिंबत्रत ककया जाता है । आिंदोलनों में सहायता करता है।

नीचे दिए गए कूट का प्रयोग कर सही उत्तर चनु नए: 2. यहद ट्टानें पारगम्य और घल
ु नशील हों और पानी

a) केवल 1 और 2 उपलब्ध हो तो ही कावटव वथलाकृनत ववकमसत होती है ।

b) केवल 2 और 3 नीचे दिए गए कूट का प्रयोग कर सही उत्तर चनु नए।

c) केवल 1 और 3 a) केवल 1

d) 1, 2 और 3 b) केवल 2
c) 1 और 2 दोनों
Q.47) ननम्नलिखित में से कौन सा/से कथन गित है /हैं? d) उपरोक्त में से कोई नहीिं
1. सीरे टेड ररज सककवल की दीवारों के मसर की ओर कटाव के
माध्यम से बनते हैं। Q.51) ननक्षेवपत र्ू-आकृनतयों के बारे में ननम्नलिखित में से

2. अपरदन रूप पस्श् मी तट पर हावी हैं और ननक्षेपण रूप कौन सा/से कथन सही है /हैं?

पूवी तट पर हावी हैं। 1. ड्रममलन च कने अिंडाकार आकार के ररज जैसी

नीचे दिए गए कूट का प्रयोग कर सही उत्तर चुननए। ववशेर्ताएिं हैं जो मुख्य रूप से हहमनदों से बनी होती हैं,

a) केवल 1 जब तक कक बजरी और रे त के कुछ द्रव्यमान होते हैं।

b) केवल 2 2. ब्रेडेड पैटनव के ननमावण के मलए बैंकों का जमाव और

c) 1 और 2 दोनों ऊध्वावधर क्षरण आवश्यक है ।

d) उपरोक्त में से कोई नहीिं नीचे दिए गए कूट का प्रयोग कर सही उत्तर चुननए।
a) केवल 1
Q.48) ननम्नलिखित में से कौन सा/से कथन गित है /हैं? b) केवल 2
1. सभी अिंतजावत भू-आकृनत प्रकियाएिं एक सामानय शब्द, c) 1 और 2 दोनों
अनाच्छादन के अिंतगवत आती हैं। d) उपरोक्त में से कोई नही
2. तापमान पररवतवन से प्रेररत वववतार और सिंकु न के
कारण अपशल्कन हो सकता है। Q.52) ननम्नलिखित में से कौन सा/से कथन सही है/हैं?

नीचे दिए गए कूट का प्रयोग कर सही उत्तर चनु नए। 1. शील्ड ज्वालामुखखयों की ववशेर्ता बेसाल्ट की तुलना में

a) केवल 1 कूलर और अचधक च पच पे लावा के प्रकोप से होती है ।

b) केवल 2 2. जब काल्डेरा फूटते हैं, तो वे ककसी भी सिंर ना का

c) 1 और 2 दोनों ननमावण करने के बजाय अपने आप चगर जाते हैं।

d) उपरोक्त में से कोई नहीिं नीचे दिए गए कूट का प्रयोग कर सही उत्तर चुननए।
a) केवल 1
Q.49) ननम्नलिखित में से कौन सी सामग्री जियोजन b) केवल 2
प्रकिया से प्रर्ाववत होती है? c) c)1 और 2 दोनों

DPP 2023 DAY 48 9


Contact us : info@onlyias.com

OnlyIAS Nothing Else Visit : dpp.onlyias.in


Contact : +91-7007 931 912

d) उपरोक्त में से कोई नही Q.56) प्िेट टे क्टोननक्स के संबंि में , ननम्नलिखित कथनों
पर ववचार करें -
Q.53) ननम्नलिखित कथनों पर ववचार करें : 1. प्लेटें स्वथर नहीिं होतीिं और आकाशमिंडल के ऊपर क्षैनतज
1. मलबा हहमवखलन हहमवखलन के समान होता है और रूप से गनत करती हैं।
यह की ड के बहाव की तुलना में बहुत तेज हो सकता है । 2. महाद्वीपीय ट्टानें महासागरीय िवट ट्टानों की
2. मडफ्लो अक्सर फटने वाले या हाल ही में वववफुहटत तुलना में बहुत छोटी होती हैं।
ज्वालामुखखयों की ढलानों पर होते हैं। 3. मध्य-महासागरीय कटक ज्वालामुखी वववफोट के
ऊपर दिए गए कथनों में से कौन-सा/से सही नहीं है /हैं? मामले में सबसे अचधक सकिय हैं।
a) केवल 1 ऊपर दिए गए कथनों में से कौन-सा/से गित है/हैं?
b) केवल 3 a) केवल 2
c) 1 और 2 दोनों b) केवल 1 और 3
d) न तो 1 और न ही 2 c) केवल 3
d) केवल 1 और 2
Q.54) ननम्नलिखित में से कौन सा/से कथन गित है /हैं?
1. बाढ बेसाल्ट प्रािंत ज्वालामख
ु ी अत्यचधक तरल लावा का Q.57) ननम्नलिखित में से कौन समुद्र ननति प्रसारण की
ननववहन करता है जो लिंबी दरू ी तक बहता है । घटना का सझ
ु ाव िे सकता है -
2. प्रमुख ढाल ज्वालामुखी ििंख
ृ ला प्रशािंत ररम हैं स्जसे 1. मशखा से दरू जाने पर ट्टानों का समयकाल(age)
“अस्ग्न ररम” के रूप में जाना जाता है । बढता है ।
नीचे दिए गए कूट का प्रयोग कर सही उत्तर चुननए। 2. एथेनोवफीयर के ऊपर क्षैनतज रूप से गनतमान प्लेट्स।
a) केवल 1 3. महासागरीय तल की तलछट महाद्वीपीय तल की
b) केवल 2 तुलना में बहुत पतली होती है ।
c) 1 और 2 दोनों 4. महासागरीय कटक के मशखर पर लगातार वववफोट होने
d) उपरोक्त में से कोई नहीिं से महासागरीय िवट को दोनों ओर धकेल हदया जाता
है ।
Q.55) ननम्नलिखित यग्ु मों पर ववचार कीस्जए-
नीचे दिए गए कूट का प्रयोग कर सही उत्तर चुननए-
प्िेट्स प्रकृनत/ववशेषताएं
a) केवल 1 और 3
1. 1-रािंसफॉमव बाउिं ड्रीज़ प्लेट्स के खीिं े जाने पर नई िवट
b) केवल 1 और 2
उत्पनन होती है
c) केवल 1, 3 और 4
2. अमभसारी सीमाएाँ जब एक प्लेट दस
ू रे के नी े आती हैं
d) ऊपर के सभी
तो िवट नष्ट्ट हो जाता है ।
3. अपसारी सीमाएिं, दरार घाहटयों और नए महासागरों का Q.58) ननम्नलिखित पर ववचार करें :
ननमावण करती हैं। 1. ध्रुव-भागने वाला बल
नीचे दिए गए कूट का प्रयोग कर सही उत्तर चनु नए- 2. प्लेसर जमा।
a) केवल 1 और 2 3. गरु
ु त्वाकर्वण
b) केवल 2 और 3 4. ज्वारीय बल।
c) केवल 1 और 3 5. पथ्
ृ वी के भीतर रे डडयोधमी धाराएाँ।
d) केवल 3

DPP 2023 DAY 48 10


Contact us : info@onlyias.com

OnlyIAS Nothing Else Visit : dpp.onlyias.in


Contact : +91-7007 931 912

टे क्टोननक मव
ू मेंट के लिए उपरोक्त में से कौन स्जम्मेिार हो b) 3 केवल
सकता है ? c) 2 तथा 3 केवल
a) केवल 1, 3, 4 और 5 d) 1 तथा 3 केवल
b) केवल 1, 2, 4 और 5
c) केवल 1, 2, 3, और 5 Q.62) प्िेट वववताननकी के संिर्ा में ननम्नलिखित कथनों

d) केवल 1, 2, 3, 4 और 5 पर ववचार कीस्जएः -


1. में टल में सिंवहन धारा प्लेट की गनत के पीछे प्रेरक
Q.59) नीचे दिए गए ननम्नलिखित कथनों पर ववचार करें - शस्क्तयों में से एक है ।
1. महाद्वीप एक प्लेट का हहवसा हैं। 2. मध्य-महासागर कटक क्षेत्र में भूकिंप के केंद्र गहराई में
2. समुद्र के बी की लकीरें और सीफ्लोर का फैलाव समुद्र होते हैं।
के वतर को प्रभाववत कर सकता है । 3. 3.महाद्वीपीय उत्थान मध्य-महासागरीय कटक का एक
3. समुद्र ननतल प्रसारण यह साबबत करता है कक महासागर भाग है
ही वववतवननक गनतववचध का वथल है । ऊपर दिए गए कथनों में से कौन-सा/से सही है /हैं?
ऊपर दिए गए कथनों में से कौन-सा/से सही है /हैं? a) 2 केवल
a) केवल 1 और 2 b) 2 तथा 3 केवल
b) केवल 2 और 3 c) 1 केवल
c) केवल 1 और 3 d) उपयुक्
व त सभी
d) ऊपर के सभी।
Q.63) ननम्नलिखित कथनों पर ववचार करें -
Q.60) समद्र
ु ति प्रसार के बारे में ननम्नलिखित कथनों पर 1. रािंसफॉमव प्लेट सीमाओिं में कोई ज्वालामख
ु ी गनतववचध
ववचार करें - नहीिं पाई जाती है ।
1. समुद्र तल प्रसार सभी मध्य-महासागरीय कटको पर 2. रसातल के मैदान व्यापक मैदान होते हैं जो महाद्वीपीय
एक समान होता है । मास्जवन और मध्य-महासागरीय कटक के बी होते हैं।
2. समुद्र तल प्रसार में टल सिंवहन का पररणाम है । 3. महासागर-महासागर अमभसरण के मामले में , पुरानी
ऊपर दिए गए कथनों में से कौन-सा/से गित है/हैं? प्लेट अपेक्षाकृत छोटी प्लेट के नी े धिंस जाती है ।
a) 1 केवल नीचे दिए गए कूट का प्रयोग कर सही उत्तर चनु नए -
b) 2 केवल a) 1 तथा 2 केवल
c) 1 तथा 2 दोनों b) 2 तथा 3 केवल
d) कोई नही c) 1 तथा 3 केवल
d) 2 केवल
Q.61) प्िेट संचिन के संिर्ा में , ननम्नलिखित कथनों पर
ववचार करें : - Q.64) हॉटस्पॉट्स के संिर्ा में ननम्नलिखित कथनों पर
1. प्लेट की गनत की दर सववत्र एक समान होती है । ववचार कीस्जएः -
2. समुद्र तल प्रसार अमभसरण प्लेट सीमाओिं पर होता है 1. प्लेट सीमाओिं के पास हॉटवपॉट ववकमसत होते हैं।
3. पथ्
ृ वी का ुिंबकीय इनतहास फैले हुए समुद्र तल में दजव 2. प्लेट सिं लन को रै क करने के मलए हॉट वपॉट का
है । उपयोग ककया जा सकता है
ऊपर दिए गए कथनों में से कौन-सा/से सही है /हैं? ऊपर दिए गए कथनों में से कौन-सा/से सही है /हैं?
a) 1 तथा 2 केवल a) 1 तथा 2 केवल

DPP 2023 DAY 48 11


Contact us : info@onlyias.com

OnlyIAS Nothing Else Visit : dpp.onlyias.in


Contact : +91-7007 931 912

b) 1 केवल a) 1 तथा 2 केवल


c) 2 केवल b) 2 तथा 3 केवल
d) कोई नही c) 1 तथा 3 केवल
d) उपयुक्
व त सभी
Q.65) महाद्वीपों और महासागरों के ववतरण के संिर्ा में ,
ननम्नलिखित कथनों पर ववचार कीस्जए: - Q.68) चट्टानों के संिर्ा में ननम्नलिखित कथनों पर ववचार
1. ममड-अटलािंहटक कटक आइसलैंड से होकर गुजरती है । कीस्जएः -
2. जापान एक ववनाशकारी प्लेट मास्जवन के पास स्वथत है । 1. प्लट
ू ोननक ट्टानें कुछ समय की अवचध के बाद
3. पूवी अफ्रीकी दरार घाटी एक ववकासशील अमभसरण तलछट के जमाव से बनती हैं।
प्लेट सीमा है . 2. कई पववत आग्नेय ट्टानों से बने हैं।
ऊपर दिए गए कथनों में से कौन-सा/से सही है /हैं? 3. पथ्
ृ वी की सतह पर वपघले हुए मैग्मा के ठिं डा होने से
a) 3 केवल बहहमुख
व ी आग्नेय ट्टानें बनती हैं।
b) 1 तथा 2 केवल ऊपर दिए गए कथनों में से कौन-सा/से गित है/हैं?
c) 2 तथा 3 केवल a) 1 तथा 3 केवल
d) 1 तथा 3 केवल b) 2 तथा 3 केवल
c) 1 केवल
Q.66) िननज ववतरण के संिर्ा में ननम्नलिखित कथनों पर d) 2 केवल
ववचार कीस्जए-
1. वववतवननक प्रकियाएिं खननज ननक्षेपों के ववतरण को Q.69) राष्ट्रीय बाि अचिकार संरक्षण आयोग
समझने में सहायता करती हैं। (एनसीपीसीआर) के संिर्ा में , ननम्नलिखित कथनों पर
2. उिर पूवी प्रायद्वीपीय क्षेत्र भारत के सबसे कम खननज ववचार करें ::
वाले क्षेत्रों में से एक है । 1. एनसीपीसीआर न तो सिंववधाननक और न ही वैधाननक
3. मैग्नेटाइट सबसे अच्छा लौह अयवक है स्जसमें लगभग ननकाय है , इसे केंद्र सरकार के कायवकारी आदे श द्वारा
70 प्रनतशत धास्त्वक सामग्री पाई जाती है । बनाया गया है ।
ऊपर दिए गए कथनों में से कौन-सा/से सही है /हैं? 2. आयोग 0 से 16 वर्व आयु वगव के व्यस्क्त को बच् े के रूप
a) 1 तथा 2 केवल में पररभावर्त करता है ।
b) 2 तथा 3 केवल 3. एनसीपीसीआर अपराधी के खखलाफ मक
ु दमा लाने के
c) 1 केवल मलए सिंबिंचधत सरकार से सिंपकव कर सकता है ।
d) 3 केवल ऊपर दिए गए कथनों में से कौन-सा/से सही है /हैं?
a) 1, 2 तथा 3
Q.67) चट्टान चि के बारे में ननम्नलिखित कथनों पर b) 3 केवल
ववचार करें - c) 1 तथा 2 केवल
1. किवटलीकरण और अवसादन ट्टान ि की कुछ d) 2 केवल
प्रमुख प्रकियाएाँ हैं।
2. आग्नेय ट्टानों को कायािंतररत ट्टानों में बदला जा Q.70) अनुिान की अनुपूरक मांगों के संबि
ं में ननम्नलिखित
सकता है । कथनों पर ववचार करें :
3. ट्टान ि तापमान और दाब पर ननभवर है । 1. सिंववधान का अनुच्छे द 115 अनुपूरक, अनतररक्त या
ऊपर दिए गए कथनों में से कौन-सा/से सही है /हैं? अचधक अनुदान की बात करता है ।

DPP 2023 DAY 48 12


Contact us : info@onlyias.com

OnlyIAS Nothing Else Visit : dpp.onlyias.in


Contact : +91-7007 931 912

2. ये अनद
ु ान सिंसद द्वारा वविीय वर्व की शरु
ु आत से पहले c) 1, 2 और 3
यानी बजट सत्र में प्रवतुत ककए जाते हैं। d) केवल 1 और 3
3. भारत के ननयिंत्रक-महालेखापरीक्षक सिंसद में अनुदान
ज्यादनतयों की मािंग के सिंबध
िं में नोहटस लाते हैं। Q.73) राष्ट्रीय अनुसूचचत जनजानत आयोग के संिर्ा में

ऊपर दिए गए कथनों में से कौन-सा/से सही है /हैं? ननम्नलिखित कथनों पर ववचार कीस्जएः

a) 1 तथा 3 केवल 1. अनुच्छे द 338A NCST को STs को प्रदान ककए गए ककसी

b) 3 केवल भी कानून के तहत ववमभनन सरु क्षा उपायों के

c) 1 तथा 2 केवल कायावनवयन की ननगरानी करने की शस्क्त दे ता है ।

d) 1, 2 तथा 3 2. सिंववधान में कहा गया है कक इसकी सिंर ना और ताकत


सिंसद द्वारा ननधावररत की जाएगी।
Q.71) संवविान के अनस
ु ार, ननम्नलिखित में से कौन से 3. NCST राष्ट्रपनत को वावर्वक रूप से और ऐसे अनय समय
मौलिक अचिकार वविे लशयों और नागररकों िोनों के लिए पर जैसा आयोग उच त समझे एक ररपोटव प्रवतुत करता
उपिब्ि है ? है ।
1. कानून के समक्ष समानता और कानूनों का समान ऊपर दिए गए कथनों में से कौन-सा/से सही नहीं है /हैं?
सिंरक्षण। a) केवल 1
2. आिंदोलन, ननवास और पेशा। b) केवल 2 और 3
3. प्रारिं मभक मशक्षा का अचधकार। c) केवल 2
4. भार्ा, मलवप और उनकी सिंवकृनत का सिंरक्षण d) केवल 1 और 3
नीचे दिए गए कूट का प्रयोग कर सही उत्तर चुननए:
a) 1, 2 तथा 3 केवल Q.74) उचचत आवास के लसद्िांतों के संबंि में ननम्नलिखित

b) 1 तथा 3 केवल कथनों पर ववचार करें :

c) 2, 3 तथा 4 केवल 1. यह मसद्धािंत ववकलािंग व्यस्क्तयों को उनकी भागीदारी

d) 2 तथा 4 केवल को सुववधाजनक बनाने के मलए अनतररक्त सहायता


प्रदान करता है ।
Q.72) 1973 की आपराचिक प्रकिया संदहता की िारा 144 के 2. कायववथल समायोजन के माध्यम से ववववधता और
संबंि में ननम्नलिखित कथनों पर ववचार करें : समावेश को बढावा दे ने के हदशाननदे श में ए आईवी और
1. CRPC की धारा 144 के उल्लिंघन पर अचधकतम सजा एड्स से ग्रमसत िममकों को िममकों की िेखणयों में
एक साल है । शाममल नहीिं ककया गया है ।
2. धारा 144 अत्यावश्यक मामलों में लगाई जाती है स्जनमें 3. ववकलािंग लोगों के अचधकार अचधननयम, 2016, धारा 2
मानव जीवन या सिंपवि को परे शानी या क्षनत पहुिं ाने की (H) में 'भेदभाव' को पररभावर्त करते हैं स्जसमें 'उच त
सिंभावना होती है । आवास से इनकार' शाममल है ।
3. कफ्यू,व धारा 144 से अलग है , क्योंकक यह लोगों को एक ऊपर दिए गए कथनों में से कौन-सा/से सही है /हैं?
ननस्श् त अवचध के मलए घर के अिंदर रहने का आदे श a) 1, 2 और 3
दे ता है । b) केवल 2 और 3
ऊपर दिए गए कथनों में से कौन-सा/से सही है /हैं? c) केवल 1 और 3
a) केवल 1 और 2 d) केवल 1 और 2
b) केवल 2 और 3

DPP 2023 DAY 48 13


Contact us : info@onlyias.com

OnlyIAS Nothing Else Visit : dpp.onlyias.in


Contact : +91-7007 931 912

Q.75) फॉरे नसा दरब्यन


ू ि( वविे शी अचिकरण) के संबंि में प्रोजेक्ट, जो चचाा में था, ककसके द्वारा ववकलसत ककया गया
ननम्नलिखित कथनों पर ववचार करें : है :
1. फॉरे नसव हरब्यूनल फॉरे नसव एक्ट, 1946 के तहत गहठत a) कानूनी मामलों के मिंत्रालय
एक अधव-नयानयक ननकाय है । b) सािंस्ख्यकी और कायविम कायावनवयन मिंत्रालय
2. सभी राज्य और केंद्र शामसत प्रदे श यह तय करने के मलए c) सिंसदीय कायव मिंत्रालय
नयायाचधकरण वथावपत कर सकते हैं कक भारत में अवैध d) गह
ृ मिंत्रालय
रूप से रहने वाला व्यस्क्त ववदे शी है या नहीिं।
3. केवल राज्य प्रशासन ही ककसी सिंहदग्ध के खखलाफ Q.79) राज्यपाि के पि के संबि
ं में ननम्नलिखित कथनों पर

हरब्यन ववचार करें :


ू ल में जा सकता है ।
ऊपर दिए गए कथनों में से कौन-सा/से सही है /हैं? 1. राज्यपाल राज्य का सिंवैधाननक प्रमुख होता है , जो उस

a) केवल 1 और 2 राज्य के मिंबत्रपररर्द की सलाह से बाध्य होता है ।

b) केवल 1 और 3 2. जब कोई पाटी बहुमत में नहीिं होती है राज्यपाल के पास

c) केवल 2 और 3 सबसे बडी पाटी के नेता को सरकार बनाने के मलए

d) 1, 2 और 3 आमिंबत्रत करने का सिंवध


ै ाननक वववेक है ,
3. नबाम रे बबया फैसले (2016) में सुप्रीम कोटव ने कहा कक
Q.76) हाि ही में िीगि एड (कानन
ू ी सहायता)स्क्िननक राज्यपाल के वववेक का प्रयोग अनच्
ु छे द 163 सीममत है
चचाा में था, ककसके द्वारा शुरू ककया गया है : और उसकी कारव वाई का ववकल्प मनमाना या
a) ववश्व ववावथ्य सिंगठन काल्पननक नहीिं होना ाहहए।
b) राष्ट्रीय महहला आयोग ऊपर दिए गए कथनों में से कौन-सा/से सही है /हैं?
c) हदल्ली राज्य कानूनी सेवा प्राचधकरण a) केवल 1 और 2
d) दोनों (B) और (C b) 1, 2 और 3
c) केवल 2 और 3
Q.77) केंद्रीय अन्वेषण ब्यूरो के संबंि में ननम्नलिखित d) केवल 1 और 3
कथनों पर ववचार करें :
1. सीबीआई सरकारी अचधकाररयों से जुडे भ्रष्ट्टा ार और Q.80) हाि ही में इंडडयन िीगि सववासेज(र्ारतीय कानूनी
हहिंसक अपराध की घटनाओिं के मामले में ववत: सिंज्ञान सेवा) बनाने की मांग चचाा में है। ननम्नलिखित में से बाहरी
ले सकती है । िोगों को महासचचव के रूप में ननयुक्त करने की वतामान
2. केंद्रीय जािं ब्यूरो काममवक ववभाग, काममवक, पें शन और प्रथा से संबचं ित चचंताएं कौन सी हैं?
लोक मशकायत मिंत्रालय के अधीन कायव करता है । 1. शस्क्तयों के पथ
ृ क्करण के मसद्धािंत के ववरुद्ध है ।
ऊपर दिए गए कथनों में से कौन-सा/से सही है /हैं? 2. ववधानयका के कामकाज की जानकारी का अभाव है ।
a) केवल 1 3. कायवपामलका को जवाबदे ह ठहराने में ववधानयकाओिं की
b) केवल 2 भूममका।
c) 1 और 2 दोनों नीचे दिए गए कूट का प्रयोग कर सही उत्तर चुननए:
d) न तो 1 और न ही 2 a) केवल 1 और 2
b) 1, 2 और 3
Q.78) हाि ही में "नेशनि ई-वविान एस्प्िकेशन (नेवा c) केवल 2 और 3
NeVA)", डडस्जटि वविानयकाओं के लिए एक लमशन मोड d) केवल 1 और 3

DPP 2023 DAY 48 14


Contact us : info@onlyias.com

OnlyIAS Nothing Else Visit : dpp.onlyias.in


Contact : +91-7007 931 912

Q.81) इिेक्रॉननकिी रांसलमटे ड पोस्टि बैिेट लसस्टम( ऊपर दिए गए कथनों में से कौन-सा/से सही नहीं है /हैं?
इिेक्रॉननक रूप से प्रेवषत पोस्टि बैिट लसस्टम )(ETPBS) a) केवल 1
के संबंि में ननम्नलिखित कथनों पर ववचार करें : b) केवल 2
1. ETPBS में डाक मतपत्र डाक द्वारा पिंजीकृत सेवा c) 1 और 2 दोनों
मतदाताओिं को मतदान के मलए भेजे जाते हैं। d) न तो 1 न ही
2. एक प्रवासी भारतीय उस ननवाव न क्षेत्र में मतदान कर
सकता है स्जसमें उसका ननवास वथान स्वथत है । Q.85) ‘अनुसूचचत जनजानत’ के संबंि में ननम्नलिखित

ऊपर दिए गए कथनों में से कौन-सा/से सही है /हैं? कथनों पर ववचार करें :

a) केवल 1 1. सिंववधान अनुच्छे द 366 के तहत अनुसूच त जनजानतयों

b) केवल 2 की मानयता के मानदिं ड को पररभावर्त करता है ।

c) 1 और 2 दोनों 2. पािं वीिं अनस


ु ू ी असम, मेघालय, बत्रपरु ा और ममजोरम

d) न तो 1 और न ही 2 में जनजातीय क्षेत्रों के प्रशासन से सिंबिंचधत है ।


3. डारलोंग समुदाय अरुणा ल प्रदे श के आहदवासी
Q.82) हाि ही में मॉस्न्रक्स अलर्समय ककससे संबंचित समुदायों से सिंबिंचधत है ।
समाचारों में िे िा गया था: ऊपर दिए गए कथनों में से कौन-सा/से सही नहीं है /हैं?
a) मीथेन प्रदर्
ू ण को कम करना a) केवल 1 और 3
b) घातक अपमशष्ट्ट की सीमा पार गनतववचधयों को b) 1, 2 और 3
ननयिंबत्रत करें । c) केवल 1 और 2
c) सभी समुद्री और समद्र
ु ी गनतववचधयों के मलए कानूनी d) केवल 2 और 3
ढािं ा।
d) जलडमरूमध्य का शासन। Q.86) र्ारत में अल्पसंख्यकों की स्स्थनत के संबि
ं में
ननम्नलिखित कथनों पर ववचार करें :
Q.83) हाि ही में , PM-DAKSH (िक्ष) (प्रिानमंत्री िक्ष और 1. भारतीय सिंववधान का अनुच्छे द 29 धाममवक
कुशि सम्पन्नता दहतग्राही) ककस उद्िे चय से चचाा में था: अल्पसिंख्यकों के साथ-साथ भार्ाई अल्पसिंख्यकों दोनों
a) ववकलािंग व्यस्क्तयों के मलए डडस्जटल लननिंग पोटव ल को सुरक्षा प्रदान करता है ।
b) वन लघु उपज की खरीद के मलए 2. 7वािं सिंवैधाननक (सिंशोधन) अचधननयम 1956 भार्ाई
c) लक्षक्षत समूहों के मलए कौशल ववकास प्रमशक्षण अल्पसिंख्यकों के मलए एक ववशेर् अचधकारी की
कायविम। ननयुस्क्त का प्रावधान करता है ।
d) महहलाओिं के मलए कौशल आत्मरक्षा प्रमशक्षण के मलए। 3. अल्पसिंख्यकों और उनके हहतों के सिंरक्षण का प्रावधान
सिंववधान की सातवीिं अनुसू ी की केंद्रीय सू ी में वखणवत
Q.84) गैरकानूनी गनतववचियां (रोकथाम) अचिननयम 1967 है ।
(यूएपीए) के संबंि में ननम्नलिखित कथनों पर ववचार करें :- ऊपर दिए गए कथनों में से कौन-सा/से सही है /हैं?
1. एक बार जब पमु लस ककसी व्यस्क्त पर यए
ू पीए के तहत a) केवल 1 और 3
आरोप लगाती है , तो जमानत दे ना बेहद मुस्श्कल हो b) केवल 2
जाता है । c) केवल 1 और 2
2. उच् तम नयायालय ने माना कक अदालतों को राज्य के d) 1, 2 और 3
मामले को उसकी गण
ु दोर् की जािं ककए बबना ववीकार
करना ाहहए।

DPP 2023 DAY 48 15


Contact us : info@onlyias.com

OnlyIAS Nothing Else Visit : dpp.onlyias.in


Contact : +91-7007 931 912

Q.87) आपातकाि के प्राविानों के संबंि में ननम्नलिखित 2. अचधननयम के अनस


ु ार, प्रनतरोध या वववरण दे ने से
कथनों पर ववचार करें : इनकार करना भारतीय दिं ड सिंहहता, 1860 के तहत
1. जब राज्य केंद्र के ककसी ननदे श का पालन करने या उसे अपराध माना जाएगा।
लागू करने में ववफल रहता है , तो राष्ट्रपनत सिंवैधाननक 3. इस अचधननयम ने केंद्र सरकार की ननयम बनाने की
आपातकाल की घोर्णा कर सकता है । शस्क्त का वववतार ककया।
2. राष्ट्रपनत शासन के पररणामववरूप, राज्य सरकार के ऊपर दिए गए कथनों में से कौन-सा/से सही है /हैं?
कायव और शस्क्तयािं सीधे राष्ट्रपनत में ननहहत होती हैं। a) केवल 1 और 2
3. अनुच्छे द 356 को अिंनतम और ननणावयक लागू करने में b) केवल 2 और 3
राष्ट्रपनत की सिंतस्ु ष्ट्ट का प्रावधान 44वें सिंववधान c) 1, 2 और 3
सिंशोधन अचधननयम, 1978 द्वारा हटा हदया गया है । d) केवल 1 और 3
ऊपर दिए गए कथनों में से कौन-सा/से सही है /हैं?
a) केवल 1 और 2 Q.90) ननम्नलिखित में से कौन पाइका ववद्रोह (1817-1825)

b) केवल 2 और 3 के कारण हैं?

c) 1, 2 और 3 1. बब्रहटश औपननवेमशक शासन की भ-ू राजवव व्यववथा के

d) केवल 1 और 3 कारण पाइकाओिं ने अपनी भमू म खो दी।


2. पाइकाओिं को नई मद्र
ु ा प्रणाली को सिंभालने की समवया
Q.88) 'नई जनगणना ननयम' के संबंि में ननम्नलिखित का सामना करना पड रहा था स्जसने मुद्रा को कौडी के
कथनों पर ववचार करें : रूप से रुपये में बदल हदया।
1. नया जनगणना ननयम आगामी जनगणना और राष्ट्रीय 3. बब्रहटश नमक नीनत उनहें नमक बनाने के उनके
जनसिंख्या रस्जवटर (एनपीआर) में ऑनलाइन वव- पारिं पररक अचधकार से विंच त कर दे ती है ।
गणना की अनुमनत दे ता है । नीचे दिए गए कूट का प्रयोग कर सही उत्तर चुननए:
2. प्रयोग के व्यापक प्र ार को सनु नस्श् त करने के मलए a) 1, 2 और 3
वप्रिंट मीडडया, रे डडयो, ऑडडयो-ववजुअल और पोवटर को b) केवल 2 और 3
प्रणाली की सू ी में जोडा गया है । c) केवल 1 और 2
ऊपर दिए गए कथनों में से कौन-सा/से सही है /हैं? d) केवल 1 और 3
a) केवल 1
b) केवल 2 Q.91) हाि ही में , सोहराई पें दटंग चचाा में थी, इसका संिर्ा है :

c) 1 और 2 दोनों a) दक्षक्षण भारत में हाथ से पें ट ककया हुआ कपडा

d) न तो 1 न ही 2 b) राजवथान में प्र मलत लघु कला


c) झारखिंड से च त्रकला का ववदे शी कला रूप।
Q.89) आपराचिक प्रकिया (पहचान) अचिननयम, 2022 के d) पस्श् म बिंगाल में सजावटी च बत्रत रे शम
संबंि में ननम्नलिखित कथनों पर ववचार करें :
1. अचधननयम में सिंग्रह की तारीख के 25 वर्ों से अचधक Q.92) सादहत्य अकािमी के संबंि में ननम्नलिखित कथनों

नहीिं के मलए एकत्र ककए गए वववरण को डडस्जटल रूप में पर ववचार करें :

बनाए रखने की आवश्यकता है। 1. साहहत्य अकादे मी द्वारा आयोस्जत एक समावेशी


साहहत्य उत्सव है ।

DPP 2023 DAY 48 16


Contact us : info@onlyias.com

OnlyIAS Nothing Else Visit : dpp.onlyias.in


Contact : +91-7007 931 912

2. साहहत्य अकादमी परु वकार 22 आचधकाररक भार्ाओिं में ऊपर दिए गए यग्ु मों में से कौन-सा/से सही है /हैं?
साहहस्त्यक कायों के मलए सालाना 22 पुरवकार प्रदान a) केवल 1
करता है । b) केवल 2 और 3
3. साहहत्य अकादमी एक सरकारी सिंवथान के रूप में c) केवल 2
सिंवकृनत मिंत्रालय के अधीन काम करती है । d) केवल 1 और 3
ऊपर दिए गए कथनों में से कौन-सा/से सही है /हैं?
a) केवल 1 Q.95) संतािी र्ाषा के संिर्ा में ननम्नलिखित कथनों पर

b) केवल 2 और 3 ववचार कीस्जएः

c) केवल 1 और 2 1. सन ् 1925 में सिंताली मलखने के मलए पिंडडत रघुनाथ मुमूव

d) केवल 3 ने ‘ओल च की’ मलवप का आववष्ट्कार ककया।


2. सिंताली भार्ा को 71वें सिंशोधन अचधननयम 1992 द्वारा
Q.93) त्रिदटश र्ारत के शासन के तहत समान लशक्षा के सिंववधान की आठवीिं अनुसू ी में जोडा गया है ।
अवसर सुननस्चचत करने में उनके महत्वपूणा योगिान के ऊपर दिए गए कथनों में से कौन-सा/से सही है /हैं?
लिए पहिी पीढ़ी की र्ारत की आिुननक नारीवादियों में से a) केवल 1
एक के रूप में । वह 1848 में र्ारत में पहिी मदहिा लशक्षक्षका b) केवल 2
बनीं और उन्होंने िडककयों के लिए एक स्कूि िोिा और c) 1 और 2 दोनों
उन्होंने एक अमेररकी लमशनरी द्वारा संचालित संस्थान में d) न तो 1 न ही 2
लशक्षक प्रलशक्षण पाठ्यिम र्ी लिया। उन्होंने “बािहत्या
प्रनतबंिक गह Q.96) ननम्नलिखित कथनों पर ववचार करें :
ृ ” र्ी स्थावपत ककया। अपने पनत के साथ,
उन्होंने पुणे में नेदटव फीमेि स्कूि और महारों की लशक्षा को 1. BIMSTEC एक क्षेत्रीय बहुपक्षीय सिंगठन है जो दक्षक्षण

बढ़ावा िे ने के लिए सोसाइटी की शुरुआत की। यह और दक्षक्षण पूवव एमशया को जोडता है ।

ननम्नलिखित में से ककस व्यस्क्तत्व के लिए सबसे अच्छा 2. बािंग्लादे श, भट


ू ान, भारत, नेपाल, िीलिंका, म्यािंमार और

वववरण है? थाईलैंड बबम्सटे क के सदवय हैं।

a) रामाबाई पिंडडत 3. बबम्सटे क लगभग 1.5 बबमलयन लोगों का घर है जो

b) महरीबाई टाटा वैस्श्वक आबादी का लगभग 22% हहवसा हैं।

c) सरोस्जनी नायडू ऊपर दिए गए कथनों में से कौन-सा/से सही है /हैं?

d) साववत्रीबाई फुले a) केवल 2


b) केवल 1 और 2
Q.94) र्ारत के संगीत वाद्ययंत्रों के संबि
ं में ननम्नलिखित c) केवल 2 और 3
युग्मों पर ववचार करें : d) केवल 1, 2 और 3
वाद्ययंत्र वववरण
1. घन वाद्य वव-थरथानेवाला, उनकी Q.97) एलशयन इन्रास्रक्चर इन्वेस्टमें ट बैंक (AIIB) के

लो दार प्रकृनत के कारण संबंि में ननम्नलिखित कथनों पर ववचार करें :

2. टाट वाद्य जानवरों की त्व ा से 1. एआईआईबी का ममशन एमशया में सामास्जक और

उत्पनन (वाद्य) टकराकर ध्वनन उत्पनन होती है आचथवक पररणामों में सध


ु ार करना है ।

3. अवनद्ध: वाद्य ध्वनन एक तार या जीवा के 2. ीन सबसे बडा शेयरधारक है और भारत के पास सबसे

किंपन से उत्पनन होती है । कम।

DPP 2023 DAY 48 17


Contact us : info@onlyias.com

OnlyIAS Nothing Else Visit : dpp.onlyias.in


Contact : +91-7007 931 912

3. एमशया इिंफ्रावरक् र बैंक के बोडव ऑफ गवनवसव में प्रत्येक Q.100) कृवष ववकास के लिए अंतरााष्ट्रीय कोष के संिर्ा में,
सदवय दे श द्वारा ननयुक्त दो गवनवर होते हैं। ननम्नलिखित कथनों पर ववचार करें :
ऊपर दिए गए कथनों में से कौन-सा/से सही है /हैं? 1. यह एक ववशेर् सिंयक्
ु त राष्ट्र एजेंसी है ।
a) केवल 1 2. यह लोगों को अपने कारोबार का वववतार करने में मदद
b) केवल 1 और 2 करता है ।
c) केवल 2 और 3 3. यह हर साल एक ग्रामीण ववकास ररपोटव प्रकामशत करता
d) 1, 2 और 3 है ।
ऊपर दिए गए कथनों में से कौन-सा/से सही है /हैं?
Q.98) मोटर वाहन समझौते के संबंि में ननम्नलिखित a) केवल 1 और 3
कथनों पर ववचार करें : b) केवल 1 और 2
1. हाल ही में भारत, नेपाल और बािंग्लादे श ने मोटर वाहन c) केवल 2 और 3
समझौते के कायावनवयन को अिंनतम रूप हदया। d) 1, 2 और 3
2. इसका उद्दे श्य लोगों से लोगों के बी सहज सिंपकव प्रदान
करना और आचथवक सिंपकव को बढाना है ।
3. आमसयान ववकास बैंक बािंग्लादे श-भूटान-भारत-नेपाल
मोटर वाहन समझौते की पहल के मलए तकनीकी,
सलाहकार और वविीय सहायता प्रदान करता रहा है ।
नीचे दिए गए कूट का प्रयोग कर सही उत्तर चुननए:
a) केवल 1, 2 और 3
b) केवल 3 और 4
c) केवल 1 और 2
d) केवल 2, 3 और 4

Q.99) र्ारत-ऑस्रे लिया संबि


ं ों के संिर्ा में , ननम्नलिखित
कथनों पर ववचार कीस्जएः
1. ऑवरे मलया भारत का 9वािं सबसे बडा व्यापाररक
भागीदार है ।
2. दोनों दे श क्वाड, रीजनल फोरम, एमशया पैमसकफक
पाटव नरमशप ऑन क्लाइमेट एिंड क्लीन डेवलपमें ट और
आमसयान के सदवय हैं।
3. भारत और ऑवरे मलया ने 2014 में असैनय परमाणु
सहयोग समझौते पर हवताक्षर ककए थे।
ऊपर दिए गए कथनों में से कौन-सा/से सही है /हैं?
a) केवल 2 और 3
b) केवल 1 और 2
c) केवल 2 और 3
d) 1, 2 और 3

DPP 2023 DAY 48 18


Contact us : info@onlyias.com

OnlyIAS Nothing Else Visit : dpp.onlyias.in


Contact : +91-7007 931 912

Q.1) Ans: B
Exp:
वलन(folds) के बारे में
• वलन एक लहरदार संरचना (लहर जैसी) है जो तब
बनती है जब चट्टानें या पथ्
ृ वी की पपडी का एक
हहस्सा संकुचचत दबाव के तहत मुडा हुआ (झक
ु ने से
ववकृत) होता है । वलन कई स्तरों (रॉक लेयसस) से बनी Q.2) Ans: B
होती हैं। Exp:
• ऊपर की ओर उत्तल(उत्थान) वलन को एंटीकलाइन अव ादी चट्टानों के बारे में :

कहा जाता है । एक एंटीलाइन वलन के कोर (केंद्र) में • कथन 1 ही है : आग्नेय और कायांतररत चट्टानों

पुरानी परते होती हैं, और बाहर की तरफ नई परते के क्षरण और जमाव के कारण पथ्
ृ वी की सतह पर
होती है । बनने वाली चट्टानों को अवसादी/तलछटी चट्टानें

• इसके ववपरीत, जो वलन नीचे की ओर उत्तल होती हैं, कहा जाता है । तलछटी चट्टानें स्तररत या स्तरीकृत

स क
ं लाइन कहलाती हैं। एक ससंकलाइन वलन के चट्टानें होती हैं। इन चट्टानों के ननमासण में

मूल में नई परते होती हैं, और बाहर की तरफ पुरानी काबसननक पदाथस भी बडी भसू मका ननभाते हैं।

परते होती हैं। o तलछटी चट्टानें पथ्


ृ वी की सतह के लगभग
75% क्षेत्र में पाई जाती हैं लेककन वे पथ्
ृ वी की
वलन के प्रकार पपडी के आयतन का लगभग 5% ही बनाती
• मसमत वलन वह होता है जजसमें अक्षीय तल हैं। प्रकार के आिार पर तलछटी चट्टानों
लंबवत होता है । उदा. जस्वट्जरलैंड के जरु ा पवसत. की संरचना: शेल (80%), बलुआ पत्थर
• अ मसमत तह वह है जजसमें अक्षीय तल झुका होता (12%), चूना पत्थर (8%).
है । उदा. ब्रिटे न के दक्षक्षणी पेनीन पवसत o तलछट की प्रकृतत के आधार पर तलछटी
• इसोक्लीनल बालन में ऐसे अवयव/शाखा होते हैं चट्टानों को तनम्न तरीके े वर्गीकृत ककया
जो अननवायस रूप से एक दस
ू रे के समानांतर होते हैं जाता है :
और इस प्रकार अक्षीय तल के लगभग समानांतर ▪ यांब्रत्रक रूप से ननसमसत तलछटी
होते हैं। उदा. पाककस्तान में कालाचचंता पवसत चट्टानें जैसे सैंडस्टोन,
• उथल पथ
ु ल वलन में अत्यचिक झुकाव वाला अक्षीय कांग्लोमेरेट्स, क्ले रॉक्स, शेल
तल होता है , जजससे एक तरफ/शाखा की परत उलट और लोस।
जाती है । उदा. भारत में पीर पंजाल रें ज ▪ काबसननक रूप से ननसमसत तलछटी
• एक लेटे हुए वलन में एक अननवायस रूप से क्षैनतज चट्टानें जैसे चूना पत्थर, कोयला,
अक्षीय तल होता है । उदा. ब्रिटे न के कैररक कैसल पीट।
पवसत ▪ रासायननक रूप से ननसमसत
• मोनोजक्लनल वलन में दो क्षैनतज (या लगभग इतने तलछटी चट्टानें जैसे चाक
ही) अवयव होते हैं जो एक छोटे झुकाव वाले अवयव चट्टानें, जजप्सम और साल्ट रॉक।
से जुडे होते हैं। उदा. ऑस्रे सलया की ग्रेट डडवाइडडंग • कथन 2 र्गलत है : आचथसक मूल्य के खननज अवसादी
रें ज। चट्टानों में कम होते हैं। लेककन ये सीमें ट बनाने के
सलए हे मेटाइट लौह अयस्क, फॉस्फेट, कई ब्रबजल्डंग
स्टोन, कोयला और खननजों के स्रोत हैं। छोटे समुद्री

DPP 2023 DAY 48 19


Contact us : info@onlyias.com

OnlyIAS Nothing Else Visit : dpp.onlyias.in


Contact : +91-7007 931 912

जीवों के क्षय से बनने वाला पेरोसलयम भी तलछटी o समचित ज्वालामख


ु ी सभी ज्वालामखु खयों में
चट्टानों की समट्टी में पाया जाता है । सबसे सुरम्य हो सकते हैं। एक क्लाससक
• कथन 3 ही है : तलछटी चट्टानें कुछ समद्
ृ ि समचित ज्वालामुखी एक अवतल आकार के
समट्टी भी बनाती हैं। भारतीय उपमहाद्वीप में , इंडो- साथ शंक्वाकार होता है जो शीर्स के ननकट
गंगा के मैदान में जलोढ़ ननक्षेप तलछटी संचय के ही अचिक ढलान वाला होता है । इन पहाडों में
हैं।इस मैदान में और हमारे दे श के पूवी तटीय मैदानों आमतौर पर बफस से ढकी चोहटयााँ होती हैं जो
में ववसभन्न प्रकार की समट्टी और दोमट पाई जाती आसपास के पहाडी इलाकों के ऊपर ऊाँची
है । बलुआ पत्थर मध्य प्रदे श के बडे क्षेत्रों, पूवी खडी होती हैं। समचित शंकु बडे ज्वालामुखी
राजस्थान, हहमालय, आंध्र प्रदे श, ब्रबहार और (कई हजारों फीट या मीटर ऊंचे) होते हैं जो
ओडडशा के कुछ हहस्सों में पाया जाता है ।. आम तौर पर लावा प्रवाह, पायरोक्लाजस्टक
सन्दभस: Principles of Geography part 1 . class 11th Page जमा, और मडफ्लो (लाहर) ननक्षेप, साथ ही
number - 24 साथ लावा गुंबदों से बने होते हैं। समग्र
ज्वालामुखी लंबी अवचि (दससयों से सैकडों
Q.3) Ans: B हजारों वर्ों) में सकिय होते हैं, और समय-
Exp: समय पर फटते हैं। इन ज्वालामुखखयों में
ज्वालामुखी व इ के प्रकार के बारे में :
बेसाल्ट की तुलना में अचिक ठं डा और
• ज्वालामख
ु ी एक ऐसा स्थान होता है जहां गैस,ें राख
अचिक चचपचचपे लावा के ववस्फोट की
और/या वपघली हुई चट्टान सामग्री - लावा - जमीन
ववशेर्ता होती है । इन ज्वालामुखखयों के
पर आ जाती है । एक ज्वालामुखी को सकिय
पररणामस्वरूप अक्सर ववस्फोटक
ज्वालामख
ु ी कहा जाता है यहद उजल्लखखत सामग्री
ववस्फोट होते हैं। लावा के साथ, बडी मात्रा
हाल ही में ननकली हो/ननकलती रहती है ।
में पाइरोक्लाजस्टक सामग्री और राख
o ठोस पपसटी के नीचे की परत में टल होती है ।
जमीन पर अपना रास्ता बना लेती है । यह
इसका घनत्व पपसटी की तल
ु ना में अचिक
सामग्री वेंट के उद्घाटन के आसपास जमा
होता है । में टल में एक कमजोर क्षेत्र होता है
हो जाती है जजससे परतों का ननमासण होता
जजसे एस्थेनोस्फीयर कहा जाता है । यहीं से
है ।
वपघली हुई चट्टानें सतह पर अपना रास्ता
o शील्ड ज्वालामुखी: बेसाल्ट प्रवाह को
बना लेती हैं। मेंटल के ऊपरी हहस्से में
छोडकर, शील्ड ज्वालामख
ु ी पथ्ृ वी पर सभी
मौजद
ू पदाथस को मैग्मा कहते हैं। एक बार
ज्वालामुखखयों में सबसे बडे होते हैं। हवाई
जब यह पपसटी की ओर बढ़ना शुरू कर दे ता
ज्वालामुखी सबसे प्रससद्ि उदाहरण हैं। ये
है या सतह पर पहुंच जाता है , तो इसे लावा
ज्वालामख
ु ी ज्यादातर बेसाल्ट, एक प्रकार
कहा जाता है ।
का लावा जो फूटने पर बहुत तरल होता है ,
o जमीन तक पहुंचने वाली सामग्री में लावा
से बने होते हैं । इस कारण से, ये
प्रवाह, पायरोक्लाजस्टक मलबे, ज्वालामुखी
ज्वालामुखी ढलान वाले नहीं होते हैं। अगर
बम, राख और िूल और नाइरोजन यौचगक,
ककसी तरह पानी वेंट में चला जाता है तो वे
सल्फर यौचगक और थोडी मात्रा में क्लोरीन,
ववस्फोटक हो जाते हैं; अन्यथा, इनमें कम-
हाइड्रोजन और आगसन शासमल हैं।
ववस्फोटकता की ववशेर्ता होती है ।
• ज्वालामुखी के प्रकार:
ननकलने वाला लावा एक फव्वारे के रूप में

DPP 2023 DAY 48 20


Contact us : info@onlyias.com

OnlyIAS Nothing Else Visit : dpp.onlyias.in


Contact : +91-7007 931 912

ननकलकर शंकु को हटाकर को वेंट के शीर्स Q.4) Ans: A


Exp:
पर चगरता है और एक ससंडर कोन में
• कथन 1 ही है : अम्लीय चट्टानों में सससलका
ववकससत होता है ।
(क्वाट्सज और फेल्डस्पार) की उच्च मात्रा, 80 प्रनतशत
o काल्डेरा ज्वालामख
ु ी: ये पथ्
ृ वी के
तक होती है ।शेर् भाग एल्यसु मननयम, क्षार,
ज्वालामुखखयों में सबसे अचिक ववस्फोटक
मैग्नीसशयम, आयरन ऑक्साइड, चूना आहद में बंट
हैं। वे आमतौर पर इतने ववस्फोटक होते हैं
जाता है । सससलकॉन की अचिकता के कारण अम्लीय
कक जब वे फटते हैं तो वे ककसी भी ऊंचे ढांचे
मैग्मा तेजी से ठं डा होता है , और यह प्रवाहहत नहीं
के ननमासण के बजाय खद
ु ही चगर जाते हैं।
होता है और दरू तक नही फैलता है । इस प्रकार की
ढह गए अवसादों को काल्डेरा कहा जाता है ।
चट्टान से ऊंचे पहाड बनते हैं।
उनकी ववस्फोटकता इंचगत करती है कक
o जबकक क्षारीय चट्टानो में सससलका
लावा की आपूनतस करने वाला मैग्मा कक्ष न
(लगभग 40 प्रनतशत) होती हैं; मैग्नेसशया
केवल ववशाल है , बजल्क आसपास के क्षेत्र में
की मात्रा 40 प्रनतशत तक होती है , और शेर्
भी मौजूद है । उदाहरण के सलए, टोबा झील
आयरन ऑक्साइड, चूना, एल्युसमननयम,
(इंडोनेसशया) लगभग 75,000 साल पहले
क्षार, पोटे सशयम आहद में बट जाती है ।
अपने सुपर ज्वालामख
ु ीय ववस्फोट के बाद
सससलका की मात्रा कम होने के कारण, ऐसी
बनी थी। यह दनु नया की सबसे बडी िेटर
चट्टानों की मूल सामग्री िीरे -िीरे ठं डी होती
झील है ।
है और प्रवाहहत होकर दरू तक फैल जाती
o ववदर प्रकार का ज्वालामुखी: कभी-कभी,
है । यह प्रवाह और शीतलन पठारों को जन्म
एक बहुत पतला मैग्मा पथ् ृ वी की सतह में
दे ता है ।
नछद्रो और दरारों के माध्यम से ननकल जाता
• कथन 2 र्गलत है: अम्लीय चट्टानों में लौह और
है और कुछ कुछ समय के अंतराल पर लंबे
मैग्नीसशयम जैसे भारी खननजों की मात्रा कम होती
समय तक बहता है , एक ववशाल क्षेत्र में
है । इससलए, वे कम घनी होती हैं और क्षारीय चट्टानों
फैलता है , अंत में एक स्तररत, लहरदार
की तुलना में हल्के रं ग की होती हैं। ये चट्टानें िस्ट
(लहर जैसी), सपाट सतह का ननमासण करता
के ससयाल भाग का ननमासण करती हैं। जबकक क्षारीय
है । उदाहरण: साइबेररयन रै प, डेक्कन रै प,
चट्टानों में भारी तत्वों की उपजस्थनत होती है जो इन
स्नेक बेससन, आइसलैंडडक शील्ड,
चट्टानों को गहरा रं ग प्रदान करते हैं।
कैनेडडयन शील्ड।
• कथन 3 गलत है: अम्लीय चट्टानें कठोर, ठोस,
o मध्य-महा ार्गर कटक ज्वालामुखी: ये
ववशाल और अपक्षय के प्रनतरोिी होती हैं। ग्रेनाइट,
ज्वालामुखी महासागरीय क्षेत्रों में पाए जाते
क्वाट्सज और फेल्डस्पार ववसशष्ट उदाहरण हैं जबकक
हैं। 70,000 ककमी से अचिक लंबी मध्य-
क्षारीय चट्टानें बहुत कठोर नहीं होती हैं, इन चट्टानों
महासागर की कटको की एक प्रणाली है जो
को अपेक्षाकृत आसानी से बनाया जाता है । बेसाल्ट,
सभी महासागरीय घाहटयों में फैली हुई है ।
गैिो और डोलराइट इसके ववसशष्ट उदाहरण हैं।
इस ररज के मध्य भाग में बार-बार ववस्फोट
न्दर्भ: Geography by Mahesh kumar Barnwal.3rd
होते हैं। edition .Page no.-25
सन्दभस: Fundamentals of Physical Geography Ncert-
Class 11th Chapter- 3
Q.5) Ans: A
Exp:

DPP 2023 DAY 48 21


Contact us : info@onlyias.com

OnlyIAS Nothing Else Visit : dpp.onlyias.in


Contact : +91-7007 931 912

• ज्वालामख
ु ी ववस्फोट से कभी-कभी हीरे की खान
ज्वालामुखी ववस्फोट के बारे में जैसे कीमती पत्थरों का ननमासण होता है । उदहारण:
ज्वालामख
ु ी ववस्फोट से प्राप्त पदाथस तरल, ठोस और गैसीय दक्षक्षण अफ्रीका में ककम्बरले
प्रकृनत के होते हैं। तरल पदाथस लावा है । ठो सामग्री में महीन यह ज्वालामख
• ु ी पयसटन को बढ़ावा दे ता है क्योंकक
राख और िूल के कण और कोणीय टुकडे ज्यादातर लावा के सकिय ज्वालामुखखयों से आनतशबाजी दनु नया के
होते हैं, वेंट के भीतर से ननकली चट्टान आहद होते है । र्गै ीय ववसभन्न हहस्सों से कई पयसटकों को आकवर्सत करती
पदाथभ मुख्य रूप से भाप से बने हुए, हाइड्रोक्लोररक एससड है और ज्वालामुखी द्वीपों को एक लोकवप्रय पयसटन
उत्पाद, अमोननयम क्लोराइड, सल्फर डाइऑक्साइड, स्थल बनाती है ।
हाइड्रोजन सल्फाइड, हाइड्रोजन और काबसन डाइऑक्साइड
होते हैं।
Q.6) Ans: D
Exp:
ज्वालामुखी ववस्फोट के रचनात्मक प्रर्ाव:
• कथन 1 ही है : सभी प्राकृनतक भक
ू ंप
• कथन 1 सही है : ज्वालामख
ु ीय चट्टानें, अपक्षय और
स्थलमंडल(सलथोस्फीयर) में होते हैं। स्थलमंडल
अपघटन पर बहुत उपजाऊ समट्टी पैदा करती हैं।
पथ्
ृ वी की सतह से 200 ककमी तक की गहराई के
बार-बार होने वाले ववस्फोटों के खतरों के बावजद

हहस्से को संदसभसत करता है । 'सीस्मोग्राफ' नामक
ककसान अच्छी फसल पाने के सलए इसे जोतते रहते
उपकरण सतह पर पहुंचने वाली तरं गों को ररकॉडस
हैं।
करता है ।
• कथन 2 गलत है : ज्वालामख
ु ी की धल
ू और राख खेतों
o भूकंप तरं गें मूल रूप से दो प्रकार- भू गसभसक
और बर्गीचों के सलए बहुत उपजाऊ होती है , अगर यह
तरं गें और सतही तरं गेंहोती हैं। भू गसभसक
वनस्पनत को दफन या नष्ट नहीं करती है ।
तरं गें फोकस पर ऊजास की ववमुजक्त के
• कथन 3 सही है : उन क्षेत्रों में , जहां ज्वालामुखी
कारण उत्पन्न होती हैं और पथ्
ृ वी के
सकिय रहे हैं, गहराई में पानी गमस मैग्मा के संपकस से
आंतररक भाग के माध्यम से यात्रा करके
गमस होता है । कुछ स्थानों पर, छोटे -छोटे नछद्रो से
सभी हदशाओं में चलती हैं। शरीर की तरं गें
सामनयक अंतराल पर भाप और उबलते गमस पानी का
सतह की चट्टानों के साथ परस्पर किया
एक जेट उत्सजजसत होता है । ये र्गीजर होते हैं और
करती हैं और तरं गों के नए सेट उत्पन्न
इनकी उच्च तापमान की भाप को भू-तापीय ब्रबजली
करती हैं जजन्हें तही तरं र्गें कहा जाता है ।
उत्पन्न करने के सलए उपयोग ककया जाता है ।
भू गसभसक तरं गें दो प्रकार – P तरं ग व S तरं ग
• ज्वालामुखी वैजववक शीतलन प्रकिया में मदद करते
,की होती है ।
हैं। यह पथ्
ृ वी को उसके आंतररक भाग से गमी को दरू
o P-तरं गें ध्वनन तरं गों के समान अनद
ु ै र्घयस
करने के सलए ठं डा करने में मदद करता है । इसके
तरं गें होती हैं। चकूं क P-तरं गें तरं ग की हदशा
अलावा जब ज्वालामख
ु ी की राख और सल्फर
के समानांतर कंपन करती हैं। यह प्रसार की
डाइऑक्साइड जैसे यौचगक वायुमंडल में मुक्त होते
हदशा में सामग्री पर दबाव डालती है ।
है , तो यह सूयस की कुछ ककरणों को वापस अंतररक्ष में
नतीजतन, यह सामग्री में घनत्व अंतर पैदा
परावनतसत कर सकते है , जजससे वातावरण द्वारा
करती है जजससे सामग्री फैलती और
अवशोवर्त ऊष्मा ऊजास की मात्रा कम हो जाती है । इस
ससकुडती है । जबकक S तरं र्गें प्रकाश तरं र्गों के
प्रकिया को ग्लोबल डडसमंर्ग(dimming) कहा जाता है ।
अनुरूप अनुप्रस्थ तरं र्गें होती हैं। वे प्रसार
की हदशा के लंबवत कंपन करती हैं।

DPP 2023 DAY 48 22


Contact us : info@onlyias.com

OnlyIAS Nothing Else Visit : dpp.onlyias.in


Contact : +91-7007 931 912

इससलए, वे जजस सामग्री से गज


ु रती हैं
उसमें गतस और सशखर बनाती हैं।
• कथन 2 सही है : भूकंप की लहरें दरू के स्थानों पर
जस्थत सीस्मोग्राफ में दजस हो जाती हैं। हालांकक, कुछ
ववसशष्ट क्षेत्र मौजूद होता हैं जहां लहरों की सूचना दजस
नहीं की जाती है । ऐसे क्षेत्र को 'छाया क्षेत्र' कहा जाता
है ।
o S-तरं र्ग का छाया क्षेत्र P-तरं गों की तुलना में
• कथन 3 र्गलत है : पथ्
ृ वी ववज्ञान मंत्रालय के तहत
बहुत बडा होता है । P-तरं गों का छाया क्षेत्र
राष्ट्रीय र्क
ू ं प ववज्ञान केंद्र दे श में और उसके
उपररकेंद्र से 105° और 145° के बीच पथ्ृ वी
आसपास भूकंप की ननगरानी के सलए भारत सरकार
के चारों ओर एक बैंड के रूप में प्रकट होता है
(GoI) की नोडल एजेंसी है । इस उद्दे वय के सलए,
जबकक, 105° से आगे के पूरे क्षेत्र में S-तरं गें
एनसीएस दे श भर में फैले 115 वेिशालाओं से यक्
ु त
प्राप्त नहीं होती हैं। S-तरं गों का छाया क्षेत्र न
एक राष्रीय भक
ू ं पीय नेटवकस (एनएसएन) का
केवल ववस्तार में बडा है बजल्क यह पथ्
ृ वी की
रखरखाव करता है । पयासप्त शमन उपायों को शुरू
सतह के 40 प्रनतशत से थोडा अचिक है ।
करने के सलए एनसीएस द्वारा ररपोटस ककए गए
• कथन 3 ही है : तीन प्रकार की भूकंपीय तरं गों में P-
भूकंपों की सूचना को संबचं ित केंद्रीय और राज्य
तरं गों का वेग अचिकतम होता है । ये तरं गें ठोस, द्रव
आपदा प्राचिकरणों को कम से कम समय में प्रसाररत
और गैस तीनों माध्यमों से होकर गुजर सकती हैं।
ककया जा रहा है ।
जबकक S तरं गें केवल ठोस माध्यम से यात्रा कर
सकती हैं और तरल माध्यम में गायब हो जाती हैं।
चाँकू क ये तरं गें पथ्
ृ वी के कोर(केंद्र) से नहीं गुजरती हैं,
Q.8) Ans: D
इससलए ये कोर के तरल अवस्था में होने का अंदाजा Exp:
दे ती हैं। खबरों में रहे ज्वालामख
ु ी:
सन्दभस: Fundamentals of Physical Geography Ncert- • ताल ज्वालामख
ु ी
Class 11th Chapter- 3 o यह कफलीपींस के मनीला से 50 ककमी दरू
लज
ु ोन द्वीप पर जस्थत है ।
Q.7) Ans: A o कफलीपींस दो टे क्टोननक - कफलीपींस
Exp:
सागर प्लेट और यूरेसशयन प्लेट प्लेटों की
• कथन 1 और 2 ही है: भारत का कुल 59% भूभाग
सीमाओं पर जस्थत है - इस प्रकार भूकंप
(भारत के सभी राज्यों को शासमल करके) ववसभन्न
और ज्वालामख
ु ी के सलए
तीव्रता के भूकंपों से ग्रस्त है । दे श के भूकंपीय क्षेत्रीय
अनतसंवेदनशील है ।.
मानचचत्र के अनुसार, कुल क्षेत्रफल को चार भक
ू ं पीय
o ताल कफलीपींस में सबसे सकिय
क्षेत्रों में वगीकृत ककया गया है। जोन V भूकंपीय रूप
ज्वालामुखखयों में से एक है , क्योंकक यह
से सबसे सकिय क्षेत्र है , जबकक जोन II सबसे कम है ।
प्रशांत क्षेत्र "ररंग ऑफ फायर" - तीव्र
दे श का लगभग ~ 11% क्षेत्र V में , ~ 18% क्षेत्र IV में,
भूकंपीय गनतववचि का क्षेत्र ,पर जस्थत है ।
~ 30% क्षेत्र III में और शेर् क्षेत्र II में आता है ।
• ांर्गे ज्वालामुखी:

DPP 2023 DAY 48 23


Contact us : info@onlyias.com

OnlyIAS Nothing Else Visit : dpp.onlyias.in


Contact : +91-7007 931 912

o यह दनु नया के सबसे ऊंचे सकिय


ववस्फोट की आवचिकता के वगीकरण के आिार पर,
ज्वालामुखखयों में से एक है और इक्वाडोर
ज्वालामुखी को ननम्नसलखखत िेखणयों में ववभाजजत ककया जा
के सबसे सकिय ज्वालामुखखयों में से एक
सकता है ::
है ।
1. किय ज्वालामुखी: ये ज्वालामुखी लगातार
o यह सांगे राष्रीय उद्यान के भीतर
ज्वालामख
ु ी लावा, गैस, राख और खंडडत सामग्री को
जस्थत है ।
बाहर ननकालते रहते हैं। ऐसा अनुमान है कक दनु नया
o हाल ही में , इक्वाडोर में सांगे ज्वालामुखी
में 500 से अचिक ज्वालामख
ु ी हैं। उदा. भूमध्य
फट गया, जजससे आकाश में 8,500 मीटर
सागर के एटना और स्रोमबोली, इक्वाडोर के
की ऊंचाई तक राख के बादल फैल गए।
कोटोपैक्सी आहद।
• फेद द्वीप:
2. तनष्क्ष्ट्िय ज्वालामख
ु ी: ये ज्वालामख
ु ी कुछ समय के
o न्यूजीलैंड में सफेद द्वीप जजसे व्हाकारी के
सलए अपने ववस्फोट के बाद शांत हो जाते हैं और
नाम से भी जाना जाता है , उत्तरी द्वीप के
भववष्य में ववस्फोट के कोई संकेत नहीं होते हैं लेककन
पूवी तट से लगभग 50 ककमी दरू एक क्षेत्र में
अचानक ये हहंसक रूप से फट जाते हैं और मानव
जस्थत है जजसे ताओपो ज्वालामख
ु ी क्षेत्र के स्वास्थ्य को भारी नक
ु सान पहुंचाते हैं। उदा. जापान
रूप में जाना जाता है । यह दे श का सबसे
में फुजजयामा, इंडोनेसशया में िाकाटोआ आहद हैं।
सकिय ज्वालामुखी है और यह
3. मत
ृ या ववलुप्त ज्वालामख
ु ी: ये वे ज्वालामुखी हैं जो
स्रै टोवोलकानो (ज्वालामख
ु ी शंक्वाकार हजारों वर्ों से नहीं फटे हैं और भववष्य में ववस्फोट का
आकार की ववशेर्ता है और इसकी परतें वर्ो
कोई संकेत भी नहीं है । उदा. एंडीज पहाडों में
से लावा के प्रवाह और सख्त होने से ननसमसत
एकोंकागुआ हैं।
होती हैं) िेणी के अंतगसत आता है ।
• माउं ट मेरापी:
o मेरापी (आग का पहाड) इंडोनेसशया के 130 Q.9) Ans: D
Exp:
सकिय ज्वालामुखखयों में सबसे अचिक
सकिय है । • कथन 1 ही है : अन्तवेिी आग्नेय चट्टानें पथ्
ृ वी
की सतह के नीचे मध्यम गहराई पर मैग्मा के जमने
o यह 2,911 मीटर तक ऊाँचा होता है और
से बनती हैं। मैग्मा के िीमी गनत से ठं डा होने के
इसके ननचले ककनारों पर घनी वनस्पनतयों
कारण ऐसी चट्टानों में बहुत बडे किस्टल बनते हैं,
के साथ खडी ढलानें होती हैं।
जबकक पथ्
ृ वी के गमस आंतररक भाग से बाहर आने पर
o यह जावा द्वीप के केंद्र और इंडोनेसशया की लावा तेजी से ठं डा हो जाता है , जजससे वे महीन दाने
सांस्कृनतक राजिानी योग्याकातास के पास वाले हदखते हैं।
जस्थत है । • कथन 2 ही है : बहहमुख
स ी आग्नेय चट्टानों को
ज्वालामुखीय चट्टान भी कहा जाता है । उदाहरण हैं-
Only IASद्वारा अततररक्त ज्ञान बेसाल्ट। इस बेसाजल्टक सामग्री का उपयोग सडकों
किय ज्वालामुखी के बारे में : के ननमासण के सलए ककया जाता है और स्थानीय रूप
से रे गरु के रूप में जानी जाने वाली उपजाऊ काली
समट्टी पैदा करता है । दस
ू री ओर, अन्तवेिी आग्नेय
चट्टानों को प्लूटोननक चट्टानें कहा जाता है ।

DPP 2023 DAY 48 24


Contact us : info@onlyias.com

OnlyIAS Nothing Else Visit : dpp.onlyias.in


Contact : +91-7007 931 912

उदाहरण हैं- मोटे दाने वाला ग्रेनाइट, डोलराइट। है जोववयन ग्रहों का ननमासण काफी दरू स्थान पर
इसका उपयोग बडे पैमाने पर एक इमारत के पत्थर हुआ था।
के रूप में ककया जाता है और मंहदरों जैसे कई प्राचीन • इस प्रकार जोववयन ग्रहों को "गैस हदग्गज" के रूप
स्मारक ग्रेनाइट चट्टानों से बने होते हैं। में संदसभसत ककया जाता है क्योंकक वे ज्यादातर गैस
• कथन 3 ही है : आग्नेय चट्टानें बडी होती हैं जजनमें से बने होते हैं, हालांकक उनमें से कुछ या सभी में
कोई परत नहीं होती है , ये कठोर, ठोस, जीवावमों से शायद छोटे ठोस कोर होते हैं। सौर हवाएं इतनी तीव्र
मुक्त और बेसाल्ट के रूप में गहरे या ग्रेनाइट के रूप नहीं थीं कक जोववयन ग्रहों से गैसों को समान रूप से
में हल्के रं ग की होती हैं जो उनमें मौजूद सससलका की हटा दें ।
अचिक या कम मात्रा पर ननभसर करती हैं।
• सभी में महत्वपण
ू स ग्रहीय चब
ंु कीय क्षेत्र, वलय और
o चूंकक मैग्मा िातु अयस्कों का मुख्य स्रोत है ,
बहुत से उपग्रह हैं।
उनमें से कई आग्नेय चट्टानों से जुडे हैं।
उनमें पाए जाने वाले महान आचथसक मूल्य
सन्दभस: Fundamentals of Physical Geography Ncert-
के खननज चब
ुं कीय लोहा, ननकल, तांबा, Class 11th Chapter- 2
सीसा, जस्ता, िोमाइट, मैंगनीज और
दल
ु भ
स तत्व जैसे सोना, हीरे और प्लेहटनम
पाए जाते है । Q.11) Ans: D
Exp:
Q.10) Ans: C कथन 1 र्गलत है : लैकोसलथ एक समतलीय आिार
के साथ बडे गुंबद के आकार के अन्तवेिी वपंड होते हैं
Exp:
बुध, शि
ु , पथ् और नीचे से एक पाइप जैसी नसलका से जुडे होते हैं।
• ृ वी और मंर्गल ग्रह, स्थलीय ग्रह
कहलाते हैं क्योंकक उनके पास पथ् यह समचित ज्वालामुखखयों के सतही ज्वालामख
ु ी
ृ वी की सुखी िरती
जैसी एक ठोस, चट्टानी सतह है । स्थलीय ग्रह गुंबदों से समलता जुलता है , केवल ये अचिक गहराई

सौरमंडल के चार अंतरतम ग्रह हैं। पर जस्थत होते हैं। इसे लावा के स्थानीय स्रोत के रूप

• स्थलीय ग्रहों का ननमासण मूल तारे के पास के क्षेत्र में में माना जा सकता है जो सतह पर अपना रास्ता

हुआ था, जहां यह इतना गमस था कक गैसों को ठोस बनाते है । कनासटक के पठार को ग्रेनाइट चट्टानों की

कणों में संघननत नहीं ककया जा सकता था। सौर हवा डोमल पहाडडयों के साथ दे खा जाता है । इनमें से

सय
ू स के ननकट सबसे तीव्र थी; इससलए, इसने अचिकांश, जो अब एक्सफोलीएटे ड(ननस्ताररत) हैं,

स्थलीय ग्रहों से बहुत सारी गैस और िूल उडा दी। लैकोसलथ या बाथोसलथ के उदाहरण हैं।

स्थलीय ग्रह छोटे होते हैं और उनका कम o बाथोसलथ चम्


ु बकीय सामग्री का एक बडा

गुरुत्वाकर्सण ननकलने वाली गैसों को रोक नहीं पाता वपंड होता है जो िस्ट की गहरी गहराई में

है । ठं डा होता है और बडे गुंबदों के रूप में

बह ववकससत होता है । वे सतह पर तभी प्रकट


• ृ स्पतत, शतन, यरू े न और नेपच्यन
ू को जोववयन
(बह होते हैं जब अनाच्छादन प्रकियाएाँ
ृ स्पनत जैस)े ग्रहों के रूप में जाना जाता है , क्योंकक
वे सभी पथ् अनतव्यापी सामग्री को हटा दे ती हैं। वे बडे
ृ वी की तुलना में ववशाल हैं, और उनके पास
बह क्षेत्रों को आच्छाहदत करते हैं, और कभी-
ृ स्पनत की तरह एक गैसीय - ज्यादातर हाइड्रोजन,
कुछ हीसलयम और बफस के साथ रे स गैसों की प्रकृनत कभी गहराई तक होते हैं जो कई ककमी हो
सकती है । ये ग्रेनाइट वपंड होते हैं।

DPP 2023 DAY 48 25


Contact us : info@onlyias.com

OnlyIAS Nothing Else Visit : dpp.onlyias.in


Contact : +91-7007 931 912

बाथोसलथ मैग्मा कक्षों का ठं डा भाग होता ज्यादातर सतह से लगभग 12-16 ककमी की गहराई
है । पर होती है ।
• कथन 2 र्गलत है : o कायांतरण की प्रकिया ऊपर की चट्टानों के
o डाइक् : जब लावा भूसम में ववकससत नछद्रो दाब में या गमस आग्नेय सामग्री के संपकस के
और दरारों के माध्यम से अपना रास्ता पररणामस्वरूप गहराई पर होती है ।
बनाता है , तो यह जमीन के लगभग लंबवत o दाब के दबाव में कायांतररत चट्टानों के
रूप में जम जाता है । ननमासण को गनतशील कायांतरण के रूप में
▪ यह दीवार जैसी संरचना ववकससत जाना जाता है ।
करने के सलए उसी जस्थनत में ठं डा o भू पपसटीके भीतर प्रचसलत उच्च तापमान के
हो जाता है । ऐसी संरचनाओं को प्रभाव में तलछटी और आग्नेय चट्टानों के
डाइक कहा जाता है । खननजों के रूप में पररवतसन या पुन:
▪ ये पजवचमी महाराष्र क्षेत्र में सबसे किस्टलीकरण को थमभल या प
ं कभ कायापलट
अचिक पाए जाने वाले अन्तवेिी की प्रकिया के रूप में जाना जाता है ।
रूप हैं। • कथन 2 र्गलत है: गनतशील कायांतरण के मामले में ,
▪ इन्हें ववस्फोटों के सलए फीडर ग्रेनाइट को गनीस में बदल हदया जाता है ; समट्टी
माना जाता है जजसके कारण और शेल भू पपसटी की अचिक गहराई पर सशस्ट में
डेक्कन रै प का ववकास हुआ। बदल जाते हैं। गनीस और सशस्ट चट्टानों में एक
o स ल: अन्तवेिी आग्नेय चट्टानों के ननकट अच्छी तरह से चचजननत बंिी संरचना होती है जजसमें
क्षैनतज वपंडो को सामग्री की मोटाई के घटक खननजों को लगभग समानांतर परतों में
आिार पर ससल या शीट कहा जाता है । व्यवजस्थत ककया जाता है ।
पतली परते शीट कहलाती हैं। जबकक मोटे o थमसल कायांतरण के मामले में, बलुआ पत्थर
क्षैनतज ननक्षेपों को ससल्स कहा जाता है । क्वाटस जाइट में बदल जाता है ; समट्टी और शेल
स्लेट में तब्दील हो जाते हैं। कोयला 90
प्रनतशत से अचिक काबसन के साथ एन्रेसाइट
Q.12) Ans: D और ग्रेफाइट में बदल जाता है । संगमरमर
Exp:
तलछटी चूना पत्थर के बराबर एक रूपांतररत
• कथन 1 र्गलत है : कायांतरण का अथस है 'रूप में
चट्टान बन जाती है ।
पररवतसन'। सभी चट्टानों में पररवतसन होता है । जब
चट्टानों का मल
ू स्वरूप-उनका रं ग, कठोरता,
बनावट और खननज संरचना आंसशक या पण
ू स रूप से Q.13) Ans: B
Exp:
बदल जाती है , तो यह गमी और दबाव की अनुकूल
अंतजाभत बलों के बारे में :
पररजस्थनतयों में रूपांतररत चट्टानों को जन्म दे ती
• पथ्
ृ वी की पपसटी को प्रभाववत करने वाली शजक्तयों को
है ।.
उनके उद्गम के स्रोत के आिार पर दो व्यापक िेखणयों
o खननजों का पन
ु : किस्टलीकरण आग्नेय और
में बांटा गया है , वे अंतजासत बल और बहहजासत बल हैं।
अवसादी चट्टानों दोनों में होता है । पररवतसन तब
• अंतजासत बल कई ऊध्वासिर असंबद्िताओं को जन्म
होता है जब चट्टान एक ठोस या प्लाजस्टक अवस्था
दे ते हैं, जजससे पथ्
ृ वी की सतह पर कई प्रकार की राहत
सुवविाओं का ननमासण होता है ।

DPP 2023 DAY 48 26


Contact us : info@onlyias.com

OnlyIAS Nothing Else Visit : dpp.onlyias.in


Contact : +91-7007 931 912

डायस्रोकफक मव
ू में ट् Q.14) Ans: B
Exp:
o डायस्रोकफक बलों में ऊध्वाभधर और क्षैततज दोनों
कथन 1 र्गलत है : समुद्र ननतल प्रसारण
र्गततयां शासमल हैं, ष्क्जन्हें िमशः एवपरोजेतनक
पररकल्पना हे स द्वारा 1961 में प्रस्ताववत
आंदोलनों (महाद्वीप बनाने की र्गतत) और
की गई थी। आथसर होम्स ने संवहनी वतसमान
ऑरोजेतनक आंदोलनों (पवभत तनमाभण आंदोलनों) के
ससद्िांत हदया था। हे स ने तकस हदया कक
रूप में जाना जाता है ।
महासागरीय दरारों के सशखर पर लगातार
ऊध्वस गनत दो प्रकार की होती है - उध्वस गनत और
ववस्फोट से समुद्री िस्ट का टूटना होता है
अिोमुखी गनत।
और नया लावा उसमें समा जाता है , जजससे
o कथन 1 ही है : ऊपर की ओर गनत से महाद्वीपों या
समद्र
ु ी िस्ट दोनों तरफ िकेल हदया जाता
उसके भागों का उदय होता है , तटीय क्षेत्रों का उदय
है । इस प्रकार महासागरीय तल फैल जाता
होता है । उदा. भारत में कच्छ की खाडी के पास
है ।
लगभग 24 ककमी लंबा एक भूसम क्षेत्र कई
• कथन 2 और 3 सही हैं: समद्र
ु तल के प्रसार की
ककलोमीटर की ऊंचाई में उभरा है , इसे ‘अल्लाह-बंद’
पररकल्पना के प्रमाण हैं:
के रूप में जाना जाता है ।
o मध्य-महासागरीय कटक के सशखर के दोनों
o अिोमुखी गनत ननवासह या जलमग्न का कारण बनती
ओर समान दरू ी पर जस्थत चट्टानें ननमासण
है । मुंबई के वप्रंस डॉक इलाके में जलमग्न जंगल
की अवचि, रासायननक संरचना और
जलमग्न होने का एक उदाहरण है ।
चंब
ु कीय गण
ु ों के संदभस में उल्लेखनीय
o कथन 2 ही है : क्षैततज बल कफर े दो प्रकार के होते
समानताएं दशासती हैं। मध्य-महासागरीय
हैं- ंपीडन बल और तनाव बल।
कटक के करीब की चट्टानों में सामान्य
▪ संपीडनकारी बलों के कारण, चट्टान की
ध्रव
ु ता होती है और ये सबसे छोटी होती हैं।
परतें मुडी हुई हो जाती हैं- ऊपर की ओर मुडी
सशखा से दरू जाने पर चट्टानों की आयु
हुई चट्टानें एंहटक्लाइन कहलाती हैं और
बढ़ती जाती है ।
नीचे की ओर मुडी हुई चट्टानें ससंकलाइन
o महासागरीय िस्ट चट्टानें महाद्वीपीय
कहलाती हैं।
चट्टानों की तुलना में बहुत छोटी होती हैं।
▪ तनाव बल: तनाव के बल क्षैनतज रूप से
महासागरीय िस्ट में चट्टानों की आयु
कायस करते हैं, लेककन ववपरीत हदशाओं में ।
200 समसलयन वर्स से अचिक परु ानी नहीं है ।
तीव्र तनाव बलों के संचालन के तहत, रॉक
कुछ महाद्वीपीय चट्टानें 3,200 समसलयन
स्रे टम टूट जाता या खंडडत हो जाता है
वर्स पुरानी हैं।
जजसके पररणामस्वरूप िस्ट में दरारें और
o समुद्र तल पर तलछट अप्रत्यासशत रूप से
फ्रैक्चर का ननमासण होता है ।
बहुत पतली होती है । वैज्ञाननक उम्मीद कर
अचानक हलचलें: अचानक भू-आकृनत संबंिी हलचलें
रहे थे, अगर समुद्र के फशस महाद्वीप के रूप
ज्यादातर सलथोस्फेररक प्लेट माजजसन (टे क्टोननक प्लेट
में पुराने थे, तो बहुत लंबी अवचि के सलए
माजजसन) पर होती हैं। में टल (संवहन िाराओं) में मैग्मा के
तलछट का एक परू ा िम होगा। हालााँकक,
कारण बने दबाव के कारण प्लेट माजजसन अत्यचिक अजस्थर
तलछट स्तंभ कहीं भी 200 समसलयन वर्स से
क्षेत्र हैं। इन आंदोलनों से छोटी अवचि में काफी ववकृनत होती
अचिक पुराना नहीं पाया गया।
है । उदाहरण: भूकंप।

DPP 2023 DAY 48 27


Contact us : info@onlyias.com

OnlyIAS Nothing Else Visit : dpp.onlyias.in


Contact : +91-7007 931 912

o गहरी खाइयों में गहरे भक


ू ं प की घटनाएं Exp:
होती हैं जबकक मध्य महासागरीय ररज क्षेत्रों • कथन 1 ही है : महाद्वीपीय शेल्फ महाद्वीपीय

में , भूकंप के केंद्रों में उथली गहराई होती है । प्लेट का मंद ढलान वाला समद्र
ु ी ववस्तार है । सभी

o महासागरीय िस्ट की कम उम्र के साथ- महासागरों का महाद्वीपीय शेल्फ एक साथ

साथ तथ्य यह है कक एक महासागर के महासागरों के कुल क्षेत्रफल का 7.5% कवर करता है ।

फैलने से दस • महाद्वीपीय का ढाल ,1° या उससे भी कम है । शेल्फ


ू रे के ससकुडने का कारण नहीं
बनता है , हे स ने समुद्री िस्ट की खपत के आमतौर पर बहुत खडी ढलान पर समाप्त होती है ,

बारे में सोचा। उन्होंने आगे कहा कक सशखर जजसे शेल्फ िेक कहा जाता है ।

पर ज्वालामख • महाद्वीपीय अलमाररयां नहदयों, ग्लेसशयरों आहद


ु ी ववस्फोट के कारण समद्र

का तल िक्का दे जाता है , समद्र द्वारा नीचे लाए गए तलछट की चर मोटाई से ढकी
ु ी खाइयों में
डूब जाता है और भस्म हो जाता है । हुई हैं। महाद्वीपीय अलमाररयों द्वारा लंबे समय से
Reference: Fundamentals of Physical geography Class प्राप्त भारी तलछटी जमा, जीवावम ईंिन
X1. Page no. 33 [पेरोसलयम] का स्रोत बन जाते हैं।
• उदाहरण: दक्षक्षण-पूवस एसशया का महाद्वीपीय
Q.15) Ans: C शेल्फ, न्यूफाउं डलैंड के आसपास ग्रेट बैंक,
Exp: ऑस्रे सलया और न्यू चगनी के बीच जलमग्न क्षेत्र।
• कथन 1 ही है : भ्रंश घाहटयां दो िस्टल भागों के बीच
• कथन 2 सही है: महाद्वीपीय समतल की औसत
एक गतस, अवसाद या बेससन का प्रनतननचित्व करती
चौडाई 70 – 80 ककमी के बीच है । चचली के तट,
हैं। भ्रंश घाहटयााँ वास्तव में िस्टल भागों के
सुमात्रा के पजवचमी तट, आहद जैसे कुछ हासशये पर
ववस्थापन और मध्य भाग के दो सामान्य भ्रंशों के
अलमाररयां लगभग अनुपजस्थत या बहुत संकरी हैं।
बीच िंसने के कारण बनती हैं। ये तनावग्रस्त बलों के
[महासागर – महाद्वीप असभसरण और महासागर –
कारण दो ववपरीत हदशाओं में रॉक ब्लॉकों की गनत के
महासागर असभसरण]। यह संयुक्त राज्य अमेररका
कारण बनते हैं। भ्रंश घाहटयााँ लंबी और संकरी हैं।
के पूवी तट के साथ 120 ककमी तक चौडा है । इसके
उदा. जॉडसन का मत
ृ सागर, कैसलफोननसया में डेथ ववपरीत, आकसहटक महासागर में साइबेररयन शेल्फ,
वैली। नमसदा, तापी और ऊपरी दामोदर नदी घाहटयााँ
दनु नया में सबसे बडा, 1,500 ककमी चौडा है ।
भारत में भ्रंश घाहटयों के उदाहरण हैं।
अलमाररयों की गहराई भी सभन्न होती है । यह कुछ
• कथन 2 ही है : ब्लॉक पवसत तब बनते हैं जब पथ्
ृ वी क्षेत्रों में 30 मीटर जजतना उथला हो सकता है जबकक
की पपडी दो समानांतर भ्रंशों के दोनों ककनारों पर
कुछ क्षेत्रों में यह 600 मीटर जजतना गहरा हो सकता
(संपीडन) डूब जाती है और मध्य भाग अपनी जगह
है ।
पर रहता है । इससलए, दो भ्रंश घाहटयों के बीच एक
• कथन 3 सही है : वे आचथसक खननजों के सलए संभाववत
ब्लॉक पवसत पाया जा सकता है । जो भूसम डूबती है उसे
स्थल हैं [दनु नया के पेरोसलयम और गैस के उत्पादन
हडपने और ऊपर उठे हुए ब्लॉकों के रूप में जाना
का 20% अलमाररयों से आता है । पॉलीमेटेसलक
जाता है , जजसे हॉस्टस कहा जाता है , जैसे भारत में
नोड्यूल्स (मैंगनीज नोड्यल्
ू स, आयरन और
सतपडु ा पवसत।
मैंगनीज हाइड्रॉक्साइड की संकेंहद्रत परतें ) आहद
Reference: Geography by Mahesh Kumar Barnwal.3rd
edition . Page number-30 ववसभन्न खननज अयस्कों जैसे मैंगनीज, आयरन
कॉपर, सोना आहद के अच्छे स्रोत हैं। समुद्री भोजन
लगभग परू ी तरह से महाद्वीपीय अलमाररयों से
Q.16) Ans: D

DPP 2023 DAY 48 28


Contact us : info@onlyias.com

OnlyIAS Nothing Else Visit : dpp.onlyias.in


Contact : +91-7007 931 912

आता है । वे सबसे अच्छी मछली पकडने के मैदान के रूप में परु ाने थे, तो बहुत लंबी
प्रदान करते हैं। अवचि के सलए तलछट का एक पूरा
िम होगा। हालााँकक, तलछट स्तंभ
Q.17) Ans: B
Exp: कहीं भी 200 समसलयन वर्स से
o कथन 1 र्गलत है : समुद्र ननतल प्रसारण अचिक पुराना नहीं पाया गया।
पररकल्पना हे स द्वारा 1961 में प्रस्ताववत ▪ गहरी खाइयों में गहरे भूकंप की
की गई थी। आथसर होम्स ने संवहनी करें ट घटनाएं होती हैं जबकक मध्य
ससद्िांत हदया था। हे स ने तकस हदया कक महासागरीय ररज क्षेत्रों में , भक
ू ंप
महासागरीय लकीरों के सशखर पर लगातार के केंद्रों में उथली गहराई होती है ।
ववस्फोट से समुद्री िस्ट का टूटना होता है ▪ महासागरीय िस्ट की कम उम्र के
और नया लावा उसमें समा जाता है , जजससे साथ-साथ तथ्य यह है कक एक
समुद्री िस्ट दोनों तरफ िकेल हदया जाता महासागर के फैलने से दस
ू रे के
है । इस प्रकार महासागरीय ननतल प्रसा है । ससकुडने का कारण नहीं बनता है ,
o कथन 2 और 3 सही हैं: समुद्र तल के प्रसार हे स ने समद्र
ु ी िस्ट की खपत के
की पररकल्पना के प्रमाण हैं: बारे में सोचा। उन्होंने आगे कहा
▪ मध्य-महासागरीय कटक के कक सशखर पर ज्वालामुखी
सशखर के दोनों ओर समान दरू ी पर ववस्फोट के कारण समद्र
ु का तल
जस्थत चट्टानें ननमासण की अवचि, िक्का दे जाता है , समुद्री खाइयों में
रासायननक संरचना और चंब
ु कीय डूब जाता है और भस्म हो जाता है ।
Reference: Fundamentals of Physical geography Class
गुणों के संदभस में उल्लेखनीय
X1. Page no. 33
समानताएं दशासती हैं। मध्य-
महासागरीय कटक के करीब की
Q.18) Ans: C
चट्टानों में सामान्य ध्रुवता होती है Exp:
और ये सबसे छोटी होती हैं। सशखा • कथन 1 सही है : भ्रंश घाहटयां दो िस्टल भागों के बीच
से दरू जाने पर चट्टानों की आयु एक गतस, अवसाद या बेससन का प्रनतननचित्व करती
बढ़ती जाती है । हैं। भ्रंश घाहटयााँ वास्तव में िस्टल भागों के
▪ महासागरीय िस्ट चट्टानें, ववस्थापन और मध्य भाग के दो सामान्य भ्रंशों के
महाद्वीपीय चट्टानों की तुलना में बीच िंसने के कारण बनती हैं। ये तनावग्रस्त बलों के
बहुत छोटी होती हैं। महासागरीय कारण दो ववपरीत हदशाओं में रॉक ब्लॉकों की गनत के
िस्ट में चट्टानों की आयु 200 कारण बनते हैं। भ्रंश घाहटयााँ लंबी और संकरी होती
समसलयन वर्स से अचिक पुरानी हैं। उदा. जॉडसन का मत
ृ सागर, कैसलफोननसया में डेथ
नहीं है । कुछ महाद्वीपीय चट्टानें वैली। नमसदा, तापी और ऊपरी दामोदर नदी घाहटयााँ
3,200 समसलयन वर्स पुरानी हैं। भारत में भ्रंश घाहटयों के उदाहरण हैं।
▪ समुद्र तल पर तलछट • कथन 2 सही है : ब्लॉक पवसत तब बनते हैं जब पथ्
ृ वी
अप्रत्यासशत रूप से बहुत पतली की पपसटी, दो समानांतर भ्रंशों के दोनों ककनारों पर
होती है । वैज्ञाननक उम्मीद कर रहे (संपीडन) डूब जाती है और मध्य भाग अपनी जगह
थे, अगर समुद्र ननतल महाद्वीप पर रहता है । इससलए, दो भ्रंश घाहटयों के बीच एक

DPP 2023 DAY 48 29


Contact us : info@onlyias.com

OnlyIAS Nothing Else Visit : dpp.onlyias.in


Contact : +91-7007 931 912

ब्लॉक पवसत पाया जा सकता है । जो भसू म डूबती है उसे उथला हो सकता है जबकक कुछ क्षेत्रों में यह 600
हॉस्टस कहा जाता है , जैसे भारत में सतपुडा पवसत। मीटर जजतना गहरा हो सकता है ।
Reference: Geography by Mahesh Kumar Barnwal.3rd • कथन 3 सही है : वे आचथसक खननजों के सलए संभाववत
edition . Page number-30
स्थल हैं [दनु नया के पेरोसलयम और गैस के उत्पादन
का 20% अलमाररयों से आता है । पॉलीमेटेसलक
Q.19) Ans: D नोड्यूल्स (मैंगनीज नोड्यल्
ू स, आयरन और
Exp:
मैंगनीज हाइड्रॉक्साइड की संकेंहद्रत परतें ) आहद
• कथन 1 ही है : महाद्वीपीय शेल्फ ,महाद्वीपीय
ववसभन्न खननज अयस्कों जैसे मैंगनीज, आयरन
प्लेट का मंद ढलान वाला समद्र
ु ी ववस्तार है । प्रत्येक
कॉपर, सोना आहद के अच्छे स्रोत हैं। समद्र
ु ी भोजन
महाद्वीप के इन ववस्ताररत हासशये पर अपेक्षाकृत
लगभग पूरी तरह से महाद्वीपीय अलमाररयों से
उथले समुद्र और खाडी का कब्जा है । सभी महासागरों
आता है ।
का महाद्वीपीय शेल्फ एक साथ महासागरों के कुल
क्षेत्रफल का 7.5% कवर करता है ।
o महाद्वीपीय का ढाल 1° या उससे भी कम Q.20) Ans: C
Exp:
है । शेल्फ आमतौर पर बहुत खडी ढलान पर
• कथन 1 सही है : जब पानी या एससड में कुछ घुल जाता
समाप्त होती है , जजसे शेल्फ िेक कहा जाता
है , तो पानी या एससड में घुली हुई सामग्री को घोल
है ।
कहा जाता है । कई खननज पानी से घल ु जाते हैं,
o महाद्वीपीय शेल्फ नहदयों, ग्लेसशयरों आहद
खासकर जब, बाररश के पानी की तरह, इसमें एक
द्वारा नीचे लाए गए तलछट से ढकी हुई
कमजोर एससड बनाने के सलए पयासप्त काबसन
होती हैं। महाद्वीपीय शेल्फ द्वारा लंबे
डाइऑक्साइड होता है ।
समय से प्राप्त भारी तलछटी जमा,
• कथन 2 गलत है : घल
ु नशील चट्टान बनाने वाले
जीवावम ईंिन [पेरोसलयम] का स्रोत बन
खननज जैसे नाइरे ट, सल्फेट और पोटे सशयम
जाते हैं।
ववलयन प्रकिया से प्रभाववत होते हैं। इससलए,
o उदाहरण: दक्षक्षण-पूवस एसशया का
बरसात के मौसम में कोई अवशेर् छोडे ब्रबना ये
महाद्वीपीय शेल्फ, न्यफ
ू ाउं डलैंड के
खननज आसानी से बाहर ननकल जाते हैं और शुष्क
आसपास ग्रेट बैंक, ऑस्रे सलया और न्यू
क्षेत्रों में जमा हो जाते हैं।
चगनी के बीच जलमग्न क्षेत्र।
• कथन 3 सही है : सामान्य नमक (सोडडयम क्लोराइड)
• कथन 2 सही है: महाद्वीपीय समतल की औसत
भी एक चट्टान बनाने वाला खननज है । सोडडयम
चौडाई 70 – 80 ककमी के बीच है । चचली के तट,
काबोनेट, या सोडा ऐश, Na2CO3, प्रकृनत में व्यापक
सम
ु ात्रा के पजवचमी तट, आहद जैसे कुछ शेल्फ
रूप से पाया जाता है , जो खननज पानी के घटकों के
लगभग अनुपजस्थत होते हैं। [महासागर – महाद्वीप
रूप में और ठोस खननज नारोन, रोना और
असभसरण और महासागर – महासागर असभसरण]।
थमोनाराइट के रूप में होता है । इस क्षारीय नमक की
यह संयुक्त राज्य अमेररका के पूवी तट के साथ 120
बडी मात्रा का उपयोग कांच, डडटजेंट और क्लीऩ्र
ककमी तक चौडा है । इसके ववपरीत, आकसहटक
बनाने में ककया जाता है ।
महासागर में साइबेररयन शेल्फ, दनु नया में सबसे
बडा, 1,500 ककमी चौडा है । अलमाररयों की गहराई Reference: Fundamental of physical geography (11th
भी सभन्न होती है । यह कुछ क्षेत्रों में 30 मीटर जजतना NCERT) page no. 45-57

DPP 2023 DAY 48 30


Contact us : info@onlyias.com

OnlyIAS Nothing Else Visit : dpp.onlyias.in


Contact : +91-7007 931 912

Q.21) Ans: C o फ्रॉस्ट वेडडंग तब होती है जब पानी दरार में पड जाता


Exp:
है , जम जाता है और फैल जाता है । यह प्रकिया
• कथन 1 गलत है: जल का रासायननक योग
चट्टानों को तोडती है । जब इस प्रकिया को दोहराया
जलयोजन है । जलयोजन ,रासायननक अपक्षय का
जाता है , तो चट्टानों में दरारें बडी और बडी हो जाती
एक रूप है जजसमें खननज के रासायननक बंिन बदल
हैं और चट्टान टूट सकती हैं, या टूट सकती हैं। फ्रॉस्ट
जाते हैं क्योंकक यह जल के साथ अंतरकिया करता है ।
वेडडंग, फ्रीजजंग और ववगलन भौनतक अपक्षय
यह प्रकिया प्रनतवती और लंबी है ।
प्रकिया के उदाहरण हैं।
• कथन 2 सही है : खननज पानी लेते हैं और फैलते हैं;
यह ववस्तार सामग्री या चट्टान की मात्रा में वद्
ृ चि Reference: Fundamental of physical geography (11th
का कारण बनता है । इस प्रकिया की ननरं तर NCERT) page no. 45-57
पुनराववृ त्त चट्टानों के ववघटन का कारण बन सकती
है । Q.23) Ans: C
Exp:
• कथन 3 सही है : कई समट्टी के खननज गीले और
• कथन 1 र्गलत है : उच्चवती (पवसतीय) भागों में गुरुत्व
सूखने के दौरान फूल जाते हैं और ससकुड जाते हैं और
के प्रभाव से ढाल के सहारे शैल पदाथों का वह
ृ त पैमाने
इस प्रकिया की पुनराववृ त्त के पररणामस्वरूप उपरी
पर ऊपर से नीचे की ओर स्थानांतरण, वह
ृ त ् संचलन’
सामग्री में दरार आ जाती है । रोमनछद्रों में लवण तेजी
कहलाता है । यह िीरे -िीरे या अचानक हो सकता है ,
से और बार-बार जलयोजन से गुजरते हैं और रॉक
ढलानों की ढाल, अपक्षनयत मलबे के वजन और
फ्रैक्चररंग में मदद करते हैं। जलयोजन के कारण
बाररश के पानी द्वारा आपूनतस की जाने वाली
खननजों में मात्रा में पररवतसन भी भौनतक अपक्षय में
चचकनाई वाली नमी के आिार पर।
मदद करता है ।
• कथन 2 सही है : वह
ृ त ् संचलन से पहले कई सकिय
Reference: Fundamental of physical geography (11th कारण होते हैं। वे हैं
NCERT) page no. 45-57 o प्राकृनतक या कृब्रत्रम तरीकों से ऊपर से
सामग्री को नीचे से समथसन हटाना;
Q.22) Ans: D o ढलानों की ढाल और ऊंचाई में वद्
ृ चि;
Exp:
o स्वाभाववक रूप से या कृब्रत्रम भराव द्वारा
र्ौततक अपक्षय के बारे में
ओवरलोडडंग;
o भौनतक अपक्षय, जजसे यांब्रत्रक अपक्षय के रूप में भी
o भारी वर्ास, ढलान सामग्री की संतजृ प्त और
जाना जाता है , प्रकियाओं का एक वगस है जो ब्रबना
स्नेहन के कारण ओवरलोडडंग;
रासायननक पररवतसन के चट्टानों के ववघटन का
o मूल ढलान वाली सतहों से सामग्री या भार
कारण बनता है । अचिकांश भौनतक अपक्षय प्रकियाएं
को हटाना;
थमसल ववस्तार और दबाव मुक्त होने के कारण होती
o भूकंप, ववस्फोट या मशीनरी की घटना;
हैं। ये प्रकियाएं िीमी और छोटी होती हैं और चट्टानों
o अत्यचिक प्राकृनतक ररसाव;
को बहुत नुकसान पहुंचाती हैं।
o झीलों, जलाशयों और नहदयों से पानी की
o भौनतक या यांब्रत्रक अपक्षय प्रकियाएं कुछ बलों पर
भारी कमी के कारण ढलानों या नदी के
ननभसर करती हैं। वे बल हो सकते हैं:
ककनारे से पानी का बहहवासह िीमा हो जाता
(i) गुरुत्वाकर्सण बल जैसे अनतभाररत दाब, भार और
है ;
(ii) तापमान पररवतसन, किस्टल वद्
ृ चि

DPP 2023 DAY 48 31


Contact us : info@onlyias.com

OnlyIAS Nothing Else Visit : dpp.onlyias.in


Contact : +91-7007 931 912

o प्राकृनतक वनस्पनतयों का अंिािि


ंु पर संकरी पटररयों में होता है । मलबे का हहमस्खलन
ननष्कासन। मडफ्लो से तेज हो सकता है ।
• कथन 3 र्गलत है : द्रव्यमान के संचलन में • कथन 3 सही है : वनस्पनत आच्छादन के अभाव में
,गुरुत्वाकर्सण द्वारा सहायता प्राप्त होती है और कोई और भारी वर्ास के साथ, अपक्षनयत पदाथों की मोटी
भी भ-ू आकृनतक एजेंट जैसे बहता पानी, हहमनद, परतें पानी से संतप्ृ त हो जाती हैं और या तो िीरे -िीरे
हवा, लहरें और िाराएं जन आंदोलनों की प्रकिया में या तेजी से ननजवचत चैनलों के साथ नीचे बहती हैं।
भाग नहीं लेती हैं। इसका मतलब है कक जन आंदोलन यह एक घाटी के भीतर कीचड की िारा की तरह
क्षरण के तहत नहीं आते हैं, हालांकक सामग्री का एक हदखता है और इसे मडफ्लो कहा जाता है । मडफ़्लो
स्थान से दस
ू रे स्थान पर स्थानांतरण (गरु
ु त्वाकर्सण अक्सर फटने वाले या हाल ही में फटे ज्वालामखु खयों
द्वारा सहायता प्राप्त) होता है। की ढलानों पर होते हैं। ज्वालामुखी की राख, िूल और
• कथन 4 गलत है : गुरुत्वाकर्सण सभी पदाथों पर अन्य टुकडे भारी बाररश के कारण कीचड में बदल
अपना बल लगाता है ।इससलए, जन आंदोलन के जाते हैं और जीभ या कीचड की िाराओं के रूप में बह
सलए अपक्षय कोई पूवासपक्ष
े ा नहीं है , हालांकक यह जन जाते हैं, जजससे मानव बजस्तयों को बहुत नुकसान
आंदोलनों में सहायता करता है । अपक्षनयत सामग्री होता है ।
के बजाय अपक्षनयत ढलानों पर जन आंदोलन बहुत
Reference : Fundamental of physical geography (11th
सकिय हैं। द्रव्यमान की गनत िीमी से तीव्र तक हो NCERT) page no. 45-57
सकती है , सामग्री के उथले से गहरे स्तंभों को
प्रभाववत करती है ।
Q.25) Ans: B
Exp:
Reference: Fundamental of physical geography (11th
NCERT) page no. 45-57 • कथन 1 र्गलत है : भूस्खलन बहुत तीव्र गनत से होने
वाली गनत है जो तब होती है जब समट्टी या चट्टान

Q.24) Ans: A का एक बडा हहस्सा अचानक चगर जाता है ।


Exp: • कथन 2 र्गलत है : द्रव्यमान के वपछडे घण
ू न
स के ब्रबना
• कथन 1 र्गलत है : एक तीव्र गनत के रूप में पथ्
ृ वी का पथ्
ृ वी के मलबे के तेजी से लढ़
ु कने या कफसलने को
प्रवाह पानी की संतप्ृ त मद
ृ ा या गाद का पहाडडयों या मलबा-स्खलन के रूप में जाना जाता है । एक
ननम्न-कोण वाली छतों के साथ नीचे की ओर गनत है । ऊध्वासिर या ऊपर की ओर लटके हुए भाग से पथ्
ृ वी
वे ज्यादातर आद्रस जलवायु क्षेत्रों में प्रचसलत हैं और के मलबे का लगभग मक्
ु त रूप से चगरना मलबा-
कोमल ढलानों पर पाए जाते हैं। सॉसलफ़्लुक्शन, नीचे स्खलन कहलाता है ।
की ओर बहने वाली मद
ृ ा के द्रव्यमान या पानी से • कथन 3 ही है : स्लजम्पंग ववशेर् रूप से आम है जहां
संतप्ृ त या चचकनाई वाले महीन दाने वाले चट्टान के पारगम्य मलबे या चट्टान की परतें समट्टी जैसे
मलबे की िीमी गनत है । अभेद्य स्तर से अचिक हो जाती हैं। नम समट्टी एक
• कथन 2 सही है : ढलान की प्रवणता के आिार पर, चचकनी कफसलन वाली सतह प्रदान करती है जजस पर
उच्च ढलान के मामले में नरम तलछटी आिार भी ऊपरी परतें आसानी से पीछे की ओर घूमने के साथ
नीचे की ओर खखसक सकते हैं। गहरे अपक्षनयत स्लाइड करती हैं। अलग-अलग रॉक मास का बेडडंग,
आग्नेय चट्टानें भी नीचे खखसक सकती हैं। तीव्र ज्वाइंट या फॉल्ट सतहों के नीचे खखसकना एक
संचलन के रूप में मलबा हहमस्खलन खडी ढलानों रॉकस्लाइड है । खडी ढलानों पर, रॉक स्लाइडडंग बहुत
तेज और ववनाशकारील होती है । रॉक फॉल खद
ु को

DPP 2023 DAY 48 32


Contact us : info@onlyias.com

OnlyIAS Nothing Else Visit : dpp.onlyias.in


Contact : +91-7007 931 912

ढलान से दरू रखते हुए ककसी भी खडी ढलान पर रॉक हैं। वेदरड बेडरॉक, हटल, आउटवॉश डडपॉजजट, इओसलयन
ब्लॉकों का मक्
ु त रूप से चगरना है । चट्टान का चगरना सैंड, लोएस, एल्यूववयम और लोकल ओवरवॉश।
चट्टान की सतही परतों से होता है । • ववकल्प B ही है : स्थलाकृनत का समट्टी के ववकास पर
गहरा प्रभाव पडता है । कटाव के कारण पहाडडयों के
Reference: Fundamental of physical geography (11th
NCERT) page no. 45-57 ककनारे की समट्टी उथली हो जाती है । लीचचंग हाननयों के
कारण पहाडडयों की चोटी पर समट्टी गहरी, लेककन रं ग में
हल्की होती है । घाहटयों में समट्टी गहरी और अचिक
Q.26) Ans: D
Exp: क्षक्षनतज वाली होती है । मौसम और जलवायु के ववसभन्न
• कथन 1 ही है : पवत्तयां और काबसननक घटक मद
ृ ा की तत्वों की कियाओं के माध्यम से अपक्षय को यांब्रत्रक
ऊपरी काबसननक परत बनाने के सलए ववघहटत हो जाते ववघटन और चट्टानों के रासायननक अपघटन के रूप में
हैं, जजसे ह्यूम के रूप में जाना जाता है । समट्टी में पररभावर्त ककया गया है ।
नयूमस की मात्रा इसकी उवसरता में बहुत महत्वपूणस • ववकल्प C ही है : समट्टी के ननमासण में जलवायु एक
भूसमका ननभाती है । समट्टी की बनावट के सलए महत्वपूणस सकिय कारक है । समट्टी के ववकास में शासमल
जजम्मेदार तीन मुख्य घटक रे त, गाद और समट्टी हैं। जलवायु तत्व हैं:
इन तीन घटकों के आिार पर समट्टी की खननज (i) तीव्रता, आववृ त्त और वर्ास की अवचि के संदभस में
बनावट सभन्न होती है । नमी- वाष्पीकरण और आद्रस ता
• कथन 2 ही है : समट्टी को भूसम की सतह पर पतले (ii) मौसमी और दै ननक वववविताओं के संदभस में
रूप से ववतररत ककया जाता है और एक मल
ू भत
ू तापमान।
अंतरफलक के रूप में कायस करता है जहां वायुमंडल, • ववकल्प D र्गलत है : समट्टी की बनावट समट्टी के ननमासण
स्थलमंडल, जलमंडल और जीवमंडल समलते हैं। के सलए जजम्मेदार कारक नहीं है । मूल सामग्री (खननज
अचिकांश समट्टी एक अकाबसननक सामग्री है , और पोर्क तत्व), समय, जलवायु, राहत और जीव ऐसे
इससलए समट्टी को आमतौर पर स्थलमंडल के हहस्से कारक हैं जो समट्टी के ननमासण के सलए जजम्मेदार हैं।
के रूप में वगीकृत ककया जाता है , लेककन यह अन्य
Reference: Fundamental of physical geography (11th
तीन पथ्
ृ वी मण्डलों से भी घननष्ठ रूप से संबंचित है । NCERT) page no. 45-57
• कथन 3 ही है : समट्टी जो पथ्
ृ वी की पपडी की सबसे
ऊपरी परत पर बनती है , हजारों वर्ों से पहाडों के
Q.28) Ans: D
ननरं तर अपक्षय का पररणाम है । यह क्षय का पररणाम Exp:
है और यह एक पररवनतसत और ववकासशील प्रकिया • कथन 1 र्गलत है : अनाच्छादन में सभी बहहजासत भू-
है । आकृनत प्रकियाएं शासमल हैं। यह रासायननक और
भौनतक दोनों तरह से कायस करता है । रासायननक
Reference: https://lotusarise.com/soil-genesis
अनाच्छादन को रासायननक अपक्षय भी कहा जाता
है और भौनतक अनाच्छादन को कभी-कभी यांब्रत्रक
Q.27) Ans: D अपक्षय भी कहा जाता है ।
Exp:
• कथन 2 ही है : पथ्
ृ वी की सामग्री का क्षरण और
• ववकल्प A ही है : मूल सामग्री भूगसभसक सामग्री है जजससे
पररवहन हवा, बहते पानी, ग्लेसशयरों, लहरों और
समट्टी का ननमासण होता हैं। मल
ू सामग्री के सात रूपांतर
भज
ू ल द्वारा लाया जाता है । इन एजेंटों को जलवायु
पररष्क्स्थततयों द्वारा तनयंत्रत्रत ककया जाता है ।

DPP 2023 DAY 48 33


Contact us : info@onlyias.com

OnlyIAS Nothing Else Visit : dpp.onlyias.in


Contact : +91-7007 931 912

• कथन 3 ही है : सौर ववककरण को वायम


ु ंडल द्वारा • कथन 3 ही है : पजवचमी घाट क्षेत्र की तल
ु ना में ,
अवरोचित ककया जाता है और, हालांकक कुछ ऊजास हहमालय क्षेत्र में स्लाइड ववशाल हैं। ज्यादातर
वायुमंडल से वापस अंतररक्ष में परावनतसत होती है , मामलों में अंतननसहहत सलथोलॉजी के साथ ओवरबडसन
लगभग 60% गनतज ऊजास (पवन) में पररवनतसत हो कफसलने के दौरान ववस्थावपत हो जाता है , खासकर
जाती है जो तरं गों को चलाती है , इससलए इन भूकंपीय कारक के कारण। हहमालय की ऊंचाई
प्रकियाओं के सलए ौर ववककरण ऊजाभ का अंततम पजवचमी घाट से अचिक है । ढलान तुलनात्मक रूप से
स्रोत है । अचिक खडी हैं, जजससे भूस्खलन की संभावना बढ़
• कथन 4 ही है : जल ववज्ञान चि को "जल चि" के जाती है । हहमालय की बारहमासी नहदयााँ भारी मात्रा
रूप में भी जाना जाता है ; यह पथ्ृ वी पर सामान्य जल में गाद और मलबा ले जाती हैं। ग्लेसशयरों के वपघलने
पुनचसिण प्रणाली है । सौर ववककरण के कारण, पानी से गसमसयों के दौरान पानी का प्रवाह भी बढ़ जाता है
आमतौर पर समुद्र, झीलों आहद से वाजष्पत हो जाता जजससे अचिक भूस्खलन होता है ।
है । वैजववक स्तर पर आने वाले ववककरण का लगभग
Reference: Fundamental of physical geography (11th
33% हाइड्रोलॉजजकल चि को संचासलत करता है । NCERT) page no. 45-57

Reference:
https://www.sciencedirect.com/topics/earth-and- Q.30) Ans: C
planetary-sciences/denudation Exp:
• कथन 1 र्गलत है : ओरोजेननक प्रकियाओं में गंभीर
Q.29) Ans: D वलन के माध्यम से पवसत ननमासण और पथ्
ृ वी की
Exp: पपडी के लंबे और संकीणस बेल्ट को प्रभाववत करना
• कथन 1 ही है : हहमालय और पजवचमी घाट दोनों ही शासमल है । पपडी गंभीर रूप से वलनों में ववकृत हो
भूस्खलन के प्रनत सवासचिक संवेदनशील क्षेत्र हैं। जाती है ।
हहमालय पवसत बेल्ट में गंभीर भूकंपीय गनतववचि के कथन 2 ही है : सभी प्रकियाएं जो पथ्
• ृ वी की पपडी के
अिीन वववतसननक रूप से अजस्थर युवा भूवैज्ञाननक कुछ हहस्सों का स्थानांतरण, उन्नयन या ननमासण
संरचनाएं शासमल हैं। हहमालय वववतसननक रूप से करती हैं, डायस्रोकफज्म के अंतगसत आती हैं, जजसमें
सकिय हैं और ज्यादातर तलछटी चट्टानों और ऑरोजेनी, भक
ू ं प और प्लेट टे क्टोननक्स की
असंगहठत और अिस-समेककत ननक्षेपों से बने हैं। प्रकियाओं के पररणामस्वरूप पथ्
ृ वी की पपडी में
इसकी ढलान भी बहुत खडी हैं। फॉल्ट और फ्रैक्चर होता है ।
• कथन 2 ही है : हहमालय की तुलना में , तसमलनाडु, कथन 3 ही है : एवपरोजेननक प्रकियाओं में पथ्
• ृ वी की
कनासटक, केरल और पजवचमी तट के साथ पजवचमी पपडी के एक बडे हहस्से का उत्थान शासमल है । इसके
घाट की सीमा पर जस्थत नीलचगरी अपेक्षाकृत पररणामस्वरूप पथ्
ृ वी की पपडी का एक सािारण
वववतसननक रूप से जस्थर हैं और ज्यादातर बहुत कठोर ववरूपण हो सकता है । एवपरोजेतनक एक महाद्वीपीय
चट्टानों से बनी हैं; लेककन, कफर भी, मलबा-स्खलन तनमाभण प्रकिया है ।
और भूस्खलन होते हैं, हालांकक हहमालय में उतनी
बार नहीं होते हैं। पजवचमी घाट और नीलचगरी Reference: Fundamental of physical geography (11th
NCERT) page no. 45-57
भूगभीय रूप से जस्थर हैं, लेककन नव-वववतसननक
गनतववचि से प्रभाववत पठारी माजजसन को ऊपर उठा
Q.31) Ans: D
चक
ु े हैं।
Exp:

DPP 2023 DAY 48 34


Contact us : info@onlyias.com

OnlyIAS Nothing Else Visit : dpp.onlyias.in


Contact : +91-7007 931 912

खतनजों के बारे में नद्यभार को वहन ककया जाता है । िाराओं के सलए


• एक खननज एक प्राकृनतक रूप से पाया जाने वाला भार बहुत भारी हो जाता है और ननम्न ढाल पर ले
मुख्य रूप से अकाबसननक पदाथस है , जजसमें एक जाया जाता है और वही ननक्षेवपत कर हदया जाता है
व्यवजस्थत परमाजण्वक संरचना और एक ननजवचत और एक व्यापक ननम्न से उच्च शंकु के आकार के
रासायननक संरचना और भौनतक गुण होते हैं। जमा के रूप में फैल जाता है जजसे जलोढ़ पंखा कहा
लेककन काबसननक खननजों के भी उदाहरण हैं। जाता है ।
• एक खननज की एक ववसशष्ट रासायननक संरचना • कथन 3 र्गलत है : आद्रभ क्षेत्रों में जलोढ़ पंखे आमतौर
होती है जो अन्य खननजों से अलग होती है । साथ ही, पर ससर से पैर तक कोमल ढलान के साथ कम
एक खननज के भौनतक गण
ु अन्य खननजों से सभन्न शंक्वाकार ददखते हैं और वे शष्ु क और अिस-शष्ु क
होते हैं। कुछ गुणों में किस्टल संरचना, कठोरता, जलवायु में खडी ढलान के साथ उच्च शंकु के रूप में
घनत्व और रं ग शासमल हैं। उदा. हीरा एक अत्यंत हदखाई दे ते हैं।
कठोर, पारभासी किस्टल है ।
Reference: Fundamental of physical geography (11th
• एक खननज दो या दो से अचिक तत्वों से बना होता NCERT) page no. 58 -73
है । लेककन, कभी-कभी सल्फर, तांबा, चांदी, सोना,
ग्रेफाइट आहद जैसे एकल तत्व खननज भी पाए जाते
Q.33) Ans: D
हैं। Exp:
• खननज हमारे दै ननक जीवन में बनु नयादी कच्चे माल Meander. के बारे में
हैं, और आचथसक, सामाजजक और तकनीकी ववकास के
सलए महत्वपूणस हैं। इनका उपयोग कृवर्, ननमासण, • एक ववसपस एक नदी चैनल में एक मोड है । ववसपस

भोजन, पैकेजजंग, बतसन, पररवहन आहद में ककया तब बनते हैं जब नदी का पानी चैनल के बाहरी

जाता है । ककनारों को समटा दे ता है । पानी चैनल के अंदर


तलछट जमा करता है । मींडसस केवल समतल भूसम
Reference: https://www2.bgs.ac.uk/mineralsuk/mine पर होते हैं जहााँ नदी बडी और स्थावपत होती है ।
ralsYou/whatAre.htm
• बडे बाढ़ और डेल्टा मैदानों में , नहदयााँ शायद ही कभी
सीिे रास्ते में बहती हैं। लूप जैसे चैनल पैटनस को
Q.32) Ans: A
मेन्डसस कहा जाता है ।
Exp:
कथन 1 ही है : एक जलोढ़ पंखा एक त्रत्रर्ज • बडी नहदयों के तट पर अवतल तट के साथ सकिय
• ु के
आकार का बजरी, रे त और यहां तक कक तलछट के ननक्षेपण और उत्तल तट के साथ कटाव होता है ।

छोटे टुकडे, जैसे गाद का जमाव होता है । इस अवतल बैंक एक कट-ऑफ बैंक के रूप में जाना

तलछट को जलोढ़ कहा जाता है । जलोढ़ पंखे जाता है जो एक तीव्र कगार के रूप में हदखाई दे ता है

आमतौर पर पहाडों, पहाडडयों या घाहटयों की खडी और उत्तल बैंक एक लंबी, मंद ढाल वाले होते है ।

दीवारों के साथ बहने वाले पानी के रूप में बनाए • जैस-े जैसे मेन्डसस गहरे छोरों में ववकससत होते हैं,

जाते हैं। जलोढ़ पंखे तब बनते हैं जब उच्च स्तरों से वैसे-वैसे वे मुडाव ब्रबंदओ
ु ं पर अपरदन के कारण

बहने वाली िाराएाँ ननम्न ढाल वाले फुट ढलान वाले कट-ऑफ हो सकते हैं और ऑक्स-बो (गोखुर झील)

मैदानों में चगर जाती हैं। के रूप में छोड हदए जाते हैं।

• कथन 2 ही है : आम तौर पर जलोढ़ पंखे में पहाडी Reference: Fundamental of physical geography (11th
ढलानों पर बहने वाली िाराओं द्वारा बहुत मोटे NCERT) page no. 58 -73

DPP 2023 DAY 48 35


Contact us : info@onlyias.com

OnlyIAS Nothing Else Visit : dpp.onlyias.in


Contact : +91-7007 931 912

Q.34) Ans: B अवसाद में एक झील पाई जाती है , जजसे टानभ कहा
Exp:
जाता है ।
• कथन 1 ही है : सकस एक प्रकार का दहमनदीय
• ववकल्प 2 ही है : एरे त एक पतली, चट्टान की सशखा
अपरदन स्थलरूप है । इसे कोरी के नाम से भी जाना
है जो दो समीपवती ग्लेसशयरों द्वारा चट्टान में एक
जाता है । वे गहरे , लंबे और चौडे कंु ड या बेससन होते हैं
खडी ररज पहनने के बाद बची है । एक हॉनस तब होता
जजनमें बहुत खडी अवतल होती हैं जो इसके ससर के
है जब ग्लेसशयर तीन या अचिक आटे स को नष्ट कर
साथ-साथ ककनारों पर ऊंची दीवारों को लंबवत रूप से
दे ते हैं, आमतौर पर एक तेज िार वाली चोटी का
चगराती हैं। एक सकस मूल रूप से एक कटोरे के आकार
ननमासण करते हैं। सकस अवतल, गोलाकार बेससन होते
का अवसाद है जो ग्लेसशयर की अपरदन गनतववचि
हैं जजन्हें ग्लेसशयर के आिार पर उकेरा जाता है
द्वारा ननसमसत होता है ।
क्योंकक यह पररदृवय को समटा दे ता है ।
• कथन 2 र्गलत है : सकस एक कटोरे के आकार का
• ववकल्प 3 ही है : वपरासमड सशखर, जजसे कभी-कभी
अवसाद है जो एक अल्पाइन ग्लेसशयर द्वारा पहाड
चरम मामलों में दहमनद श्र्ग
ं ृ कहा जाता है , एक
से उकेरा गया है । सकस के भीतर कभी-कभी केंद्रीय
कोणीय, तेज नुकीला पवसत सशखर है जो एक केंद्रीय
अवसाद में एक झील पाई जाती है , जजसे टानभ कहा
ब्रबंद ु से कई ग्लेसशयरों के ववचलन के कारण चक्कर
जाता है ।
के कटाव के पररणामस्वरूप होता है । वपरासमड की
• कथन 3 ही है : हहमरे खा वह रे खा है (जैसा कक एक
चोहटयााँ अक्सर नुनाटक के उदाहरण हैं। ये तब बनते
पहाड पर उन्नयन के ब्रबंद ु पर) जजस पर चगरने वाली
हैं जब एक पवसत के ककनारे पर तीन या अचिक कोरी
बफस की मात्रा गसमसयों में बफस के वपघलने की मात्रा के
बनते हैं।
बराबर होती है । ग्लेसशयर जलवायु पररवतसन के
• ववकल्प 4 ही है : टानभ एक छोटी पहाडी झील है ,
संकेतक के रूप में महत्वपूणस हैं क्योंकक भौनतक
ववशेर् रूप से एक हहमाच्छाहदत खडी दीवार वाले
पररवतसन ग्लेसशयरों में चाहे वे बढ़ रहे हैं या ससकुड
एम्फीचथएटर में एक सेट जजसे सकस के रूप में जाना
रहे हैं, आगे बढ़ रहे हैं या घट रहे हैं, तापमान और वर्ास
जाता है ।
में पररवतसन के दृवय प्रमाण प्रदान करते हैं।
Reference: Fundamental of physical geography (11th
Reference: https://www.epa.gov NCERT) page no. 58 -73

Q.35) Ans: D Q.36) Ans: D


Exp: Exp:
• ववकल्प 1 ही है : सकस एक प्रकार का हहमनदीय कथन 1 ही है : चूना पत्थर के ववघटन से कंदराओं का ननमासण
अपरदन स्थलरूप है । इसे कोरी के नाम से भी जाना होता है । वर्ास का पानी हवा से काबसन डाइऑक्साइड अवशोवर्त
जाता है । वे गहरे , लंबे और चौडे कंु ड या बेससन होते हैं है और जैसे ही यह समट्टी के माध्यम से ररसता है , जो एक
जजनमें बहुत खडी अवतल होती हैं जो इसके ससर के कमजोर एससड में बदल जाता है । यह िीरे -िीरे जॉइन्ट्स,
साथ-साथ ककनारों पर ऊंची दीवारों को लंबवत रूप से बेडडंग प्लेन और फ्रैक्चर के साथ चूना पत्थर को घोल दे ता है ,
चगराती हैं। एक सकस मूल रूप से एक कटोरे के आकार जजनमें से कुछ गुफाओं का ननमासण करने के सलए पयासप्त रूप
का अवसाद है जो ग्लेसशयर की अपरदन गनतववचि से बढ़ जाते हैं।
द्वारा ननसमसत होता है । सकस एक कटोरे के आकार का कथन 2 ही है : भूजल रासायननक अपक्षय के माध्यम से
अवसाद है जो एक अल्पाइन ग्लेसशयर द्वारा पहाड क्षरण का कारण बन सकता है। सबसे पहले, पानी जो जमीन
से उकेरा गया है । सकस के भीतर कभी-कभी केंद्रीय में ररसता है , काबसन डाइऑक्साइड के साथ समलकर एक

DPP 2023 DAY 48 36


Contact us : info@onlyias.com

OnlyIAS Nothing Else Visit : dpp.onlyias.in


Contact : +91-7007 931 912

कमजोर एससड काबोननक एससड बनाता है । समय के साथ, ये पररवतसन के कारण यांब्रत्रक और रासायननक अपक्षय
पॉकेट्स भूसमगत बडे छे दों में ववकससत हो जाते हैं - गुफाएाँ या प्रकियाओं के संपकस में आती हैं, तेजी से क्षय होती हैं और
कंदराएाँ। मूसलािार बाररश अपक्षय सामग्री को आसानी से हटाने में
कथन 3 ही है : गफ
ु ाओं के अंदर बनने वाली दो संरचनाएं मदद करती है यानी रे चगस्तान में अपक्षनयत मलबा न केवल
स्टै लेग्माइट् और स्टै लक्
े टाइट् सबसे प्रससद्ि गफ
ु ा हवा से बजल्क बाररश से भी चलता है । .
संरचनाओं में से कुछ हैं। वे बफस के आकार के जमा होते हैं जो
तब बनते हैं जब पानी चूना पत्थर के ऊपर से घुल जाता है और ंदर्भ: भौनतक भूगोल का मल
ू (11वीं एनसीईआरटी) पष्ृ ठ

कफर नीचे की गुफाओं की छत या फशस के साथ कैजल्शयम संख्या। 58 -73

काबोनेट को कफर से जमा कर दे ता है ।.


Q.38) Ans: C
Reference: https://lisbdnet.com/how-does- Exp:
groundwater-cause-caves-to-
• कथन 1 ही है: पहाडों के पास के मंद ढलान ,पेडीमें ट
form/#Are_caves_formed_by_groundwater
कहलाते हैं। यह पाववस अपरदन या यांब्रत्रक अपक्षय द्वारा
ननसमसत होता है । ववसशष्ट पेडडमें ट में 0.5 और 7 डडग्री के कोण
Q.37) Ans: C
Exp: के साथ ढलान होते हैं और आकार में अवतल होते हैं।
• कथन 1 ही है : हवा की क्षैनतज गनत को सूयस के गमस होने • कथन 2 र्गलत है : पेडडमें ट का ननमासण शष्ु क और अिस-शुष्क
और पथ्
ृ वी की घण
ू न
स गनत के कारण होने वाली हवा के रूप में क्षेत्रों में सबसे अच्छा होता है , जहां थोडे समय के सलए वर्ास तीव्र
वखणसत ककया गया है । रे चगस्तानी फशस शुष्क और बंजर होने होती है । नम क्षेत्रों में बनने वाले पेडडमें ट आमतौर पर
के कारण बहुत अचिक और बहुत जल्दी गमस हो जाते हैं। गमस वनस्पनत द्वारा अस्पष्ट होते हैं और उनका ननरीक्षण कर
फशस सीिे अपने ऊपर की हवा को गमस करते हैं और पाना मजु वकल हो सकता है ।
पररणामस्वरूप गमस हल्की हवा में अशांनत के साथ ऊपर की • कथन 3 ही है : रूप और कायस में पेडडमें ट और जलोढ़ पंख
ओर गनत होती है , और इसके रास्ते में कोई भी बािा एडी, में कोई अंतर नहीं है; हालााँकक, पेडडमें ट एक अपरदनात्मक
बवंडर, अपड्राफ्ट और डाउनड्राफ्ट सेट करती है । एडीज हवा लैंडफॉमस है जबकक पंख एक कंस्रक्शनल लैंडफामस है । पथ्
ृ वी
की एक गोलाकार गनत है जो छोटे भाँवर पैदा करती है । बवंडर की पपसटी के ववकास के ऑरोजेननक चरण से लेकर संिमण
हवा का एक स्तंभ है जो बेलनाकार या कीप के आकार में तक के दौरान प्रमुख वववतसननक चिों के अंत में पेडडप्लेन
तेजी से गोल-गोल घूमता है । अपड्राफ्ट हवा का एक ऊपर की बनते हैं।
ओर प्रवाह है । डॉवंड्राफ्ट हवा की नीचे की ओर बहने वाली
ंदर्भ: भौनतक भूगोल का मल
ू (11वीं एनसीईआरटी) पष्ृ ठ
िारा है ।
संख्या। 58 -73
• कथन 2 र्गलत है : चट्टानों की सतह से िूल और छोटे कणों
को उठाना और हटाना अपस्फीनत कहलाता है । महीन िूल
और रे त , मूल स्थान से मीलों दरू हट जाती है , और रे चगस्तान Q.39) Ans: A
Exp:
के बाहर जमा ककया जा सकता है । पवनों द्वारा चट्टानों की
• ववकल्प A ही है : चट्टानों या नंगी समट्टी के ऊपर से
सतहों का रे त-ववस्फोट जब वे रे त के कणों को उनके ववरुद्ि
अपक्षनयत मेंटल, एक हदशा में हवा की िाराओं के लगातार
फेंकते हैं, घर्सण कहलाते हैं।
चलने से उड जाता है , इसे डडफ्लेशन हॉलो कहा जाता है ।
• कथन 3 ही है : मरुस्थल में वर्ास कम होती है , लेककन यह
• ववकल्प B र्गलत है : पेडस्टल रॉक को मशरूम रॉक या रॉक
बहुत कम समय के सलए होती है । रे चगस्तानी चट्टानें
पेडस्टल के रूप में भी जाना जाता है । ये चट्टानें हवा से चलने
,वनस्पनत से रहहत होती हैं, जो दै ननक तापमान में भारी
वाली रे त से नछन्न-सभन्न, गढ़ी हुई, नक़्काशीदार, उभरी हुई

DPP 2023 DAY 48 37


Contact us : info@onlyias.com

OnlyIAS Nothing Else Visit : dpp.onlyias.in


Contact : +91-7007 931 912

या पॉसलश की जाती हैं। रे त और चट्टान के कणों की एक बडी बहुत लिंबी अवचध में नहदयों या नालों द्वारा भमू म की सतह के
मात्रा को हवाओं द्वारा जमीन के करीब ले जाया जाता है जो कटाव से बनती हैं।
ऊपर से नीचे की ओर अचिक होता है । इसके पररणामस्वरूप 0 घाहटयााँ छोटी और सिंकरी नालों के रूप में शुरू होती हैं, नहदयााँ
मशरूम जैसे आकार में चट्टान के खंभे बन जाते हैं। धीरे -धीरे लिंबी और ौडी नामलयों में ववकमसत होंगी, घाहटयों
• ववकल्प C र्गलत है : जब रे चगस्तान में उच्चावच संरचनाएं को जनम दे ने के मलए गमलयााँ और गहरी, ौडी और लिंबी होंगी।
हवा की गनतववचियो से, मैदानों में कम हो जाती हैं, तो उन्हें o कवठ एक गहरी घाटी है स्जसमें बहुत सीधी भुजाएाँ होती हैं
पेडडप्लेन कहा जाता है । पथ्ृ वी की पपसटी के ववकास के और एक घाटी की ववशेर्ता खडी सीढीनुमा पाश्वव ढलान है ।
ऑरोजेननक चरण से मंच चरण तक संिमण के दौरान प्रमख
ु o इसके ववपरीत, एक घाटी इसके नी े की तुलना में इसके
वववतसननक चिों के अंत में पेडडप्लेन बनते हैं। शीर्व पर पाई जाती है । वावतव में , घाटी कवठ का एक प्रकार है ।
• ववकल्प D र्गलत है : पहाडों के पास िीरे -िीरे झक
ु े हुए घाटी के प्रकार ट्टानों के प्रकार और सिंर ना पर ननभवर करते
चट्टानी फशस उनके पैरों पर मलबे के पतले आवरण के साथ हैं स्जसमें वे बनते हैं। उदाहरण के मलए, घाटी आमतौर पर
या ब्रबना पेडडमें ट कहलाते हैं। क्षैनतज बबवतरों वाली तलछटी ट्टानों में बनती है और कठोर
ट्टानों में घाहटयााँ बनती हैं।
ंदर्भ: भौनतक भूगोल का मल
ू (11वीं एनसीईआरटी) पष्ृ ठ
संख्या। 58 -73 सिंदभव: भौनतक भूगोल का मल
ू (11वीिं एनसीईआरटी) पष्ट्ृ ठ
सिंख्या। 58 -73

Q.40) Ans: D
Exp:
• कथन 1 गित है : अनुप्रवथ टीले हवा की हदशा के लिंबवत Q.42) Ans: C
सिंरेखखत होते हैं। ये टीले तब बनते हैं जब हवा की हदशा स्वथर Exp:
होती है और रे त का स्रोत हवा की हदशा के समकोण पर होता • वववकल्प A गित है : पवन, हवा का ऐसा संचलन है , जो

है । सय
ू व द्वारा पथ्
ृ वी के असमान ताप और पथ्
ृ वी के ववयिं के

• कथन 2 गित है : जब रे तीली सतह आिंमशक रूप से वनवपनत घूणन


व के कारण होती है । हवाओिं के कारण होने वाली कुछ

से आच्छाहदत होती है तो परवलनयक टीले बनते हैं। ये टीले अपरदनात्मक भू-आकृनतयााँ हैं, इरोडेड एररड लैंडफॉमव –

एक ही हवा की हदशा के साथ उलटे बर न हैं। वे य-ू आकार के अपवफीनत बेमसन, मशरूम ट्टानें, इनसेलबगव, डेमोइसेल,

होते हैं और बर नों की तुलना में बहुत लिंबे और सिंकरे होते हैं। डेमोइसेल, ज़ुगेन, वविंड बब्रज और खखडककयािं। ननक्षेपी शुष्ट्क

• कथन 3 गित है : रे त की आपूनतव कम होने और हवा की हदशा भू-आकृनतयााँ – लहर के ननशान, रे त के टीले, अनद
ु ै ध्यव टीले,

स्वथर होने पर अनद अनुप्रवथ टीले, बर न, परवलनयक हटब्बा, वटार हटब्बा और


ु ै ध्यव टीले बनते हैं।
लोएस।
संिर्ा: भौनतक भग
ू ोल का मल
ू (11वीिं एनसीईआरटी) पष्ट्ृ ठ • ववकल्प B गित है: लहरें अननवायव रूप से समद्र
ु के भीतर
सिंख्या। 58 -73 पानी के अणुओिं की गनत हैं, और हमारे महासागरों और
समुद्रों की सतह परतों तक ही सीममत हैं। समुद्री लहरों और

Q.41) Ans: D अनय धाराओिं और समाधान प्रकियाओिं द्वारा समुद्री क्षरण


Exp: के कारण बनने वाली तटीय ववशेर्ताओिं में ट्टानें, कोव्स,
• उपरोक्त सभी कथन सही हैं: गुफाएिं, इिंडेंटेड कोवटलाइन, वटै क्स, च मनी, आ ,व इनलेट,
o घाटी अक्सर पहाडडयों या पहाडों के बी लने वाला एक वेव-कट प्लेटफॉमव आहद शाममल हैं।
लिंबा नन ला क्षेत्र होता है , स्जसमें आमतौर पर एक नदी या • ववकल्प C सही है : प्राकृनतक लेवे और प्वाइिंट बार कुछ
धारा एक छोर से दस
ू रे छोर तक जाती है । अचधकािंश घाहटयााँ महत्वपण
ू व भ-ू आकृनतयािं हैं जो बाढ के मैदानों से जड
ु ी हुई हैं।

DPP 2023 DAY 48 38


Contact us : info@onlyias.com

OnlyIAS Nothing Else Visit : dpp.onlyias.in


Contact : +91-7007 931 912

प्राकृनतक जलधाराएाँ बडी नहदयों के ककनारे पाई जाती हैं। वे o सकसस कटोरे के आकार के, एम्फीचथएटर जैसे अवसाद होते
नहदयों के ककनारे मोटे ननक्षेपों की नन ली, रै खखक और हैं जो हहमनदों को पहाडों और घाटी के ककनारे उच्च ऊंचाई पर
समानािंतर लकीरें हैं, स्जनहें अक्सर अलग-अलग टीले में काट बनाते हैं। दं गल ग्लेसशयरों की अपरदनकारी भू-आकृनतयााँ हैं।
हदया जाता है । प्वाइिंट बार को मेनडर बार के रूप में भी जाना o एस्कसस रे त और बजरी से बनी लकीरें हैं, जो ग्लेसशयरों के
जाता है । वे बडी नहदयों के ककनारे के अवतल ककनारे पर पाए भीतर और नीचे सुरंगों के माध्यम से बहने वाले हहमनदों के
जाते हैं और तट के ककनारे बहते पानी द्वारा एक रे खीय वपघले पानी या ग्लेसशयरों के ऊपर वपघले पानी के चैनलों के
फैशन में जमा तलछट हैं। वे प्रोफाइल और ौडाई में लगभग माध्यम से जमा होती हैं। एस्कसस हहमनदों की ननक्षेपी भू-
समान हैं और ममचित आकार के तलछट होते हैं। आकृनतयााँ हैं।
• ववकल्प D गित है : एक ग्लेमशयर किवटलीय बफव, o प्लेआस एक सख
ू ा, वनस्पनत-मक्
ु त, समतल क्षेत्र है जो एक
ट्टान, तलछट, और अक्सर तरल पानी का एक बडा, अससंचचत मरुस्थलीय बेससन के सबसे ननचले भाग पर जस्थत
बारहमासी सिं य है जो जमीन पर उत्पनन होता है और अपने है । यह एक ऐसा स्थान है जहां अल्पकासलक झीलें गीली
वजन और गुरुत्वाकर्वण के प्रभाव में ढलान से नी े ला अवचि के दौरान बनती हैं, और स्तरीकृत समट्टी, गाद और
जाता है । हहमनदों के कारण होने वाली अपरदन भू-आकृनतयााँ रे त, और आमतौर पर घुलनशील लवणों के नीचे होती हैं।
हैं, क्कर, सीिंग और दााँतेदार लकीरें , ग्लेमशयर घाटी / किंु ड। समद्र
ु तटों हवाओं की अपरदनात्मक स्थलाकृनतयााँ हैं।
ननक्षेवपत भू-आकृनतयााँ मोराइनस, एवकसव, आउटवाश प्लेन,
ड्रमलाइनस हैं। ंदर्भ: भौनतक भग
ू ोल का मल
ू (11वीं एनसीईआरटी) पष्ृ ठ

संिर्ा: भौनतक भग संख्या। 58 -73


ू ोल का मल
ू (11वीिं एनसीईआरटी) पष्ट्ृ ठ
सिंख्या। 58 -73
Q.44) Ans: D
Exp:
Q.43) Ans: A
• उपरोक्त र्ी कथन ही हैं:
Exp:
• ववकल्प 1 और 2 ही हैं: 0 भ-ू आकृनत की प्रारं सभक अवस्था में , यहद वर्ास होती है , तो

o जलोढ़ पंख बजरी, रे त और यहां तक कक तलछट के छोटे - वर्ास का पानी दीवारों, नालों, नालों और अंततः नहदयों के रूप

छोटे टुकडों जैसे गाद का एक ब्रत्रभुज के आकार का ननक्षेप में प्राकृनतक ढलान के साथ समुद्र की ओर बहने लगेगा और

होता है । इस तलछट को जलोढ़ कहा जाता है । जलोढ़ पंख नीचे की ओर कटाव हावी हो जाता है ।

आमतौर पर पहाडों, पहाडडयों या घाहटयों की खडी दीवारों के o भू-आकृनत की पररपक्व अवस्था में , िाराएाँ अच्छे एकीकरण

साथ बहने वाले पानी के रूप में बनाए जाते हैं। जब कोई नदी के साथ भरपूर होती हैं। घाहटयााँ अभी भी वी-आकार की हैं

ककसी झील या समद्र लेककन गहरी हैं; रं क िाराएं व्यापक बाढ़ के मैदानों के सलए
ु में पहुाँचती है तो पानी िीमा हो जाता है
और तलछट ले जाने की शजक्त खो दे ता है । तलछट , नदी के पयासप्त हैं, जजसके भीतर घाटी के भीतर सीसमत मेन्डसस में

मुहाने पर चगराया जाता है । कुछ नहदयााँ इतनी तलछट िाराएं बह सकती हैं। समतल और ववस्तत
ृ अंतिासरा क्षेत्र के

चगराती हैं कक लहरें और ज्वार इसे दरू नहीं ले जा सकते। यह दलदल गायब से हो जाते हैं और िारा का ववभाजन तेज हो

डेल्टा बनाने वाली परतों में बनता है । जाते हैं। झरने और रै वपड्स गायब हो जाते हैं।

o नदी की छतों के ननक्षेपण स्थल रूप हैं जलोढ़ पंखे, डेल्टास, 0 भ-ू आकृनत के पुराने चरण में , नहदयााँ अपने बाढ़ के मैदानों

बाढ़ के मैदान, प्राकृनतक लेवे और पॉइंट बार, मेन्डसस, िेडेड से टे ढ़ी-मेढ़ी तरीके से बहती हैं। भू-आकृनत इतनी कम राहत

चैनल। और मंद ढलान प्राप्त करती है कक यह लगभग एक लहरदार

• ववकल्प 3, 4 र्गलत हैं: मैदान बन जाती है जजसे पेनप्लेन के रूप में जाना जाता है । परू ा

DPP 2023 DAY 48 39


Contact us : info@onlyias.com

OnlyIAS Nothing Else Visit : dpp.onlyias.in


Contact : +91-7007 931 912

क्षेत्र भारी मात्रा में तलछट से आच्छाहदत हो जाता है और क्षेत्र • गरु


ु त्वाकर्सण बल जैसे ओवरबडसन
का ढलान बहुत कोमल हो जाता है । दबाव, भार और कतरनी तनाव;

Q.45) Ans: C 1. तापमान पररवतसन, किस्टल वद्


ृ चि या पशु
Exp:
गनतववचि के कारण ववस्तार बल;
• कथन 1 ही है :
2. पानी के दबाव को गीला करने और सुखाने के
ओ स्तंर्:
चिों द्वारा ननयंब्रत्रत ककया जाता है ।
• स्टै लेक्टाइट्स और स्टै लेग्माइट्स के संयोजन या
संलयन से स्तंभ बनते हैं। केवल आईएएस द्वारा अनतररक्त बढ़त
• खंभों के व्यास अलग-अलग होते हैं।
उतराई और ववस्तार

• ननरं तर कटाव के कारण रॉक लोड को कवर करने


का उन्मूलन ऊध्वासिर दबाव ररलीज का कारण
बनता है जजसके पररणामस्वरूप चट्टान की ऊपरी
परत चट्टान द्रव्यमान के ववखंडन का उत्पादन
करती है ।
• ये तनक्षेपी र्-ू आकृततयााँ हैं।
• कथन 2 ही है : हहमाच्छाहदत पहाडों में सकसस सबसे
• फ्रैक्चर जमीन की सतह के लगभग समानांतर
आम भू-आकृनतयां हैं। सकसस अक्सर हहमनद घाहटयों के
होंगे।
शीर्स पर पाए जाते हैं। पहाड की चोहटयों से नीचे जाते
समय जमा हुई बफस इन चिों को काट दे ती है । वे गहरे , • घुमावदार जमीन की सतह के क्षेत्रों में ,
लंबे और चौडे कंु ड या बेससन होते हैं जजनमें बहुत खडी िनुर्ाकार फ्रैक्चर चट्टान की ववशाल चादरें या
अवतल होती है जो इसके ससर के साथ-साथ ककनारों पर एक्सफोसलएशन स्लैब बनाने के सलए झुकते हैं।
ऊंची दीवारों को खडी करती है ।
• दबाव छोडने और उतारने के कारण ववस्तार के
ंदर्भ : 11वीं कक्षा ch-07 भौनतक भग
ू ोल NCERT
कारण छूटने वाली चादरें क्षैनतज सीमा में सैकडों
या हजारों मीटर तक माप सकती हैं।
Q.46) Ans: D
Exp: • बडे, चचकने गोल गुम्बदों को एक्सफोसलएशन
• र्ी कथन ही हैं: मौसम और जलवायु के
डोम कहा जाता है ।
ववसभन्न तत्वों की कियाओं के माध्यम से अपक्षय
को यांब्रत्रक ववघटन और चट्टानों के रासायननक तापमान पररवतसन और ववस्तार
अपघटन के रूप में पररभावर्त ककया गया है ।
• चट्टानों में कई खननजों के संकुचन और ववस्तार
o अपक्षय प्रकियाओं के तीन प्रमुख समूह हैं:
की अपनी सीमाएं होती हैं।
0. रासायननक;
1. भौनतक या यांब्रत्रक;
• तापमान में वद्
ृ चि के साथ, सभी खननज बढ़
2. जैववक अपक्षय प्रकियाएं।
जाते हैं और अपने पडोसी के खखलाफ जोर दे ते हैं
o भौनतक या यांब्रत्रक अपक्षय प्रकियाएं कुछ लागू
बलों पर ननभसर करती हैं। लागू बल हो सकते हैं:

DPP 2023 DAY 48 40


Contact us : info@onlyias.com

OnlyIAS Nothing Else Visit : dpp.onlyias.in


Contact : +91-7007 931 912

ववककरण करने वाले हहमनद अपने ससरों के समलने तक ससर


और जैसे ही तापमान चगरता है, एक समान
की ओर काटते हैं, तो ऊाँची, नुकीले और खडी ककनारों वाली
ससकुडन होती है ।
चोहटयााँ हॉनस कहलाती हैं। सकसस की ओर की दीवारों या हे डवॉल
• तापमान में दै ननक पररवतसन के कारण चट्टानों के बीच के ववभाजन प्रगनतशील क्षरण के कारण संकीणस हो
की सतही परतों के खननज कणों के बीच यह जाते हैं और दााँतेदार या आरी-दांतेदार लकीरों में बदल जाते हैं
आंतररक गनत बार-बार होती है। जजन्हें कभी-कभी बहुत तेज सशखाओं और एक ज़ग-़ैग
रूपरे खा के साथ आटे स कहा जाता है ।
• यह प्रकिया उच्च ऊंचाई और शुष्क जलवायु में
प्रभावी होती है जहां दै ननक तापमान सभन्नता
चरम पर होती है ।

फ्रीजजंग, ववगलन और फ्रॉस्ट वेजजंग



• फ्रॉस्ट अपक्षय वपघलने और जमने के आवतसक • कथन 2 ही है : भारत का पजवचमी तट एक उच्च
चिों के दौरान चट्टानों के उद्घाटन और दरारों के चट्टानी पीछे हटने वाला तट है । कटाव के रूप
भीतर बफस के ववकास के कारण होता है । पजवचमी तट पर हावी हैं। भारत का पूवी तट एक
ननम्न अवसादी तट है । पूवी तट पर ननक्षेपण रूप
• यह प्रकिया मध्य-अक्षांशों में उच्च ऊंचाई पर हावी हैं।
प्रभावी होती है जहां वपघलने और ठं ड अक्सर
Depositional Erosional
आवती होती है । forms. landforms

• हहमनद क्षेत्र हर हदन फ्रॉस्ट वेजजंग के अिीन


होते हैं।

• इस पाठ्यिम में , ठं ड की दर महत्वपण


ू स है ।

• पानी के जल्दी जमने से उसका उच्च दाब और


Q.48) Ans: a
तेजी से ववस्तार होता है ।
Exp:
• कथन 1 र्गलत है : सभी बहहजासत भू-आकृनत प्रकियाएं
• पररणामी ववस्तार जोडों, दरारों और छोटे
एक सामान्य शब्द, अनाच्छादन के अंतगसत आती हैं।
इंटरग्रेन्युलर फ्रैक्चर को प्रभाववत करता है और
अपक्षय, बडे पैमाने पर बबासदी/आंदोलन, कटाव और
चट्टान के टूटने तक चौडा और चौडा हो जाता है ।
पररवहन को अनाच्छादन में शासमल ककया गया है ।
. बहहजासत भू-आकृनत प्रकियाएं एक क्षेत्र से दस
ू रे क्षेत्र में
सभन्न होती हैं। वनस्पनत का घनत्व, प्रकार और ववतरण
ंदर्भ : 11वीं कक्षा ch – 06 भौनतक भूगोल NCERT जो काफी हद तक वर्ास और तापमान पर ननभसर करता है ,
परोक्ष रूप से बहहजासत भू-आकृनत प्रकियाओं पर प्रभाव
Q.47) Ans: A डालता है ।
Exp:
• कथन 2 ही है : तापमान पररवतसन से प्रेररत ववस्तार
• कथन 1 र्गलत है : बान्सस, हहमनदों द्वारा ननसमसत
और संकुचन के कारण , अपशल्कन हो सकता है ।
अपरदनात्मक भ-ू आकृनतयां हैं। यहद तीन या अचिक

DPP 2023 DAY 48 41


Contact us : info@onlyias.com

OnlyIAS Nothing Else Visit : dpp.onlyias.in


Contact : +91-7007 931 912

अपशल्कन, एक पररणाम है लेककन एक प्रकिया नहीं है । आिारसशला और अपक्षय के उत्पादों पर। इससलए, जन
घुमावदार जमीन की सतह के क्षेत्रों में, िनुर्ाकार फ्रैक्चर आंदोलन के सलए अपक्षय कोई पूवासपेक्षा नहीं है , हालांकक यह
बडे पैमाने पर चादरें या चट्टान के छूटने वाले स्लैब का जन आंदोलनों में सहायता करता है । अपक्षनयत सामग्री के
उत्पादन करते हैं। उतराई और दबाव मुक्त होने के कारण बजाय अपक्षनयत ढलानों पर जन आंदोलन बहुत सकिय हैं।
ववस्तार के पररणामस्वरूप छूटने वाली चादरें क्षैनतज
• कथन 2 ही है : यहद चट्टानें पारगम्य और घुलनशील हैं
सीमा में सैकडों या हजारों मीटर तक माप सकती हैं। इस
और पानी उपलब्ि है तभी कास्टस स्थलाकृनत ववकससत होती
प्रकिया के कारण बडे, चचकने गोल गुंबद, जजन्हें
है । एडड्रयाहटक समद्र
ु से सटे बाल्कन में कास्टस क्षेत्र के चूना
एक्सफोसलएशन डोम कहा जाता है , का पररणाम होता है ।
पत्थर चट्टानों में ववकससत ववसशष्ट स्थलाकृनत के बाद
ंदर्भ : 11वीं कक्षा ch-06 र्ौततक र्र्ग
ू ोल एन ीईआरटी
समािान और ननक्षेपण की प्रकियाओं के माध्यम से भज
ू ल
की किया द्वारा ननसमसत ववसशष्ट भू-आकृनतयों को हदखाने
Q.49) Ans: d
Exp: वाला कोई भी चूना पत्थर या डोलोसमहटक क्षेत्र कास्टस
स्थलाकृनत कहलाता है ।
हाइड्रेशन
ंदर्भ : 11वीं कक्षा ch – 06 और 07 र्ौततक र्ूर्गोल NCERT
• जलयोजन ,जल का रासायननक योग है । यह प्रकिया
प्रनतवती और लंबी है , इस प्रकिया की ननरं तर पुनराववृ त्त
Q.51) Ans: a
चट्टानों के ववघटन का कारण बन सकती है । कई समट्टी के Exp:
खननज ससकुड जाते हैं और इस प्रकिया की पुनराववृ त्त के कथन 1 ही है :ड्रमसलन चचकनी अंडाकार आकार की ररज
पररणामस्वरूप ऊपर की सामग्री में दरार आ जाती है । जैसी ववशेर्ताएं होती हैं जो मख्
ु य रूप से हहमनदों से बनी होती
रोमनछद्रों में लवण तेजी से और बार-बार जलयोजन से हैं, जब तक कक बजरी और रे त के कुछ द्रव्यमान होते हैं।
गज
ु रते हैं और रॉक फ्रैक्चररंग में मदद करते हैं। जलयोजन ड्रमसलन की लंबी कुल्हाडडयां बफस की गनत की हदशा के
के कारण खननजों में मात्रा में पररवतसन भी छूटना और समानांतर होती हैं। ग्लेसशयर में दरारों के माध्यम से भारी भरी
दानेदार ववघटन के माध्यम से भौनतक अपक्षय में मदद हुई बफस के नीचे रॉक मलबे के डंवपंग के कारण ड्रमसलन का
करे गा। ननमासण होता है ।
कथन 2 र्गलत है : जब नहदयााँ मोटे पदाथस को ले जाती हैं, तो
ंदर्भ : 11वीं कक्षा ch – 06 र्ौततक र्ूर्गोल NCERT
मोटे पदाथों का चयनात्मक ननक्षेपण हो सकता है जजससे एक
केंद्रीय पट्टी बन जाती है जो प्रवाह को ककनारों की ओर मोड
दे ती है ; और यह प्रवाह ककनारों पर पाववस क्षरण को बढ़ाता है ।
Q.50) Ans: b
Exp: जैस-े जैसे घाटी चौडी होती है , पानी का स्तंभ कम होता जाता
है और अचिक से अचिक सामग्री द्वीपों और पाववस सलाखों के
• कथन 1 र्गलत है : मौसम और जलवायु के ववसभन्न तत्वों रूप में जमा हो जाती है जजससे जल प्रवाह के कई अलग-अलग
की कियाओं के माध्यम से अपक्षय को यांब्रत्रक ववघटन और चैनल ववकससत होते हैं। िेडेड पैटनस के ननमासण के सलए बैंकों
चट्टानों के रासायननक अपघटन के रूप में पररभावर्त ककया का जमाव और पाववस क्षरण आववयक है ।
गया है । द्रव्यमान की गनत िीमी से तीव्र तक हो सकती है , ंदर्भ : 11वीं कक्षा ch- 07 र्ौततक र्र्ग
ू ोल NCERT
सामग्री के उथले से गहरे स्तंभों को प्रभाववत करती है और
इसमें रें गना, प्रवाह, स्लाइड जैसी प्रकियाएं शासमल है ।
Q.52) Ans: b
गुरुत्वाकर्सण सभी पदाथों पर अपना बल लगाता है , दोनों
Exp:

DPP 2023 DAY 48 42


Contact us : info@onlyias.com

OnlyIAS Nothing Else Visit : dpp.onlyias.in


Contact : +91-7007 931 912

कथन 1 र्गलत है :समग्र ज्वालामख


ु ी • मध्य-महासागर ररज ज्वालामुखी

▪ समचित ज्वालामख
ु ी बेसाल्ट की तुलना में कूलर और • ये ज्वालामख
ु ी समद्र
ु ी क्षेत्रों में पाए जाते हैं।
अचिक चचपचचपे लावा के प्रकोप की ववशेर्ता है । • समुद्र के घाहटयों में 70000 ककमी से अचिक
तक फैले मध्य-महासागरीय कटक की एक
▪ इनका ननमासण अनेक ववस्फोटक ववस्फोटों से हुआ है ।
प्रणाली मौजद
ू है ।
▪ बडी मात्रा में पाइरोक्लाजस्टक सामग्री और राख लावा के • इस कटक के मध्य क्षेत्र में बार-बार ववस्फोट
साथ जमीन में समल जाती है । होते रहते हैं।

▪ यह सामग्री वेंट के उद्घाटन के पास इकट्ठा होती है


ंदर्भ : 11वीं कक्षा ch - 03 र्ौततक र्ूर्गोल NCERT
जजसके पररणामस्वरूप परतों का ननमासण होता है ।

▪ कफलीपींस में मायोन ज्वालामख


ु ी, जापान में माउं ट फूजी Q.53) Ans: d
और वासशंगटन में माउं ट रे ननयर दनु नया के प्रमुख Exp:
समचित ज्वालामख
ु ी हैं।
• कथन 1 ही है : मलबे के हहमस्खलन की ववशेर्ता आद्रस
▪ प्रमुख संयुक्त ज्वालामुखी िख
ं ृ लाएं प्रशांत ररम हैं जजन्हें क्षेत्रों के साथ या वनस्पनत आवरण के ब्रबना होती है और यह
“आग का ररम” के रूप में जाना जाता है । खडी ढलानों पर संकरी पटररयों में होता है । मलबे का
हहमस्खलन मडफ्लो की तुलना में बहुत तेज हो सकता है ।
कथन 2 ही है :ओ काल्डेरा मलबा हहमस्खलन हहमस्खलन के समान है ।

▪ काल्डेरा को पथ्
ृ वी पर सबसे ववस्फोटक ज्वालामुखखयों o मलबा हहमस्खलन तीव्र प्रवाह जन आंदोलनों के
के रूप में जाना जाता है । प्रकारों में से एक है । तीव्र गनतयााँ ज्यादातर आद्रस
जलवायु क्षेत्रों में प्रचसलत हैं और कोमल से खडी
▪ वे आम तौर पर ववस्फोटक होते हैं।
ढलानों पर होती हैं।
▪ जब वे फटते हैं, तो वे ककसी भी संरचना का ननमासण करने
• कथन 2 ही है : मडफ्लो पानी का प्रवाह है जजसमें बडी
के बजाय अपने आप चगर जाते हैं।
मात्रा में ननलंब्रबत कण और गाद होती है । मडफ्लो खडी
▪ ढह गए गड्ढों को काल्डेरास के रूप में जाना जाता है ढलानों पर होते हैं जहां वनस्पनत तेजी से कटाव को रोकने के
सलए पयासप्त नहीं है , लेककन अन्य शतों को पूरा करने पर मंद
ढलानों पर हो सकता है । अन्य कारक छोटी अवचि में भारी
केवल आईएए द्वारा अततररक्त बढ़त वर्ास और आसानी से नष्ट होने योग्य स्रोत सामग्री हैं।
• बाढ़ बेसाल्ट प्रांत मडफ्लो ककसी भी जलवायु व्यवस्था में उत्पन्न हो सकते हैं

बाढ़ बेसाल्ट प्रांत ज्वालामख लेककन शुष्क और अिस-शुष्क क्षेत्रों में सबसे आम हैं। मडफ़्लो
• ु ी अत्यचिक तरल
लावा का ननवसहन करता है जो लंबी दरू ी तक अक्सर फटने वाले या हाल ही में फटे ज्वालामखु खयों की

बहता है । ढलानों पर होते हैं। जब मडफ्लो चैनलों से पीडमोंट या मैदानी


इलाकों में ननकलते हैं, तो वे बहुत ववनाशकारी, सडकों, पुलों
• वववव के कई हहस्से मोटे बेसाल्ट लावा प्रवाह से
और घरों को घेर सकते हैं।
आच्छाहदत हैं।

DPP 2023 DAY 48 43


Contact us : info@onlyias.com

OnlyIAS Nothing Else Visit : dpp.onlyias.in


Contact : +91-7007 931 912

ंदर्भ : 11वीं कक्षा ch – 06 र्ौततक र्र्ग


ू ोल NCERT से क्षैनतज रूप से स्लाइड करती हैं। रांसफॉमस बाउं ड्री़ भक
ू ंप
फॉल्ट ़ोन बनाती हैं। कैसलफोननसया का सैन एंडड्रयास फॉल्ट
एक रांसफॉमस बाउं ड्री है ।
Q.54) Ans: b • जोडी 2 ही है : असभसारी सीमाएाँ वह हैं जहााँ दो या अचिक
Exp:
स्थलमंडलीय प्लेटें टकराती हैं। एक प्लेट दस
ू रे के नीचे
• कथन 1 ही है : बाढ़ बेसाल्ट प्रांत खखसकती है , जजसे सबडक्शन के रूप में जाना जाता है । तीन
प्रकार की असभसरण सीमाओं को मान्यता दी जाती है :
बाढ़ बेसाल्ट प्रांत ज्वालामख
ु ी अत्यचिक तरल लावा का महाद्वीप-महाद्वीप, महासागर-महाद्वीप और महासागर-
ननवसहन करता है जो लंबी दरू ी तक बहता है । सागर।
• जोडी 3 ही है : अपसारी सीमाएाँ वह हैं जहााँ दो टे क्टोननक
वववव के अनेक भाग मोटे बेसाल्ट लावा प्रवाह से आच्छाहदत
प्लेट एक दस
ू रे से दरू जाती हैं। अपसारी सीमाएाँ भी दरार
हैं।
घाहटयााँ और नए महासागर बनाती हैं। पूवी अफ्रीका में दरार
• कथन 2 र्गलत है : समग्र ज्वालामुखी घाटी एक अलग सीमा का पररणाम है ।

समचित ज्वालामख
ु ी बेसाल्ट की तुलना में कूलर और अचिक
Q.56) Ans: a
चचपचचपे लावा के प्रकोप की ववशेर्ता है ।
Exp:

इनका ननमासण अनेक ववस्फोटों से हुआ है । • कथन 1 ही है : प्लेटें जस्थर नहीं होती हैं और क्षैनतज रूप से
अजस्थमंडल के ऊपर चलती हैं। ग्रह के आंतररक भाग के
बडी मात्रा में पाइरोक्लाजस्टक सामग्री और राख लावा के साथ भीतर रे डडयोिमी प्रकियाओं से ननकलने वाली ऊष्मा, प्लेटों
जमीन में समल जाती है । को कभी-कभी एक-दस
ू रे की ओर और कभी-कभी दरू ले जाने
का कारण बनती है ।
यह सामग्री वेंट के उद्घाटन के पास इकट्ठा होती है जजसके
पररणामस्वरूप परतों का ननमासण होता है । • कथन 2 र्गलत है : समद्र
ु तल लगातार मध्य महासागर की
लकीरों पर बन रहा है जहां टे क्टोननक प्लेट अलग हो रही हैं।
कफलीपींस में मेयोन ज्वालामख
ु ी, जापान में माउं ट फूजी और
यही कारण है कक महासागरीय िस्ट महाद्वीपीय की तुलना
वासशंगटन में माउं ट रे ननयर दनु नया के प्रमुख समचित
में बहुत छोटा है ।
ज्वालामख
ु ी हैं।

• कथन 3 ही है : मध्य महासागरीय कटक अपसारी प्लेट


प्रमुख संयुक्त ज्वालामुखी िख
ं ृ लाएं प्रशांत ररम हैं जजन्हें
सीमाओं के साथ होते हैं, जहां पथ्
ृ वी की टे क्टोननक प्लेट्स के
“अजग्न ररम” के रूप में जाना जाता है ।
अलग-अलग फैलते ही नए महासागरीय तल का ननमासण
ंदर्भ : 11वीं कक्षा ch – 03 र्ौततक र्ूर्गोल NCERT होता है । जैस-े जैसे प्लेटें अलग होती हैं, वपघली हुई चट्टानें
समुद्र तल की ओर उठती हैं, जजससे भारी ज्वालामख
ु ी
ववस्फोट होते हैं।
Q.55) Ans: b
Exp:
व्याख्या: Q.57) Ans: c
• जोडी 1 र्गलत है : रांसफॉमस बाउं ड्रीज वह जगह है जहां न तो Exp:
िस्ट बनता है और न ही नष्ट होता है क्योंकक प्लेट एक दस
ू रे

DPP 2023 DAY 48 44


Contact us : info@onlyias.com

OnlyIAS Nothing Else Visit : dpp.onlyias.in


Contact : +91-7007 931 912

• कथन 1 ही है : समद्र
ु तल के मानचचत्रण और • त्रबंद ु 4 ही है : वैगनर ने महाद्वीपों के बहाव के अन्य
महासागरीय क्षेत्रों से चट्टानों के पुराचुंबकीय अध्ययनों से कारणों के सलए ज्वारीय बल का भी सझ
ु ाव हदया। ज्वारीय
पता चला है कक सशखा से दरू जाने पर चट्टानों की आयु बढ़ बल चंद्रमा और सूयस के आकर्सण के कारण होता है जो समुद्र
जाती है । यह समुद्र तल के फैलने के प्रमुख प्रमाणों में से एक के पानी में ज्वार पैदा करता है।
है ।
• कथन 2 र्गलत है : एस्थेनोस्फीयर के ऊपर क्षैनतज रूप
• त्रबंद ु 5 ही है : पथ्
ृ वी के भीतर रे डडयोिमी िाराएं प्लेट
से घूमने वाली प्लेटें समुद्र तल के फैलने का संकेत नहीं
टे क्टोननक्स को भी प्रभाववत करती हैं। में टल के अंदर उठने
दे ती हैं।
और चगरने से कोर में रे डडयोिमी क्षय द्वारा उत्पन्न संवहन
• कथन 3 ही है : महाद्वीपीय तल की तल
ु ना में
िाराएं बनती हैं। संवहन िाराएाँ इस प्रकार प्लेटों को गनत
महासागर तल तलछट बहुत पतले होते हैं क्योंकक एक
प्रदान करती हैं।
ननरं तर नई महासागर परत उत्पन्न हो रही है , इससलए
तलछट का संचय महाद्वीपीय तल के मामले में नहीं
होता है । Q.59) Ans: d
Exp:
• कथन 4 ही है : महासागरीय कटक के सशखर पर
• कथन 1 ही है : महाद्वीप , प्लेट का हहस्सा होते हैं और
लगातार ववस्फोट होने से समद्र
ु ी िस्ट दोनों ओर
जो गनत करता है वह प्लेट हैं। कई महाद्वीप प्लेटों के
खखसक जाता है । सशखर पर ज्वालामुखी ववस्फोट के
बीच में होते हैं, उनकी सीमाओं या ककनारों पर नहीं।
कारण जो समद्र
ु तल िंस जाता है , वह समद्र
ु की खाइयों
प्लेट्स भी पथ्
ृ वी के महासागरों के नीचे जस्थत हैं। एक
में डूब जाता है और भस्म हो जाता है ।
प्लेट में अक्सर महाद्वीपीय और महासागरीय दोनों
क्षेत्र शासमल होते हैं। उदाहरण: उत्तरी अमेररकी प्लेट के
Q.58) Ans: a मूल में महाद्वीपीय िस्ट है और यह अचिकांश पक्षों से
Exp:
समद्र
ु ी िस्ट से नघरी हुई है ।
• त्रबंद ु 1 ही है : महाद्वीपों के बहाव के सलए अल्फ्रेड वेगेनर
• कथन 2 ही है : जैसे-जैसे महासागरीय िस्ट उथले
द्वारा सझ
ु ाए गए कारणों में से एक ध्रुव-भागने वाला बल था।
मध्य-महासागरीय कटक से दरू जाता है , यह ठं डा होता
ध्रुवीय-भागने वाला बल पथ्
ृ वी के घूणन
स से संबंचित है ।
जाता है और अचिक सघन होने पर डूब जाता है । इससे
महासागरीय बेससन का आयतन बढ़ जाता है और समुद्र
• प्वाइंट 2 र्गलत है : टे क्टोननक मूवमें ट के सलए प्लेसर का स्तर कम हो जाता है ।
डडपॉजजट्स का अजस्तत्व जजम्मेदार नहीं है । हालांकक, यह • कथन 3 ही है : समुद्र तल का फैलाव प्लेट
महाद्वीपीय बहाव ससद्िांत के समथसन के प्रमाणों में से एक वववतसननकी का ससफस एक हहस्सा है ।
है । सबडक्शन जोन में , सघन प्लेट का ककनारा
सबडक्ट्स, या स्लाइड्स, कम-घने वाले के
नीचे होता है । सघन सलथोस्फेररक पदाथस कफर
• त्रबंद ु 3 ही है : प्लेट वववतसननकी का मुख्य प्रेरक बल
वापस पथ्
ृ वी के मेंटल में वपघल जाता है ।
गरु
ु त्वाकर्सण है । यहद महासागरीय स्थलमंडल वाली एक
प्लेट दस
ू री प्लेट से समलती है , तो सघन महासागरीय
स्थलमंडल दस
ू री प्लेट के नीचे गोता लगाता है और में टल में Q.60) Ans: D
Exp:
डूब जाता है ।
• कथन 1 ही है : समद्र
ु तल प्रसार मध्य-महासागर
कटको के साथ होता है । यह सभी मध्य-

DPP 2023 DAY 48 45


Contact us : info@onlyias.com

OnlyIAS Nothing Else Visit : dpp.onlyias.in


Contact : +91-7007 931 912

महासागर कटको पर एक जैसा नहीं होता है । कथन 3 ही है : जब लावा ककसी मध्य-महासागर


िीरे -िीरे फैलने वाले कटक लंबी, संकरी पानी के कटक से ननकल रहा होता है , तो इससे बनने वाली
नीचे की चट्टानों और पहाडों के स्थल होते हैं। चट्टान चुम्बककत हो जाती है और लावा के ठं डा होने
तेजी से फैलने वाले कटको में बहुत अचिक के समय मौजूद चुंबकीय ध्रव
ु ता प्राप्त कर लेती है ।
कोमल ढलान होते हैं। उदाहरण- समड-अटलांहटक जैस-े जैसे िस्ट मध्य-महासागर के कटक से दरू जाता
कटक एक िीमी गनत से फैलने वाला केंद्र है । है , इसमें पथ्
ृ वी की बदलती चुंबकीय ध्रुवता का ननरं तर
ररकॉडस होता है ।

• कथन 2 ही है : समुद्र तल प्रसार में टल संवहन


का पररणाम है । में टल संवहन प्लेट वववतसननकी के Q.62) Ans: D
Exp:
सलए प्रेरक तंत्र है । में टल संवहन पथ्
ृ वी के ठोस
सससलकेट में टल की बहुत िीमी गनत से िीरे िीरे • कथन 1 ही है : प्लेट वववतसननक को चलाने वाले
होने वाली गनत है जो संवहन िाराओं के कारण बलों में : पथ्
ृ वी के भीतर मेंटल-हीट में संवहन दो
आंतररक से ग्रह की सतह तक ऊष्मा ले जाती है । मुख्य स्रोतों से आता है : रे डडयोिमी क्षय और
यह वह प्रकिया भी है जजसके द्वारा मेंटल में अवसशष्ट गमी, ररज पुश-फैली हुई लकीरों पर
चट्टानों की तरल जैसी गनतयों के माध्यम से गरु
ु त्वाकर्सण बल और सबडक्शन जोन में स्लैब
पथ्
ृ वी के गहरे आंतररक भाग में अनतररक्त ऊष्मा पुल-गुरुत्वाकर्सण बल शासमल है ।
को उसकी सतह पर स्थानांतररत ककया जाता है ।

• कथन 2 ही है : उथले-केंहद्रत भूकंप-आमतौर पर


Q.61) Ans: B 30 ककमी से कम गहरे ,मध्य-महासागर की लकीरों
Exp:
से जुडे हुए प्रतीत होते हैं। उथले फोकस भूकंप
कथन 1 र्गलत है : प्लेट की गनत की दर सवसत्र एक
आमतौर पर बडे पैमाने पर फैलते हैं, जजससे
समान नहीं होती है । ये दरें काफी सभन्न होती हैं। वे
सतह या पथ्
ृ वी की पपडी को अचिक नुकसान
प्रनत वर्स एक से दो इंच (तीन से पांच सेंटीमीटर) की
होता है ।
दर से आगे बढ़ते हैं।प्लेट के अलग-अलग हहस्से
अलग-अलग गनत से चलते हैं। प्लेटें अलग-अलग
हदशाओं में चलती हैं, टकराती हैं, दरू जाती हैं, और • कथन 3 ही है : महाद्वीपीय माजजसन महाद्वीपों
एक दस
ू रे के पीछे खखसकती हैं। के ननकट में पाया जाने वाला उथला जल क्षेत्र है ।
महाद्वीपीय माजजसन में तीन अलग-अलग
कथन 2 र्गलत है : समुद्र तल प्रसार अपसारी प्लेट
ववशेर्ताएं: महाद्वीपीय वद्
ृ चि, महाद्वीपीय ढलान
सीमाओं पर होता है । जैसे-जैसे टे क्टोननक प्लेट्स
और महाद्वीपीय शेल्फ होती हैं। महाद्वीपीय
िीरे -िीरे एक-दस
ू रे से दरू जाती हैं, में टल की संवहन माजजसन महासागरीय क्षेत्र का लगभग 28% होता
िाराओं से ननकलने वाली ऊष्मा िस्ट को अचिक
है ।
प्लाजस्टक और कम घना बना दे ती है ।कम-घने सामग्री
ऊपर उठती है , अक्सर समद्र
ु तल के पहाड या ऊंचे
Q.63) Ans: D
क्षेत्र का ननमासण करती है । Exp:
• कथन 1 र्गलत है : प्रमुख ज्वालामुखी आमतौर पर
रांसफॉमस सीमाओं पर नहीं होते हैं। रांसफॉमस प्लेट

DPP 2023 DAY 48 46


Contact us : info@onlyias.com

OnlyIAS Nothing Else Visit : dpp.onlyias.in


Contact : +91-7007 931 912

सीमा पर प्लेटों के बीच पीसने की किया के चलती रहती हैं, इससलए बनने वाला ज्वालामख
ु ी
पररणामस्वरूप दल
ु भ
स उथले भूकंप, चट्टान के बडे टे क्टोननक प्लेट के साथ उस हदशा में "गनतमान"
पाववस ववस्थापन, और िस्टल ववरूपण का एक होगा जहां टे क्टोननक प्लेट जा रही है , लेककन
व्यापक क्षेत्र होता है ।. साथ ही साथ हॉटस्पॉट लावा का उत्पादन बंद नहीं
करता है । यह प्लेटों की गनत की रे ककं ग सक्षम
• कथन 2 ही है : महाद्वीप-महाद्वीप असभसरण बनाता है ।
तब होता है जब दो महाद्वीपीय प्लेटें टकराती हैं।
जब महाद्वीपीय िस्ट की दो प्लेटें टकराती हैं, तो
Q.65) Ans: B
सामग्री ऊपर की ओर उठती है । यह एक उच्च
Exp:
पवसत िंख
ृ ला बनाती है । उदाहरण: वसलत पवसतों • कथन 1 ही है : समड-अटलांहटक कटक आइसलैंड
का ननमासण। से होकर गज
ु रती है जहां कटक को ननयोवोल्केननक
जोन के रूप में भी जाना जाता है ।.
• कथन 3 र्गलत है : महासागर-महासागर असभसरण
में , दो महासागरीय प्लेटें आपस में समलती हैं या
टकराती हैं। एक सघन महासागरीय प्लेट कम • कथन 2 ही है : जापान एक ववनाशकारी प्लेट
घनी समद्र
ु ी प्लेट के नीचे आ जाती है जजससे माजजसन के पास जस्थत है , जहां प्रशांत प्लेट को
सीमा के साथ एक खाई बन जाती है ।सबडक्शन कफलीपीन प्लेट के नीचे िकेला जा रहा है ।
़ोन के ऊपर लगातार ज्वालामुखी चट्टानों की "पैससकफक ररंग ऑफ फायर" के साथ दे श की
परतें बनाता है । एक अन्य महासागरीय िस्ट के जस्थनत के कारण जहााँ यह प्रशांत महासागर के
नीचे एक महासागर प्लेट के सबडक्शन के नीचे प्रशांत प्लेट और कफलीपीन सागर प्लेट सहहत
पररणामस्वरूप एक ज्वालामख
ु ी द्वीप चाप, एक तीन टे क्टोननक प्लेटों में जस्थत है , जापान और
महासागरीय खाई और कई भूकंप बनते हैं। भूकंप साथ-साथ चलते हैं।

Q.64) Ans: C • कथन 3 र्गलत है : पव


ू ी अफ्रीकी दरार घाटी पव
ू ी
Exp:
अफ्रीका में एक ववकासशील अपसारी प्लेट सीमा
• कथन 1 र्गलत है : हॉट स्पॉट प्लेट की सीमाओं से
है । यहााँ अफ्रीका का पूवी भाग, सोमासलयाई प्लेट,
स्वतंत्र होता है । इसके बजाय यह में टल प्लम्स
शेर् महाद्वीप से दरू जा रहा है , जजसमें न्यब्रु बयन
पर होता है । में टल प्लम पथ्ृ वी की पपडी के नीचे
प्लेट शासमल है ।
का एक क्षेत्र होता है , जहां मैग्मा आसपास के
मैग्मा की तुलना में अचिक गमस होता है ।इस
अनतररक्त गमस मैग्मा से ननकलने वाली गमी Q.66) Ans: C
Exp:
चट्टानी पपडी के वपघलने और पतले होने का
• कथन 1 सही है : खननज ननक्षेपों के ववतरण को
कारण बनती है , जजससे पथ्
ृ वी की सतह पर प्लम
समझने के सलए वववतसननक प्रकियाओं और क्षेत्रीय
के ऊपर व्यापक ज्वालामख
ु ी गनतववचि होती है ।
भूववज्ञान का अध्ययन महत्वपूणस है क्योंकक
खननज ननक्षेप ववसभन्न वववतसननक और भूगसभसक
• कथन 2 ही है : हॉटस्पॉट कभी भी अपनी जस्थनत सेहटंग्स में होते हैं। उदाहरण- सोने कोगहरी
नहीं बदलता है , लेककन टे क्टोननक प्लेट लगातार भसू मगत चट्टानो की दरारों की सशराओं में अन्य

DPP 2023 DAY 48 47


Contact us : info@onlyias.com

OnlyIAS Nothing Else Visit : dpp.onlyias.in


Contact : +91-7007 931 912

खननजों के साथ सांहद्रत ककया जा सकता है । का संपण


ू स आिार यह है कक चट्टानें समय के
टे क्टोननक बल पवसत िख
ं ृ लाओं का ननमासण करते साथ लगातार बदल रही हैं.
हुए इन चट्टानों को ऊपर उठाते हैं जहां अपक्षय
और क्षरण पथ्
ृ वी की सतह पर सशराओं में जमा
• कथन 3 ही है : रॉक साइककल प्रकिया तापमान,
सोने को प्रकट करते हैं।
दाब, समय और पथ्
ृ वी की पपडी और इसकी
सतह पर पयासवरणीय पररजस्थनतयों में पररवतसन
• कथन 2 र्गलत है : उत्तर पव
ू ी प्रायद्वीपीय बेल्ट पर ननभसर है ।
भारत की सबसे समद्
ृ ि खननज बेल्ट है । इस
बेल्ट में झारखंड, पजवचम बंगाल और उडीसा Q.68) Ans: C
राज्यों में छोटा नागपुर पठार और उडीसा पठार Exp:
शासमल हैं। इसमें बडी मात्रा में कोयला, लौह • कथन 1 र्गलत है : प्लूटोननक चट्टानें आग्नेय
अयस्क, मैंगनीज, अभ्रक, बॉक्साइट, तांबा आहद चट्टानें हैं जो जम जाती हैं और पथ्
ृ वी की पपडी
होता है । में गहराई पर बनती हैं। ऐसी चट्टानों में खननज
दाने बडे होते हैं क्योंकक वे ठं डे और िीरे -िीरे
जमते हैं।
• कथन 3 र्गलत है : हे मेटाइट लगभग 70 प्रनतशत
िातु सामग्री के साथ सबसे अच्छा लौह अयस्क
है । यह प्रायद्वीपीय भारत के िारवाड और कडप्पा • कथन 2 ही है : प्रमुख पवसत िंख
ृ लाओं के
चटटानी तंत्र में पाया जाता है । लगभग 80 केंद्रीय कोर में अन्तवेिी आग्नेय चट्टानें होती
प्रनतशत हे मेटाइट भंडार ओडडशा, झारखंड, हैं। हालांकक, अचिकांश वसलत पवसत मख्
ु य रूप से
छत्तीसगढ़ और आंध्र प्रदे श में हैं। उच्च दाब और अपेक्षाकृत कम तापमान के तहत
मेटामॉकफस क और अवसादी चट्टानो से बने होते
हैं।.
Q.67) Ans: D
Exp:
• कथन 1 ही है : एक चचत्तं चि तीन मुख्य रॉक
• कथन 3 ही है : आग्नेय चट्टानें या तो
प्रकारों: तलछटी, कायापलट और आग्नेय के बीच
अन्तवेिी (प्लट
ू ोननक) या बहहमख
ुस ी (ज्वालामख
ु ी)
भूगभीय समय के माध्यम से संिमण का वणसन
हो सकती हैं। अन्तवेिी आग्नेय चट्टानें मैग्मा
करता है । रॉक चि की प्रमुख प्रकियाएं
से बनती हैं जो ककसी ग्रह की पपडी के भीतर
किस्टलीकरण, क्षरण और अवसादन, और
ठं डा होकर जम जाता हैं। जबकक, बहहमख
ुस ी
कायापलट हैं।
आग्नेय चट्टानें पथ्ृ वी की सतह पर वपघले हुए
मैग्मा के ठं डा होने से बनती हैं।
• कथन 2 ही है : आग्नेय चट्टानें प्राथसमक
चट्टानें हैं और इन प्राथसमक चट्टानों से अवसादी Q.69) Ans: B
और कायांतररत रूप हैं। आग्नेय चट्टानों को Exp:
कायांतररत चट्टानों में बदला जा सकता • कथन 1 र्गलत है: राष्रीय बाल अचिकार
है ।आग्नेय और कायांतररत चट्टानों से ननकले संरक्षण आयोग (एनसीपीसीआर) महहला
टुकडे अवसादी चट्टानों में बनते हैं। चट्टान चि
DPP 2023 DAY 48 48
Contact us : info@onlyias.com

OnlyIAS Nothing Else Visit : dpp.onlyias.in


Contact : +91-7007 931 912

एवं बाल ववकास मंत्रालय, भारत सरकार o अपरािी के खखलाफ मुकदमा


के प्रशासननक ननयंत्रण के अिीन एक चलाने के सलए संबंचित सरकार से
वैिाननक ननकाय है। एनसीपीसीआर की संपकस
स्थापना माचस 2007 में संसद के एक o कानून के उल्लंघन की सशकायतों
अचिननयम (हदसंबर 2005), बाल अचिकार की जांच
संरक्षण आयोग (सीपीसीआर) अचिननयम, o एक व्यजक्त को बुलाना और सबूत
2005 के तहत की गई थी। आयोग 5 मांगना
माचस 2007 से चालू है। o मजजजस्रयल जांच की मांग
• कथन 2 र्गलत है : आयोग का जनादे श यह o उच्च न्यायालय या सवोच्च
सुननजवचत करना है कक सभी कानून, न्यायालय में ररट याचचका दायर
नीनतयां, कायसिम और प्रशासननक तंत्र करना
भारत के संवविान और बाल अचिकारों पर o प्रभाववत लोगों को अंतररम राहत
संयुक्त राष्र कन्वें शन में ननहहत बाल की ससफाररश
अचिकार पररप्रेक्ष्य के अनुरूप हैं। सन्दभस:
सीपीसीआर अचिननयम के तहत, बाल https://www.pib.gov.in/PressReleasePage.aspx?P
RID=1802100
पररभार्ा में 0 से 18 वर्स आयु वगस के
व्यजक्त को शासमल करता हैं।
Q.70) Ans: A
• एन ीपी ीआर की ंरचना: Exp:
o इस आयोग की संरचना में एक • कथन 1 ही है: अनद
ु ान के सलए
अध्यक्ष और छह सदस्य शासमल अनप
ु रू क मांगों की आववयकता तब होती
हैं। है जब संसद द्वारा ववननयोग अचिननयम
o जजनमें से कम से कम दो के माध्यम से चालू ववत्तीय वर्स के सलए
महहलाएं होनी चाहहए। ककसी ववशेर् सेवा के सलए अचिकृत रासश
o इनकी ननयुजक्त केंद्र सरकार द्वारा उस वर्स के सलए अपयासप्त पाई जाती
तीन साल के सलए की जाती है। है। अनुच्छे द 115 अनुपूरक, अततररक्त या
o आयोग में सेवा करने की अधधक अनुदान े ंबंधधत है ।
अचिकतम आयु अध्यक्ष के सलए • कथन 2 र्गलत है : ये अनुदान ववत्तीय वर्भ
65 वर्स और सदस्यों के सलए 60 की माष्क्प्त े पहले ं द द्वारा प्रस्तुत
वर्स है । और पाररत ककए जाते हैं। ववत्त मंत्रालय
• कथन 3 ही है: आरटीई अचिननयम, अनतररक्त अनुदान की मांग प्रस्तुत करता
2009 के तहत, एनसीपीसीआर है, जब वास्तववक व्यय संसद के
ननम्नसलखखत कायस कर सकता है : अनुमोहदत अनुदान से अचिक हो जाता है ।
• कथन 3 ही है : भारत के ननयंत्रक-
महालेखापरीक्षक संसद के ध्यान में अचिक

DPP 2023 DAY 48 49


Contact us : info@onlyias.com

OnlyIAS Nothing Else Visit : dpp.onlyias.in


Contact : +91-7007 931 912

अनुदान की मांग लाते हैं। इन ज्यादनतयों • अ ाधारण अनुदान: इसका उपयोग


और संसद को दी गई ससफाररशों की जांच असािारण उद्दे वयों के सलए ककया जाता
लोक लेखा ससमनत द्वारा की जाती है । है जो ककसी भी ववत्तीय वर्स की वतसमान
अनतररक्त अनुदान की मांग वास्तववक सेवा का हहस्सा नहीं है ।
व्यय के बाद की जाती है और उस ववत्तीय
सन्दभस:
वर्स की समाजप्त के बाद संसद में प्रस्तुत
https://www.thehindu.com/news/national/parlia
की जाती है जजसमें खचस ककया गया था। ment-proceedings-lok-sabha-passes-
supplementary-demand-for-
Onlyiasद्वा grants/article34101248.ece
रा अततररक्त ज्ञान
• अततररक्त अनद
ु ान: जब चालू ववत्तीय Q.71) Ans: B
वर्स के दौरान ककसी नई सेवा पर Exp:
अनप
ु रू क या अनतररक्त व्यय की • मौसलक अचिकार कुछ ऐसे दावे हैं जो

आववयकता उत्पन्न हो, जजस पर उस ककसी व्यजक्त के सवाांगीण ववकास के सलए

वर्स के बजट में ववचार नहीं ककया गया मौसलक हैं। इन अचिकारों को राज्य द्वारा

हो, तो अनतररक्त अनुदान हदया जा मान्यता प्राप्त है और संवविान द्वारा ही

सकता है . इसकी गारं टी दी जाती है । इन दावों को

• ज्यादती अनुदान: जब ककसी ववत्तीय वर्स मौसलक अचिकार (FRs) कहा जाता है।

के दौरान ककसी सेवा पर उस वर्स के मौसलक अचिकार राजनीनतक लोकतंत्र के

सलए दी गई रासश से अचिक रासश खचस आदशस को बढ़ावा दे कर राज्य के मनमाने

की जाती है, तो ज्यादती अनुदान हदया शासन को रोकते हैं,

जा सकता है। ज्यादती अनुदान की • शत्रु ववदे सशयों को छोडकर नागररकों और

मांग वास्तव में खचस होने और जजस ववदे सशयों दोनों के सलए उपलब्ि मौसलक

ववत्तीय वर्स से वह संबंचित है, उसके अचिकार

समाप्त हो जाने के बाद की जाती है। o अनुच्छे द 14 - कानून के समक्ष समानता

• ांकेततक अनुदान: यह तब प्रदान ककया और कानूनों का समान संरक्षण।

जाता है जब ककसी नई सेवा पर o अनुच्छे द 20 - अपरािों के सलए

प्रस्ताववत व्यय को पूरा करने के सलए दोर्ससद्चि के संबंि में संरक्षण से संबंचित

िन पुनववसननयोजन द्वारा उपलब्ि है।

कराया जा सकता है, सांकेनतक रासश के o अनुच्छे द 21 - जीवन और व्यजक्तगत

अनुदान की मांग सदन के वोट के सलए स्वतंत्रता के संरक्षण का प्रतीक है।

प्रस्तुत की जा सकती है और, यहद o अनुच्छे द 21ए - प्रारं सभक सशक्षा का

सदन की सहमनत हो मांग के सलए, अचिकार।

िन उपलब्ि कराया जा सकता है । o अनुच्छे द 22 - कुछ मामलों में चगरफ्तारी


और नजरबंदी के खखलाफ संरक्षण।
DPP 2023 DAY 48 50
Contact us : info@onlyias.com

OnlyIAS Nothing Else Visit : dpp.onlyias.in


Contact : +91-7007 931 912

o अनुच्छे द 23 - मानव के दव्ु यासपार और • कथन 1 र्गलत है : 1973 की आपराचिक प्रकिया संहहता
बलात ् िम का प्रनतर्ेि। (सीआरपीसी) की िारा 144 आम तौर पर सावसजननक सभा

o अनुच्छे द 24 - कारखानों आहद में बच्चों को प्रनतबंचित करती है । यह ककसी भी राज्य या क्षेत्र के
कायसकारी मजजस्रे ट को एक क्षेत्र में चार या अचिक लोगों के
के ननयोजन का प्रनतर्ेि।
इकट्ठा होने पर रोक लगाने का आदे श जारी करने के सलए
o अनुच्छे द 25 - अंतःकरण की स्वतंत्रता
अचिकृत करता है ।
और िमस के स्वतंत्र व्यवसाय, आचरण
• कानून के अनुसार, ऐसी 'गैरकानूनी सभा' का हर सदस्य
और प्रचार-प्रसार की स्वतंत्रता।
दं गा करने के सलए संहदग्ि हो सकता है । धारा 144 के
o अनुच्छे द 26 - िासमसक मामलों के प्रबंिन उल्लंघन पर अधधकतम जा तीन ाल है । इ के अलावा,
की स्वतंत्रता पुसलस को गैरकानूनी सभा को हटाने से रोकना एक दं डनीय
o अनुच्छे द 27 - ककसी भी िमस के प्रचार के अपराि भी है । िारा 144 उस क्षेत्र में ककसी भी प्रकार के
सलए करों के भुगतान से मुजक्त। हचथयार ले जाने पर भी प्रनतबंि लगाती है जहां िारा 144
o अनुच्छे द 28 - कुछ शैक्षखणक संस्थानों में लागू की गई है और लोगों को इसका उल्लंघन करने के सलए
िासमसक सशक्षा या पूजा में भाग लेने की हहरासत में सलया जा सकता है।

स्वतंत्रता। • कथन 2 ही है : िारा 144 उपद्रव के तत्काल मामलों में या


ककसी घटना के संभाववत खतरे में लगाई जाती है जजसमें
• कुछ मौसलक अचिकार हैं जो केवल भारत
मानव जीवन या संपवत्त को परे शानी या क्षनत होने की
के नागररकों के सलए उपलब्ि हैं::
संभावना होती है । िारा 144 के तहत कोई भी आदे श दो
o अनच्
ु छे द 15: िमस, मल
ू वंश, जानत,
महीने से अचिक समय तक लागू नहीं रहे गा लेककन राज्य
सलंग या जन्म स्थान के आिार पर
सरकार वैिता को दो महीने और अचिकतम छह महीने तक
भेदभाव के खखलाफ अचिकार; बढ़ा सकती है । जस्थनत सामान्य होने पर इसे ककसी भी समय
o अनच्
ु छे द 16: सावसजननक रोजगार वापस सलया जा सकता है ।
के मामले में अवसर की समानता • िारा 144 का पहली बार 1861 में ब्रिहटश राज द्वारा
का अचिकार; इस्तेमाल ककया गया था, और उसके बाद स्वतंत्रता संग्राम के
o अनुच्छे द 19: वाक् और दौरान सभी राष्रवादी ववरोिों को रोकने के सलए एक
असभव्यजक्त, सभा, संघ, आंदोलन, महत्वपूणस उपकरण बन गया। हालााँकक, स्वतंत्र भारत में

ननवास और पेशे की स्वतंत्रता; िारा का उपयोग वववादास्पद बना हुआ है क्योंकक इसमें बहुत
कम पररवतसन हुआ है ।
o अनुच्छे द 29 और 30: सांस्कृनतक
• कथन 3 ही है : िारा 144 आमतौर पर प्रकृनत में
और शैक्षक्षक अचिकार
ननर्ेिात्मक है । यह सावसजननक सभा से प्रनतबंचित करता है
सन्दभस:
लेककन यह सब एक साथ नहीं रोकता है । दस
ू री ओर, एक
https://www.thehindu.com/news/national/gover
nment-seeks-law-on-rights-of-foreigners-who- कफ्यूस लोगों को एक ननजवचत अवचि के सलए घर के अंदर
violate-visa-conditions/article65224111.ece रहने का आदे श दे ता है । इससलए, अचिकारी एक ननजवचत
अवचि के सलए कफ्यूस लगा सकते हैं (हालांकक, जरूरत पडने
Q.72) Ans: B पर अचिकारी कफ्यूस को बढ़ा भी सकते हैं)। कफ्यूस के दौरान
Exp: बाहर ननकलने के सलए स्थानीय पुसलस की पूवासनुमनत की भी
आववयकता होती है ।Reference:

DPP 2023 DAY 48 51


Contact us : info@onlyias.com

OnlyIAS Nothing Else Visit : dpp.onlyias.in


Contact : +91-7007 931 912

https://indianexpress.com/article/explained/controve राष्रपनत के रूप में अनस


ु चू चत जनजानतयों के संरक्षण,
rsy-over-shivaji-statue-telangana-bodhan-town-
7829183/ कल्याण और ववकास और उन्ननत के संबि
ं में ऐसे अन्य
कायों का ननवसहन करते हैं, जो ककसी भी ननहदसष्ट ननयम
द्वारा संसद द्वारा बनाया गया कानून प्राविानों के अिीन
Q.73) Ans: C
Exp: हो सकते हैं। ।
• कथन 1 ही है : राष्रीय अनुसूचचत जनजानत आयोग एक Reference:
https://www.thehindu.com/news/national/national-
संवैिाननक ननकाय है जजसे 19 फरवरी 2004 से अनुच्छे द commission-for-scheduled-tribes-is-dysfunctional-
338 में संशोिन करके और 89वें संवविान संशोिन house-panel/article65238335.ece
अचिननयम, 2003 के माध्यम से संवविान में एक नया
अनुच्छे द 338A सजम्मसलत करके स्थावपत ककया गया था। Q.74) Ans: C
अनुच्छे द 338A अन्य बातों के साथ-साथ NCST को Exp:
संवविान के तहत या ककसी अन्य कानून के तहत या सरकार • कथन 1 ही है : उचचत आवास' एक ससद्िांत है जो

को ककसी अन्य आदे श के तहत st को प्रदान ककए गए समानता को बढ़ावा दे ता है , सकारात्मक अचिकार प्रदान

ववसभन्न सुरक्षा उपायों के कायासन्वयन की ननगरानी करने करने में सक्षम बनाता है और ववकलांगता, स्वास्थ्य की

और ऐसे सुरक्षा उपायों के कामकाज का मूल्यांकन करने की जस्थनत या व्यजक्तगत ववववास के आिार पर भेदभाव को

शजक्त दे ता है । रोकता है ।

• कथन 2 र्गलत है : संवविान NCST की संरचना को • इसका उपयोग मुख्य रूप से ववकलांगता अचिकार क्षेत्र में

ननिासररत करता है , जजसमें एक अध्यक्ष, एक उपाध्यक्ष और होता है । यह ववकलांग व्यजक्तयों को समाज में उनकी पण
ू स

3 अन्य सदस्य शासमल होते हैं जजन्हें राष्रपनत द्वारा उनके और प्रभावी भागीदारी को सवु विाजनक बनाने के सलए

हस्ताक्षर और मुहर के तहत वारं ट द्वारा ननयुक्त ककया अनतररक्त सहायता प्रदान करने के सलए राज्य और ननजी

जाता है । पक्षों के सकारात्मक दानयत्व को दशासता है ।

0 कम से कम एक सदस्य एक महहला होनी चाहहए। • कथन 2 र्गलत है : अंतरासष्रीय िम संगठन 2016 में

o अध्यक्ष, उपाध्यक्ष और अन्य सदस्य 3 वर्स की अवचि के कायसस्थल समायोजन के माध्यम से वववविता और समावेश

सलए पद िारण करते हैं। को बढ़ावा दे ने के सलए एक व्यावहाररक मागसदसशसका लेकर

0 सदस्य दो से अचिक कायसकाल के सलए ननयजु क्त के पात्र आया था। कायसस्थल आवास की आववयकता ववसभन्न

नहीं हैं। जस्थनतयों में उत्पन्न हो सकती है , लेककन गाइड के सलए

o अध्यक्ष को केंद्रीय कैब्रबनेट मंब्रत्रयों का पद हदया गया है , िसमकों की चार श्ेणणयों को चन


ु ा र्गया था:

उपाध्यक्ष को एक का पद हदया गया है o ववकलांग िसमक,

o राज्य मंत्री और अन्य सदस्यों को भारत सरकार के सचचव o एचआईवी और एड्स से ग्रससत िसमक,

का दजास प्राप्त है । o गभसवती कामगार और पाररवाररक जजम्मेदाररयां, और

• कथन 3 ही है : राष्रीय अनुसूचचत जनजानत आयोग o कायसकतास जो एक ववशेर् ववववास या िमस को मानते हैं।

द्वारा ककए गए कतसव्यों और कायों में से एक राष्रपनत के • इन िेखणयों के कामगारों को काम पर ववसभन्न प्रकार की

पास वावर्सक रूप से उपजस्थत होता है और ऐसे अन्य समय बािाओं का सामना करना पडता है । इनके पररणामस्वरूप

पर जैसा आयोग उचचत समझे, उन सुरक्षा उपायों के कायस की या तो रोजगार का नुकसान हो सकता है या रोजगार तक

ररपोटस दे ता है । आयोग जैसे अन्य कायस अनुसूचचत पहुंच में कमी हो सकती है । उचचत आवास का प्राविान इन

जनजानतयों के अचिकारों और सुरक्षा उपायों से वंचचत होने बािाओं को दरू करने में एक प्रमुख भूसमका ननभाता है और

के संबंि में ववसशष्ट सशकायतों की जांच करते हैं और

DPP 2023 DAY 48 52


Contact us : info@onlyias.com

OnlyIAS Nothing Else Visit : dpp.onlyias.in


Contact : +91-7007 931 912

इस प्रकार अचिक से अचिक कायसस्थल समानता, वववविता वाला व्यजक्त ववदे शी है या नहीं। इससे पहले,
और समावेश में योगदान दे ता है । न्यायाचिकरणों के गठन की शजक्तयााँ केवल केंद्र के पास थीं।
• कथन 3 ही है : भारत में , ववकलांग लोगों के अचिकार • कथन 3 र्गलत है : संशोचित आदे श (ववदे शी (हरब्यूनल)
अचिननयम, 2016 में 'उचचत आवास' को "आववयक और आदे श, 2019) व्यजक्तयों को हरब्यूनल से संपकस करने का
उपयुक्त संशोिन और समायोजन, ककसी ववशेर् मामले में अचिकार भी दे ता है । इससे पहले, केवल राज्य प्रशासन ही
ब्रबना अनुपानतक या अनचु चत बोझ के, व्यजक्तयों को ककसी संहदग्ि के खखलाफ हरब्यूनल में जा सकता था। एक
सुननजवचत करने के सलए" के रूप में पररभावर्त ककया गया घोवर्त ववदे शी, या डीएफ, एक ऐसा व्यजक्त है जजसे फॉरे नसस
है । दस
ू रों के साथ समान रूप से अचिकारों के आनंद या हरब्यूनल (एफटी) द्वारा कचथत तौर पर अपनी नागररकता
प्रयोग को अक्षम करता है "। िारा 2 (H) 'भेदभाव' को 'उचचत साब्रबत करने में ववफल रहने के सलए राज्य पसु लस के बॉडसर
आवास से इनकार' के रूप में पररभावर्त करती है । ववंग द्वारा उसे एक अवैि अप्रवासी के रूप में चचजननत करने
• ववकाश कुमार बनाम यूपीए ी (2021): सवोच्च के सलए चचजननत ककया गया है ।
न्यायालय ने माना कक बेंचमाकस ववकलांगता, जो कक 40 ं र्भ:

प्रनतशत के कोटे के सलए एक ननहदस ष्ट ववकलांगता है , केवल https://www.thehindu.com/news/national/other-
states/assam-decides-tribunal-members-term-on-
ववकलांगों के सलए रोजगार में ववशेर् आरक्षण से संबंचित है , rate-of-declaring-foreigners-
लेककन यह ककसी अन्य प्रकार के आवास के सलए प्रनतबंि amnesty/article31051919.ece
नहीं होना चाहहए।
Reference:
Q.76) Ans: D
https://www.thehindu.com/news/national/explained-
Exp:
did-the-karnataka-high-courts-hijab-verdict-overlook-
reasonable-accommodation/article65240144.ece • हाल ही में , राष्रीय महहला आयोग (NCW) ने हदल्ली राज्य
कानन
ू ी सेवा प्राचिकरण (DSLSA) के सहयोग से एक कानन
ू ी
सहायता जक्लननक शुरू ककया है । यह महहलाओं को मफ्
ु त
Q.75) Ans: A
Exp: कानूनी सहायता दे कर उनकी सशकायतों के समािान के सलए
• कथन 1 ही है : फॉरे नसस हरब्यूनल अिस-न्यानयक ननकाय एकल खखडकी सुवविा के रूप में कायस करे गा।
हैं जजन्हें फॉरे नसस हरब्यूनल ऑडसर, 1964 और फॉरे नसस एक्ट, • कानूनी सहायता जक्लननक नई हदल्ली में एनसीडब्ल्यू
1946 के अनस
ु ार स्थावपत ककया गया है । फॉरे नसस हरब्यन
ू ल कायासलय से संचासलत होगा। जक्लननक के तहत,
की संरचना में 35 वर्स से कम आयु के अचिवक्ता जजन्हें 7 सशकायतकतासओ,ं संकट में महहलाओं, राष्रीय कानूनी सेवा
साल का अनुभव या अब प्रैजक्टस (या) असम न्यानयक सेवा प्राचिकरण (NALSA) की ववसभन्न योजनाओं पर सलाह और
से सेवाननवत्त
ृ न्यानयक अचिकारी (या) एसीएस अचिकाररयों जानकारी, मफ्
ु त कानूनी सहायता, वैवाहहक मामलों में
के सेवाननवत्त
ृ आईएएस (सचचव / अनतररक्त सचचव के पद से सुनवाई और आयोग के साथ पंजीकृत अन्य सशकायतों के
नीचे नहीं) जजन्हें अिस-न्यानयक कायों में अनुभव न हो सलए परामशस प्रदान ककया जाएगा।
शासमल नहीं हैं। । राष्ट्रीय मदहला आयोर्ग:
• कथन 2 ही है : गह
ृ मंत्रालय (एमएचए) ने ववदे शी • इसे राष्रीय महहला आयोग अचिननयम, 1990 के तहत
(हरब्यन
ू ल) आदे श, 1964 में संशोिन ककया है , और सभी जनवरी 1992 में एक वैिाननक ननकाय के रूप में स्थावपत
राज्यों और केंद्र शाससत प्रदे शों में जजला मजजस्रे टों को यह ककया गया था। इसका समशन महहलाओं को उनके उचचत
तय करने के सलए हरब्यूनल (अिस-न्यानयक ननकाय) स्थावपत अचिकारों और अचिकारों को हाससल करने के सलए उपयुक्त
करने का अचिकार हदया है । भारत में अवैि रूप से रहने अचिकारों और अचिकारों को हाससल करने के सलए जीवन के
सभी क्षेत्रों में समानता और समान भागीदारी प्राप्त करने के

DPP 2023 DAY 48 53


Contact us : info@onlyias.com

OnlyIAS Nothing Else Visit : dpp.onlyias.in


Contact : +91-7007 931 912

सलए नीनत ननमासण, वविायी उपाय, आहद बनाने की हदशा में Q.78) Ans: C
Exp:
प्रयास करना है । ।
• राष्ट्रीय मदहला आयोर्ग के कायभ • संसदीय कायस मंत्रालय (MoPA) ने दे श में सभी वविान
o महहलाओं के सलए संवैिाननक और कानूनी सुरक्षा उपायों सभाओं के कामकाज को कागज रहहत बनाने के सलए
की समीक्षा करें । डडजजटल वविानमंडलों के सलए एक समशन मोड प्रोजेक्ट
0 उपचारात्मक वविायी उपायों की ससफाररश करना। "नेशनल ई-वविान एजप्लकेशन (नेवा)" ववकससत ककया है ।
o सशकायतों के ननवारण को सुगम बनाना। राष्रीय ई-वविान एजप्लकेशन (नेवा) डडजजटल इंडडया
0 महहलाओं को प्रभाववत करने वाले सभी नीनतगत मामलों कायसिम में शासमल एक समशन मोड प्रोजेक्ट (एमएमपी) है ।
पर सरकार को सलाह दे ना। 'एक राष्र-एक आवेदन' की थीम पर दे श के सभी वविान
ं र्भ: यूपीएससी 2015
द सभाओं के कामकाज को कागज रहहत बनाना।
https://pib.gov.in/PressReleaseIframePage.aspx?PRID • सभी राज्य वविानसभाओं को 'डडजजटल सदनों' में बदलना
=1810921
ताकक वे राज्य सरकार के ववभागों के साथ डडजजटल मोड में
सच
ू ना के आदान-प्रदान सहहत डडजजटल प्लेटफॉमस पर परू े
Q.77) Ans: B
Exp: सरकारी कायस संचालन कर सकें। आवेदन ने संसद के दोनों
• कथन 1 र्गलत है : राष्रीय जांच एजेंसी (एनआईए) के सदनों में शासमल होने के प्राविानों को भी सक्षम ककया है ।
ववपरीत, सीबीआई ककसी राज्य में ककसी मामले का स्वत: • ई-वविान के सलए ववत्त पोर्ण एमओपीए द्वारा प्रदान
संज्ञान नहीं ले सकती है - चाहे वह भ्रष्टाचार के मामले में हो ककया जाता है । यह केंद्रीय प्रायोजजत योजना की तजस पर है ,
जजसमें केंद्र के सरकारी अचिकारी और पीएसयू कमसचारी यानी राज्यों के सलए 60:40; उत्तर पव
ू स और पहाडी राज्यों के
शासमल हों या हहंसक अपराि की घटना हो। केंद्र सरकार के सलए 90:10 और केंद्र शाससत प्रदे शों के सलए 100%। 2021
कमसचाररयों से जड
ु े भ्रष्टाचार के मामलों को लेने के सलए, इसे में , ब्रबहार वविान पररर्द दे श का पहला सदन बन गया,
या तो राज्य सरकार की सामान्य सहमनत या केस-टू-केस जजसने पूरी तरह से NeVA प्लेटफॉमस पर रांजजट ककया और
आिार पर ववसशष्ट सहमनत की आववयकता होती है । पेपरलेस मोड में NeVA प्लेटफॉमस पर शीतकालीन सत्र, 2021
• अन्य सभी मामलों के सलए, चाहे राज्य सरकार में का आयोजन ककया।
Reference:https://indianexpress.com/article/cities/jai
भ्रष्टाचार हो या अपराि की घटना, राज्य को सीबीआई से
pur/tension-between-rajasthan-governor-university-
जांच का अनुरोि करना होगा, और केंद्र को इसके सलए vc-building-up-since-last-year-7806689/
सहमत होना होगा। यहद राज्य ऐसा अनुरोि नहीं करता है ,
तो सीबीआई संबंचित उच्च न्यायालय या सवोच्च Q.79) Ans: D
न्यायालय के आदे श के आिार पर मामले को अपने हाथ में Exp:
ले सकती है । • कथन 1 ही है : र्ारतीय संवविान के अनुच्छे द 153 से
• कथन 2 ही है : केंद्रीय अन्वेर्ण ब्यूरो (सीबीआई) भारत अनुच्छे द 162 के तहत राज्यपाल की ननयजु क्त, उनकी
की प्रमख
ु जांच पसु लस एजेंसी है । यह कासमसक, कासमसक, शजक्तयों और राज्यपाल के कायासलय से संबंचित हर चीज
पें शन और लोक सशकायत मंत्रालय, भारत सरकार - जो पर चचास की गई है । राज्यपाल राज्य का संवैिाननक प्रमख

प्रिान मंत्री कायासलय के अंतगसत आता है ववभाग के अिीक्षण होता है , जो अपने मंब्रत्रपररर्द की सलाह से बाध्य होता है ।
में कायस करता है । । यह भारत में नोडल पुसलस एजेंसी भी है वह केंद्र सरकार और राज्य सरकार के बीच एक महत्वपूणस
जो इंटरपोल सदस्य दे शों की ओर से जांच का समन्वय करती कडी के रूप में कायस करता है ।
है ।
Reference: https://indiankanoon.org/doc/72281371/

DPP 2023 DAY 48 54


Contact us : info@onlyias.com

OnlyIAS Nothing Else Visit : dpp.onlyias.in


Contact : +91-7007 931 912

• कथन 2 र्गलत है : राज्यपाल के पास संवविान में ननयजु क्त या ववववास को खोने के पवचात सरकार को
उजल्लखखत वववेकाचिकार, संवि
ै ाननक वववेक और वववेक की बखासस्त करना
दो िेखणयां हैं। Reference:
https://indianexpress.com/article/cities/jaipur/tensio
• पररजस्थनतजन्य वववेकाचिकार एक नछपा हुआ वववेक है n-between-rajasthan-governor-university-vc-building-
जो एक प्रचसलत राजनीनतक जस्थनत की अननवायसताओं से up-since-last-year-7806689/
प्राप्त होता है । मुख्यमंत्री की ननयुजक्त तब होती है जब राज्य
वविानसभा में ककसी भी पाटी के पास स्पष्ट बहुमत नहीं Q.80) Ans: B
होता है या जब वतसमान मुख्यमंत्री की अप्रत्यासशत रूप से Exp:
मत्ृ यु हो जाती है और कोई स्पष्ट उत्तराचिकारी नहीं होता है , बाहरी लोर्गों को महा धचव के रूप में तनयक्
ु त करने की

तो यह पररजस्थनतजन्य वववेक है । अन्य पररजस्थनतजन्य मौजूदा प्रथा े ंबधं धत धचंताएाँ:

वववेक में राज्य वविान सभा के ववववास को प्रदसशसत करने में • शष्क्क्तयों के पथ
ृ क्करण के स द्धांत के णखलाफ: सेवारत
असमथस होने पर मंब्रत्रपररर्द की बखासस्तगी शासमल है । या सेवाननवत्त
ृ ससववल सेवकों को महासचचव के पद पर
राज्य वविान सभा की बखासस्तगी जब मंब्रत्रपररर्द अपना ननयुक्त करना भारतीय संवविान के तहत पररकजल्पत

बहुमत खो दे ती है । शजक्त के पथ
ृ क्करण के ससद्िांत का उल्लंघन है और
• ंववधान के अनु ार राज्यपाल के पा तनम्नसलणखत न्यायपासलका द्वारा इसे बनु नयादी ढांचे के रूप में भी

मामलों में ंवैधातनक वववेकाधधकार है : बरकरार रखा गया है । संवविान के अनच्


ु छे द 98 में दो

राष्रपनत के ववचार के सलए वविान सभा द्वारा पाररत अलग-अलग सचचवालयों का प्राविान करके इस ससद्िांत

वविेयक का आरक्षण। पर जोर हदया गया है कक सचचवालयों को कायसपासलका

0 संबचं ित राज्य में राष्रपनत शासन लगाने की ससफाररश। सरकार से स्वतंत्र होना चाहहए। एक अलग और स्वतंत्र

o जब वह ककसी ननकटवती केंद्र शाससत प्रदे श के प्रशासक के सचचवालय एक कायसशील संसदीय लोकतंत्र की एक ववशेर्ता

रूप में अपने कायों का प्रयोग कर रहा हो। है । महासचचव के पद पर ससववल सेवकों की ननयुजक्त

0 असम, मेघालय, ब्रत्रपरु ा और समजोरम सरकार द्वारा एक सचचवालय की स्वतंत्रता को कमजोर कर सकती है ।

स्वायत्त जनजातीय जजला पररर्द को खननज अन्वेर्ण के • ववधातयका के कामकाज के बारे में जानकारी का अर्ाव:

सलए लाइसेंस से प्राप्त होने वाली रॉयल्टी के रूप में दे य रासश महासचचव को कई तरह के कतसव्य और कायस सौंपे जाते हैं,

का ननिासरण। जजनके सलए संसदीय प्रकियाओं, प्रथाओं और उदाहरणों के

o राज्य के प्रशासननक मामलों के संबंि में मुख्यमंत्री से व्यापक ज्ञान और समद्


ृ ि अनभ
ु व की आववयकता होती है ।

जानकारी प्राप्त करना। अचिकांश ससववल सेवकों में इस ववशेर्ज्ञता की कमी हो

• कथन 3 ही है : 13 जुलाई 2016 को नबाम रे ब्रबया बनाम सकती है ।

डडप्टी स्पीकर में सुप्रीम कोटस के संवविान पीठ के फैसले ने • कायभपासलका को जवाबदे ह ठहराने में ववधातयकाओं की

कहा कक एक राज्यपाल कैब्रबनेट की ससफाररश पर फ्लोर र्ूसमका: भारत जैसी संसदीय व्यवस्था में, वविान पाररत

टे स्ट के सलए वविानसभा की बैठक बुलाने के सलए बाध्य है । करने के अलावा, संसद से कायसपासलका के प्रशासननक

एक राज्यपाल अपने "वववेक" को ननयोजजत नहीं कर सकता व्यवहार की जांच करने की भी अपेक्षा की जाती है । एक

है , और उसे फ्लोर टे स्ट के सलए वविानसभा को बुलाने के शजक्तशाली संसद का अथस है अचिक जवाबदे ह कायसपासलका।

सलए कैब्रबनेट की "सहायता और सलाह" का सख्ती से पालन सचचवालय जैसे महत्वपूणस वविानयका ननकायों के प्रमुख के

करना चाहहए। राज्यपाल की वववेकािीन शजक्तयााँ ननहदसष्ट सलए पूवस नौकरशाहों की ननयुजक्त से हहतों के संभाववत

क्षेत्रों तक सीसमत हैं जैसे राष्रपनत को ककसी वविेयक को टकराव की जस्थनत पैदा हो सकती है ।

स्वीकृनत दे ना या रोकना/संदसभसत करना या मुख्यमंत्री की

DPP 2023 DAY 48 55


Contact us : info@onlyias.com

OnlyIAS Nothing Else Visit : dpp.onlyias.in


Contact : +91-7007 931 912

Reference: https://www.thehindu.com/opinion/op- काला सागर बंदरगाह भम


ू ध्यसागरीय और उससे आगे तक
ed/needed-an-indian-legislative-
service/article65249778.ece पहुंचा जा सकता है ।
• मॉजन्रक्स असभसमय ऑस्रे सलया, बल्
ु गाररया, फ्रांस,
ग्रीस, जापान, रोमाननया, यूगोस्लाववया, यूनाइटे ड ककं गडम,
Q.81) Ans: C
Exp: सोववयत संघ और तुकी द्वारा हस्ताक्षररत जलडमरूमध्य
• कथन 1 ही है : सेवा मतदाताओं को ETPBS का उपयोग को शाससत करने वाला अंतरासष्रीय समझौता है जो तुकी में
करने की अनुमनत दे ने के सलए चुनाव आचरण ननयम, 1961 बोस्पोरस और डाडासनेल्स जलडमरूमध्य को ननयंब्रत्रत करने
को 2016 में संशोचित ककया गया था। इस प्रणाली के तहत, वाला अंतरासष्रीय समझौता है और नवंबर 1936 से प्रभावी है ।
डाक मतपत्र पंजीकृत सेवा मतदाताओं को इलेक्रॉननक रूप • यह समझौता तक
ु ी को काला सागर जलडमरूमध्य पर
से भेजे जाते हैं। सेवा मतदाता तब ETPB (घोर्णा पत्र और ननयंत्रण रखने का अचिकार दे ता है । युद्ि की जस्थनत में ,
कवर के साथ) डाउनलोड कर सकता है , मतपत्र पर अपना समझौता अंकारा को नौसैननक युद्िपोतों के पारगमन को
जनादे श दजस कर सकता है और इसे सामान्य मेल के माध्यम ववननयसमत करने और संघर्स में शासमल दे शों से संबचं ित
से ननवासचन क्षेत्र के ररटननांग अचिकारी को भेज सकता है । युद्िपोतों के सलए जलडमरूमध्य को अवरुद्ि करने का
पोस्ट में एक सत्यावपत घोर्णा पत्र शासमल होगा (मतदाता अचिकार दे ता है । संचि के अनच्
ु छे द 19 में काला सागर के
द्वारा हस्ताक्षररत होने के बाद एक ननयक्
ु त वररष्ठ दे शों के सलए एक अपवाद है जो रूसी युद्िपोतों को काला
अचिकारी की उपजस्थनत में जो इसे सत्यावपत करे गा)। सागर में प्रवेश करने या बाहर ननकलने से रोकने में तुकी की
• कथन 2 ही है : जन प्रनतननचित्व (संशोिन) अचिननयम, शजक्त को प्रभावी ढं ग से कमजोर कर सकता है ।
2010 के माध्यम से, पात्र प्रवासी भारतीय जो छह महीने से • अनुच्छे द 19 कहता है , "युद्िपोत युद्िरत शजक्तयों से
अचिक समय तक ववदे श में रहे थे, उन्हें मतदान करने की संबंचित हैं, चाहे वे काला सागर शजक्तयााँ हों या नहीं, जो
अनुमनत दी गई थी, लेककन केवल मतदान केंद्र पर अपने हठकानों से अलग हो गए हैं, वे वापस आ सकते हैं।"
व्यजक्तगत रूप से जहां उन्हें एक ववदे शी मतदाता के रूप में इसका मतलब है कक युद्िपोत मागस के माध्यम से अपने
नामांककत ककया गया था। एक एनआरआई ननवासचन क्षेत्र में वास्तववक हठकानों पर लौट सकते हैं और तक
ु ी इसे रोक नहीं
अपने ननवास स्थान पर मतदान कर सकता है , जैसा कक सकता है । यह शब्द रूसी बेडे पर भी लागू होता है जो वतसमान
पासपोटस में उजल्लखखत है । वह केवल व्यजक्तगत रूप से में काला सागर में है जो भूमध्यसागरीय या बाजल्टक सागर
मतदान कर सकता है और पहचान स्थावपत करने के सलए में एक आिार से संबंचित है । रूस उन्हें काला सागर से बाहर
उसे मतदान केंद्र पर अपना पासपोटस मूल रूप में प्रस्तुत ननकालने के सलए स्वतंत्र है ।
करना होगा। ंदर्भ:
ं र्भ:
द https://indianexpress.com/article/explained/turkey-
https://epaper.thehindu.com/Home/ShareArticle?Org russia-black-sea-7795658/
Id=GLG9L69V7.1&imageview=0
Q.83) Ans: C
Q.82) Ans: D Exp:
Exp: • पीएम दक्ष (प्रिानमंत्री दक्ष और कुशल संपण
ू स हहतग्राही)
• ववकल्प D ही है : काला सागर जलडमरूमध्य को तक
ु ी वर्स 2020-21 से लागू ककया जा रहा है । इसके तहत पात्र
जलडमरूमध्य के रूप में भी जाना जाता है , काला सागर लक्ष्य समूहों को अल्पावचि प्रसशक्षण कायसिम पर कौशल
जलडमरूमध्य बोस्पोरस और डाडासनेल्स जलडमरूमध्य हैं ववकास प्रसशक्षण कायसिम प्रदान ककया जाता है ; और अप-
और मरमारा सागर के माध्यम से एजजयन सागर और काला जस्कसलंग/रीजस्कसलंग; उद्यसमता ववकास कायसिम और
सागर को जोडते हैं। यह एकमात्र मागस है जजसके माध्यम से दीघसकासलक प्रसशक्षण कायसिम।

DPP 2023 DAY 48 56


Contact us : info@onlyias.com

OnlyIAS Nothing Else Visit : dpp.onlyias.in


Contact : +91-7007 931 912

• ये प्रसशक्षण कायसिम सरकारी प्रसशक्षण संस्थानों, कौशल को राज्य के मामले की गण


ु दोर् की जांच ककए उसे
ववकास और उद्यसमता मंत्रालय द्वारा गहठत क्षेत्र कौशल स्वीकार करना चाहहए।
पररर्दों और अन्य ववववसनीय संस्थानों के माध्यम से Reference:
https://indianexpress.com/article/explained/explaine
कायासजन्वत ककए जा रहे हैं। अनुसूचचत जानत, अन्य वपछडा d-bail-provisions-in-uapa-and-the-case-of-cong-ex-
वगस, ईबीसी, ववमक्
ु त जनजानत, कचरा बीनने वाले, हाथ से councillor-ishrat-jahan-7821404/
मैला ढोने वाले, रांसजेंडर और अन्य समान िेखणयों सहहत
स्वच्छता कायसकतास इस योजना के सलए पात्र होंगे।
• मंत्रालयों के तहत 3 ननगमों द्वारा कायसिम को Q.85) Ans: b
Exp:
कायासजन्वत ककया जाता है :
o राष्रीय अनुसूचचत जानत ववत्त और ववकास ननगम • कथन 1 र्गलत है : संवविान अनुसूचचत
(NSFDC), जनजानतयों (एसटी) की मान्यता के मानदं ड को
o राष्रीय वपछडा वगस ववत्त और ववकास ननगम (NBCFDC),
पररभावर्त नहीं करता है और इससलए 1931 की
o राष्रीय सफाई कमसचारी ववत्त और ववकास ननगम
जनगणना में ननहहत पररभार्ा का उपयोग
(NSKFDC)।
स्वतंत्रता के बाद के प्रारं सभक वर्ों में ककया गया था।
Reference:
https://pib.gov.in/PressReleasePage.aspx?PRID=180 • हालांकक, संवविान का अनुच्छे द 366(25) केवल
6159 अनुसूचचत जनजानतयों (एसटी) को पररभावर्त
करने के सलए एक प्रकिया प्रदान करता है :
Q.84) Ans: d “अनस
ु चू चत जनजानतयों का अथस है ऐसी
Exp: जनजानतयां या आहदवासी समुदाय या ऐसी
• कथन 1 ही है : यए
ू पीए के साथ प्रमख
ु समस्या जनजानतयों या जनजातीय समुदायों के हहस्से या
इसकी िारा 43(D)(5) में ननहहत है , जो ककसी भी समह
ू जजन्हें अनच्
ु छे द 342 के तहत समझा जाता
आरोपी व्यजक्त को जमानत पर ररहा करने से
है । इस संवविान के प्रयोजनों के सलए अनुसूचचत
रोकता है , अगर पुसलस ने आरोप पत्र दायर ककया है जनजानत हो।
कक यह मानने के सलए उचचत आिार हैं कक ऐसे
• कथन 2 र्गलत है: 5वीं अनुसच
ू ी असम, मेघालय,
व्यजक्त के खखलाफ आरोप प्राथसमक है प्रथम दृष्टया ब्रत्रपुरा और समजोरम (एएमटीएम) के अलावा अन्य
सच। राज्यों में अनस
ु चू चत क्षेत्रों और अनस
ु चू चत
• िारा 43(D)(5) का प्रभाव यह है कक एक बार जब जनजानतयों के प्रशासन और ननयंत्रण के सलए
पुसलस ककसी व्यजक्त पर यूएपीए के तहत आरोप प्राविान करती है ।
लगाने का चुनाव कर लेती है , तो जमानत दे ना बेहद • छठी अनुसूची असम, मेघालय, ब्रत्रपुरा और
मुजवकल हो जाता है । जमानत स्वतंत्रता के समजोरम में जनजातीय क्षेत्रों के प्रशासन से संबचं ित
संवैिाननक अचिकार की सुरक्षा और गारं टी है । यह है । 705 से अचिक जनजानतयां हैं जजन्हें अचिसचू चत
प्राविान न्यानयक तकस के सलए बहुत कम जगह ककया गया है । सबसे अचिक संख्या में आहदवासी
छोडता है , और यए
ू पीए के तहत जमानत दे ना समुदाय ओडडशा में पाए जाते हैं।
लगभग असंभव बना दे ता है । • कथन 3 र्गलत है: डारलोंग ब्रत्रपरु ा का एक
• कथन 2 ही है : जहूर अहमद शाह वटाली के मामले आहदवासी समद
ु ाय है , जजसकी आबादी 11,000 है ।
में , सुप्रीम कोटस ने 2019 में पुजष्ट की कक अदालतों समुदाय में सशक्षा और सांस्कृनतक गनतववचि का
उच्च प्रसार है और समुदाय के सदस्य स्थानीय

DPP 2023 DAY 48 57


Contact us : info@onlyias.com

OnlyIAS Nothing Else Visit : dpp.onlyias.in


Contact : +91-7007 931 912

प्रशासन में वररष्ठ पदों पर कायस करते हैं। उदाहरण अल्पसंख्यक घोवर्त करने जैसे मामले उक्त राज्य
के सलए, एक आहदवासी संगीतज्ञ और रो़म (एक में अपनी पसंद के शैक्षखणक संस्थानों की स्थापना
आहदवासी वाद्य यंत्र) उस्ताद थंगा डारलोंग को और प्रशासन कर सकते हैं। और राज्य स्तर पर
संस्कृनत में उनके योगदान के सलए कुछ साल पहले अल्पसंख्यकों की पहचान के सलए हदशाननदे श
प्रनतजष्ठत पद्म िी से सम्माननत ककया गया था। ननिासररत करने पर संबचं ित राज्य सरकारों द्वारा
Reference: ववचार ककया जा सकता है ।
https://www.thehindu.com/news/national/lok-sabha- Reference:
nod-to-include-two-tribes-in-tripura-st- https://www.thehindu.com/news/national/states-
list/article65268500.ece free-to-identify-minorities-and-allow-them-to-
administer-educational-
institutions/article65265596.ece
Q.86) Ans: C
Exp:
• कथन 1 ही है : अनुच्छे द 29 में प्राविान है कक Q.87) Ans: d
भारत के ककसी भी हहस्से में रहने वाले नागररकों के Exp:
ककसी भी वगस की अपनी एक अलग भार्ा, सलवप या • कथन 1 ही है : अनुच्छे द 365 कहता है कक जब भी

संस्कृनत है , उसे इसे संरक्षक्षत करने का अचिकार कोई राज्य केंद्र के ककसी ननदे श का पालन करने या

होगा और िासमसक अल्पसंख्यकों के साथ-साथ उसे लागू करने में ववफल रहता है , तो राष्रपनत के

भार्ाई अल्पसंख्यकों दोनों को संरक्षण प्रदान सलए यह मानना वैि होगा कक ऐसी जस्थनत उत्पन्न

करे गा। हो गई है जजसमें राज्य की सरकार संवविान के

• हालांकक, उच्चतम न्यायालय ने माना कक इस लेख प्राविान अनुसार नहीं चल सकती है ।

का दायरा केवल अल्पसंख्यकों तक ही सीसमत नहीं • कथन 2 र्गलत है: राष्रपनत शासन के

है , क्योंकक अनुच्छे द में ‘नागररकों के वगस’ शब्द के पररणामस्वरूप, जब ककसी राज्य में राष्रपनत

इस्तेमाल में अल्पसंख्यकों के साथ-साथ शासन लगाया जाता है तो राष्रपनत को

बहुसंख्यक भी शासमल हैं। ननम्नसलखखत असािारण शजक्तयां प्राप्त होती हैं:

• कथन 2 ही है : सातवें संवि


ै ाननक (संशोिन) o वह राज्य सरकार के कायों और राज्यपाल

अचिननयम 1956 में अनुच्छे द 350 (b) जोडा गया। या राज्य में ककसी अन्य कायसकारी

जो भारत के राष्रपनत द्वारा ननयुक्त भार्ाई प्राचिकरण में ननहहत शजक्तयों को ले

अल्पसंख्यकों के सलए एक ववशेर् अचिकारी का सकता है ।

प्राविान करता है और संवविान के तहत भार्ाई o वह घोर्णा कर सकता है कक राज्य

अल्पसंख्यकों के सलए प्रदान ककए गए सुरक्षा उपायों वविानमंडल की शजक्तयों का प्रयोग संसद

से संबंचित हर मामले की जांच करना ववशेर् द्वारा ककया जाना है ।

अचिकारी का कतसव्य होगा। • कथन 3 र्गलत है :

• कथन 3 र्गलत है : संसद और राज्य वविानयका दोनों 1975 के 38वें संशोिन अचिननयम ने अनुच्छे द 356

को अल्पसंख्यकों और उनके हहतों की सुरक्षा के को अंनतम और ननणासयक लागू करने में राष्रपनत

सलए कानून बनाने की समवती शजक्तयां हैं। की संतजु ष्ट की, जजसे ककसी भी आिार पर ककसी भी

लद्दाख, अरुणाचल प्रदे श, पंजाब, समजोरम, अदालत में चुनौती नहीं दी जाएगी। लेककन, इस

लक्षद्वीप, कवमीर, नागालैंड, मेघालय और मखणपुर प्राविान को बाद में 1978 के 44वें संशोिन

में यहूदी, बहावाद और हहंद ू िमस के अनय अचिननयम द्वारा हटा हदया गया था, जजसका अथस
ु ानययों को

DPP 2023 DAY 48 58


Contact us : info@onlyias.com

OnlyIAS Nothing Else Visit : dpp.onlyias.in


Contact : +91-7007 931 912

था कक राष्रपनत की संतजु ष्ट न्यानयक समीक्षा से परे


Onlyias द्वारा ववशेर्ांक
नहीं है ।
जनर्गणना इततहा :
Reference:
https://indianexpress.com/article/india/bjp-mp- • दे श के ववसभन्न हहस्सों में 1865 और 1872 के बीच
roopa-ganguly-rajya-sabha-birbhum-violence- एक व्यवजस्थत और आिुननक जनसंख्या जनगणना,
7835810/
अपने वतसमान स्वरूप में , गैर-समकासलक रूप से
आयोजजत की गई थी।
Q.88) Ans: a • 1872 में समाप्त हुए इस प्रयास को लोकवप्रय रूप से
Exp:
भारत की पहली जनसंख्या जनगणना के रूप में
• कथन 1 ही है : भारत सरकार ने जनगणना के
चचजननत ककया गया है ।
दौरान पूछे जाने वाले प्रवनों की अनुसूची में
• हालांकक, भारत में पहली समकासलक जनगणना
“इलेक्रॉननक रूप” और “स्व-गणना” को शासमल
1881 में हुई थी। तब से, हर दस साल में एक बार
करने के सलए जनगणना ननयम, 1990 में संशोिन
ननबासि रूप से जनगणना की जाती रही है ।
ककया है । यह आगामी जनगणना और राष्रीय
जनसंख्या रजजस्टर (एनपीआर) में ऑनलाइन स्व-
Reference:
गणना की अनुमनत दे गा। एनपीआर दे श के https://indianexpress.com/article/india/centre-
सामान्य ननवाससयों का एक रजजस्टर है और इसका notifies-rules-for-online-self-enumeration-during-
census-and-npr-7816862/
उद्दे वय दे श के प्रत्येक सामान्य ननवासी का एक
व्यापक पहचान डेटाबेस बनाना है ।
• कथन 2 र्गलत है : जनगणना अचिननयम, 1948 Q.89) Ans: b
Exp:
की िारा 8 में गणनाकतास को जनगणना के संबंि में
• कथन 1 र्गलत है: आपराचिक प्रकिया (पहचान)
प्रवन पूछने का अचिकार हदया गया है और कुछ
अचिननयम, एकत्र ककए जा सकने वाले डेटा के
अपवादों के साथ उत्तरदाताओं के सलए उत्तर दे ना
प्रकार का ववस्तार करता है , जजन व्यजक्तयों से ऐसा
अननवायस बनाता है । इसके अलावा, "वप्रंट मीडडया,
डेटा एकत्र ककया जा सकता है , और प्राचिकरण जो
इलेक्रॉननक मीडडया, सोशल मीडडया" को अभ्यास
इस तरह के संग्रह को अचिकृत कर सकता है ।
के व्यापक प्रचार को सुननजवचत करने के तरीकों की
अचिननयम में संग्रह की तारीख से 75 वर्ों के सलए
सूची में जोडा गया है । नए जनगणना ननयमों से
एकत्र ककए गए वववरण को डडजजटल या
पहले, सूचचयों में केवल रे डडयो, ऑडडयो-ववजुअल
इलेक्रॉननक रूप में बनाए रखने की आववयकता है ।
और पोस्टर शासमल थे।
उन व्यजक्तयों के मामले में ररकॉडस को नष्ट ककया
• इससे पहले, जनगणना 2021 के पहले चरण में ,
जा सकता है जजन्हें पहले दोर्ी नहीं ठहराया गया है ,
2020 में होने वाली एक एनपीआर प्रकिया को
और जो सभी कानूनी उपायों को समाप्त करने के
कोववड-19 संकट के कारण अननजवचत काल के सलए
बाद ब्रबना मक
ु दमे के ररहा कर हदए गए हैं, या
स्थचगत कर हदया गया था। जनसंख्या जनगणना
अदालत द्वारा बरी कर हदया गया है ।
ककसी दे श या दे श के ककसी हहस्से के सभी व्यजक्तयों
• कथन 2 ही है : इस अचिननयम के अनुसार,
के एक ववसशष्ट समय पर जनसांजख्यकीय, आचथसक
प्रनतरोि या वववरण दे ने से इनकार करना भारतीय
और सामाजजक डेटा एकत्र करने, संकसलत करने,
दं ड संहहता, 1860 के तहत अपराि माना जाएगा।
वववलेर्ण करने, प्रसार करने की प्रकिया है ।
इस तरह के प्रनतरोि या इनकार के मामले में,
पुसलस अचिकारी या जेल अचिकारी राज्य द्वारा

DPP 2023 DAY 48 59


Contact us : info@onlyias.com

OnlyIAS Nothing Else Visit : dpp.onlyias.in


Contact : +91-7007 931 912

बनाए गए ननयमों के तहत ननिासररत तरीके से बजस्तयां लाईं, जजसके कारण पाइकाओं ने अपनी
वववरण एकत्र कर सकते हैं। राज्य सरकार हो या सम्पदा खो दी।
केंद्र सरकार। इस अचिननयम के तहत, एक • त्रिदटश शोर्क नीततयां: खद
ु ास के राजा को हटाने के
मजजस्रे ट ककसी व्यजक्त को सीआरपीसी (दं ड साथ, ब्रिहटश प्रशासन की शोर्णकारी नीनतयां
प्रकिया संहहता) के तहत जांच या कायसवाही के ओडडशा के लोगों के सलए असहनीय हो गईं।
उद्दे वय से वववरण दे ने का ननदे श दे सकता है । अथसव्यवस्था और राजस्व प्रणासलयों में ननरं तर
• कथन 3 ही है : आपराचिक प्रकिया (पहचान), हस्तक्षेप से ककसानों का शोर्ण और उत्पीडन हुआ
अचिननयम कैहदयों की पहचान अचिननयम, 1920 और अंततः अंग्रेजों के खखलाफ ववद्रोह शुरू हो गया।
के तहत तलाश करना है । इस अचिननयम ने केवल • नई मद्र
ु ा प्रणाली : अंग्रेजों ने मद्र
ु ा प्रणाली को कौडी
राज्य सरकार में ननयम बनाने की शजक्त ननहहत से बदलकर रुपया कर हदया। ग्रामीणों को नई मुद्रा
की। इस अचिननयम के संशोिन ने इस शजक्त का को संभालने में काफी समस्याओं का सामना करना
ववस्तार केंद्र सरकार को भी ककया है । वववरण एकत्र पडा और स्थानीय महाजनों द्वारा उनका घोर
करने के तरीके जैसे ववसभन्न मामलों पर केंद्र या शोर्ण ककया गया।
राज्य सरकार ननयम बना सकती है । • त्रिदटश नमक नीतत: ओडडशा के लंबे समद्र
ु ी तट से
Reference: भारी मात्रा में नमक का उत्पादन होता था जजसका
https://www.thehindu.com/news/national/criminal-
procedure-identification-bill-to-be-tabled-in-lok- उपयोग इस भूसम के लोग स्वतंत्र रूप से करते थे।
sabha-on-monday/article65266837.ece हालााँकक, ब्रिहटश अचिकाररयों ने जमींदारों और
तटीय क्षेत्र के स्थानीय लोगों को नमक बनाने के
Q.90) Ans: a उनके पारं पररक अचिकारों से वंचचत कर हदया।
Exp: Reference:
• पाइका ववद्रोह एक सशस्त्र ववद्रोह था जो ब्रिहटश https://www.thehindu.com/news/national/paika-
rebellion-to-be-included-as-case-study-in-history-
ईस्ट इंडडया कंपनी (ईआईसी) के खखलाफ ओडडशा textbook-minister/article37805249.ece
में हुआ था। यह पहले ससपाही ववद्रोह से लगभग 40
साल पहले हुआ था। पाइका ओडडशा के गजपनत
Q.91) Ans: c
शासकों के ककसान समसलसशया थे जजन्होंने राजा को Exp:
सैन्य सेवाएं दीं। उनके पास ककराया-मुक्त भसू म थी • सोहराई पें हटंग झारखंड के हजारीबाग जजले में
जो उन्हें खुदास साम्राज्य में उनकी सैन्य सेवा के सलए आहदवासी महहलाओं द्वारा इस्तेमाल की जाने
दी गई थी। वाली पें हटंग का एक स्वदे शी कला रूप है । मवेसशयों
• अंग्रेजों ने ओडडशा में खुद को स्थावपत ककया जब और फसल का स्वागत करने के सलए समट्टी की
ईआईसी ने 1803 में खद
ु ास के राजा, राजा मुकंु द दे व दीवारों पर पेंहटंग की जाती थी। महहलाएं अपने घरों
को गद्दी से उतार हदया। बक्शी जगबंिु ववद्यािर की सफाई करती हैं और अपनी दीवारों को सजाने के
के अिीन पाइका, गजपनत राजा की समसलसशया सलए सोहराई कला के सभवत्त चचत्रों का उपयोग करती
सेना के वंशानग
ु त प्रमख
ु , आहदवाससयों और समाज थीं।
के अन्य वगस का समथसन लेते हुए ववद्रोह शुरू ककया । • यह कला रूप 10,000-4,000 ईसा पूवस से जारी है ।
पाइका ववद्रोह के प्रमख
ु कारण: पहले, यह ज्यादातर गुफाओं में प्रचसलत था, लेककन
• र्ू-राजस्व पद्धतत: अंग्रेजों के आगमन और बाद में इसे समट्टी की दीवारों वाले घरों में
औपननवेसशक शासन की स्थापना ने नई भू-राजस्व स्थानांतररत कर हदया गया।

DPP 2023 DAY 48 60


Contact us : info@onlyias.com

OnlyIAS Nothing Else Visit : dpp.onlyias.in


Contact : +91-7007 931 912

• ोहराई कला की प्रमख


ु ववशेर्ताएं इ प्रकार हैं: Reference: UPSC 2015 question
https://www.downtoearth.org.in/news/environment/
o सोहराई कला रूप मोनोिोमैहटक या रं गीन santhali-sohrai-murals-an-etch-in-time-81680
दोनों हो सकता है । लोग दीवार को सफेद
समट्टी की परत से ढक दे ते हैं, जबकक परत
Q.92) Ans: a
अभी भी गीली होती है , वे उस पर अपनी Exp:
उं गसलयों से खींचते हैं। • कथन 1 ही है : साहहत्य अकादमी साहहत्य
o उनके डडजाइन फूलों और फलों से लेकर अकादमी के पत्रों का उत्सव है । यह भारत का सबसे
प्रकृनत से प्रेररत ववसभन्न डडजाइनों तक हैं। समावेशी साहहत्य उत्सव है । साहहत्योत्सव उत्सव
जजस गाय के गोबर का इस्तेमाल पहले घर भारत की स्वतंत्रता की 75वीं वर्सगांठ के उपलक्ष्य में
की दीवारों को ढाँ कने के सलए ककया जाता आयोजजत समारोह का एक हहस्सा होगा। इस
था, उसका उपयोग रं ग जोडने के सलए महोत्सव के दौरान प्रनतजष्ठत साहहत्य अकादमी
ककया जाता है । पुरस्कार भी प्रदान ककए जाएंगे।
o कलाकार अपने चचत्रों में सहज होते हैं। • कथन 2 र्गलत है : साहहत्य अकादमी पुरस्कार 1954
कला के डडजाइन आम तौर पर कलाकार में स्थावपत ककया गया था। यह साहहत्य का एक
की स्मनृ त से तैयार ककए जाते हैं। कलाकार सम्मान है जजसे साहहत्य अकादमी, भारत की
का व्यजक्तगत अनुभव और प्रकृनत के साथ राष्रीय पत्र अकादमी द्वारा प्रनतवर्स प्रदान ककया
उनकी बातचीत सबसे बडा प्रभाव प्रदसशसत जाता है । अकादे मी उन भार्ाओं में साहहजत्यक
करती है । कृनतयों के सलए सालाना 24 परु स्कार प्रदान करती
• ोहराई खोवर पें दटंर्ग को 2020 में र्ौर्गोसलक ंकेत है , और भारत की भार्ाओं में साहहजत्यक अनुवादों
(जीआई) टै र्ग समला। के सलए समान संख्या में पुरस्कार प्रदान करती है ।
• भारत के संवविान में उजल्लखखत 22 भार्ाओं के
Onlyias द्वारा
अलावा, साहहत्य अकादमी ने अंग्रेजी और
ववशेर्ांक
राजस्थानी को उन भार्ाओं के रूप में मान्यता दी है
ोहराई:
जजनमें इसका कायसिम लागू ककया जा सकता है ।
• सोहराई भारतीय राज्यों झारखंड, छत्तीसगढ़,
ज्ञानपीठ परु स्कार के बाद साहहत्य अकादमी
ओडडशा और पजवचम बंगाल में संथाल, मुंडा,
पुरस्कार भारत सरकार द्वारा हदया जाने वाला
प्रजापनत, खरु मी और उरांव जनजानतयों का
दस
ू रा सबसे बडा साहहजत्यक सम्मान है ।
पांच हदवसीय त्योहार है ।
• कथन 3 र्गलत है : 1954 में, स्
ं कृतत मंत्रालय ने
• यह एक हावेस्ट फेजस्टवल है जो सहदस यों की
ादहत्य अकादमी, र्ारत की राष्ट्रीय पत्र अकादमी
फसल के मौसम की शरु
ु आत में आयोजजत
की स्थापना की। हालांकक यह रकार द्वारा
ककया जाता है । इसे पशु उत्सव भी कहा जाता
स्थावपत ककया र्गया था, लेककन एक स्वायत्त
है ।
ंर्गठन के रूप में कायभ करता है । 1960 में इ े
• यह अमावस्या अवचि के दौरान हहंद ू कैलेंडर के
ो ायटी पंजीकरण अधधतनयम, 1860 के तहत
कानतसक (अक्टूबर-नवंबर) महीने में मनाया
एक ो ायटी के रूप में पंजीकृत ककया र्गया था।
जाता है ।
यह र्ारत की र्ार्ाओं में ादहत्य को बढ़ावा दे ने के
• संथाल जनजानत के सलए सोहराई सबसे
सलए मवपभत एक ंर्गठन है और ददल्ली में मंडी
महत्वपूणस त्योहार है ।
हाउ के पा रवींद्र र्वन में ष्क्स्थत है ।

DPP 2023 DAY 48 61


Contact us : info@onlyias.com

OnlyIAS Nothing Else Visit : dpp.onlyias.in


Contact : +91-7007 931 912

• एक ववस्तार के रूप में , उन्होंने ‘सत्यशोिक वववाह’


Reference:
https://pib.gov.in/PressReleseDetail.aspx?PRID=1804 शुरू ककया, जहां शादी करने वाले जोडे को सशक्षा और
402 समानता को बढ़ावा दे ने का संकल्प लेना होता है ।
1897 में साववत्रीबाई ने पूरे महाराष्र में फैले
Q.93) Ans: D बुबोननक प्लेग के पीडडतों के सलए एक जक्लननक
Exp: खोला। वविवाओं के ससर मुंडवाने की परं परा के
ाववत्रीबाई फुले: खखलाफ उन्होंने नाइयों द्वारा बहहष्कार का
• साववत्रीबाई फुले का जन्म 3 जनवरी, 1831 को आयोजन ककया। उनके सम्मान में 2015 में पण
ु े
महाराष्र के नायगांव में हुआ था। फुले को व्यापक ववववववद्यालय का नाम बदलकर साववत्रीबाई फुले
रूप से ब्रिहटश भारत के शासन के तहत समान ववववववद्यालय कर हदया गया।
सशक्षा के अवसर सनु नजवचत करने में उनके Reference:
महत्वपूणस योगदान के सलए भारत की पहली पीढ़ी https://www.google.com/amp/s/indianexpress.com/a
rticle/explained/explained-who-are-the-phules-what-
की पहली आिनु नक नारीवाहदयों में से एक माना is-the-row-around-maharashtra-governors-remarks-
जाता है । on-them-7805497/lite/
• 1848 में , वह भारत में पहली महहला सशक्षक्षका बनीं
और अपने पनत, समाज सि
ु ारक ज्योनतराव फुले के Q.94) Ans: a
साथ समलकर उन्होंने लडककयों के सलए एक स्कूल Exp:
खोला। उन्होंने अहमदनगर में एक अमेररकी • यग्ु म 1 ही है : घन वाद्य (इडडयोफोन्स या ठोस

समशनरी द्वारा संचासलत संस्थान और पण यंत्र) घन वाद्या स्व-कंपनकतास हैं, अथासत उनकी
ु े के
नॉमसल स्कूल में एक सशक्षक प्रसशक्षण पाठ्यिम भी लोचदार प्रकृनत के कारण, उनकी अपनी एक ध्वनन

सलया। इसके बाद उन्होंने पण होती है , जो घर्सण या हवा से टकराने, तोडने या


ु े के महारवाडा में
लडककयों को पढ़ाना शुरू ककया। दोनों ने जानत- उत्तेजजत होने पर तरं गों में उत्सजजसत होती है । ये

आिाररत पहचान के आिार पर भेदभाव के खखलाफ उपकरण ननजवचत वपच बनाने में सक्षम नहीं हैं जो

भी काम ककया, जजसका पण एक राग बनाने के सलए आववयक हैं। इससलए


ु े में समाज के रूहढ़वादी
वगों ने जोरदार ववरोि ककया। शास्त्रीय संगीत में इनका प्रयोग सीसमत है । उडीसा

• ज्योनतबा फुले और साववत्रीबाई फुले ने गभसवती के कोणाकस के सय


ू स मंहदर में हमें झांज बजाती एक

वविवाओं और बलात्कार पीडडतों की सुरक्षा के सलए महहला की लगभग 8 फीट ऊंची यह ववशाल मूनतस

एक चाइल्डकैअर सेंटर ‘बाल्यता प्रनतबंदक गह हदखाई दे ती है । उदाहरण: बेल, भजन चक्कल,ू भुंज,
ृ ’ की
स्थापना की। साववत्रीबाई फुले ने सत्यशोिक आहद।

समाज के काम को ननदे सशत करने में भी महत्वपूणस • युग्म 2 र्गलत है : टाट वाद्य (कॉडोफोन या तार वाले

भूसमका ननभाई, जजसे उनके पनत ने सीमांत की वाद्ययंत्र) में , ध्वनन एक तार या तार के कंपन से

ननचली जानतयों के सलए समान अचिकार प्राप्त उत्पन्न होती है । ये कंपन तना खींची गई डोरी को

करने के उद्दे वय से बनाया था। फुले दं पवत्त ने दो तोडकर या झक


ु कर (रावणस्त्रम सबसे पहले ज्ञात

शैक्षक्षक रस्टों की शुरुआत की- नेहटव फीमेल स्कूल, झक


ु े हुए वाद्ययंत्रों में से एक है ) के कारण होता है ।

पुणे और द सोसाइटी फॉर प्रमोशन द एजक इनका प्रनतननचित्व पुराने हदनों की कई मनू तसयों और
ु े शन
ऑफ महार, मांग और अन्य- जजसके तहत कई सभवत्त चचत्रों में पाया जा सकता है , उदाहरण के सलए,

स्कूल थे।

DPP 2023 DAY 48 62


Contact us : info@onlyias.com

OnlyIAS Nothing Else Visit : dpp.onlyias.in


Contact : +91-7007 931 912

भरहुत और सांची स्तप


ू में , अमरावती की प्रमख
ु ता उच्चारण को इंचगत करने के सलए एक शब्द में एक
आहद। अक्षर के ऊपर या नीचे (या कभी-कभी उसके बगल
• या़ का उल्लेख दस
ू री शताब्दी ईस्वी के पुराने में ) चचनन होते हैं।
तसमल ग्रंथों में समलता है । उदाहरण: संतूर, सारं गी, • कथन 2 र्गलत है : 92वें संवविान संशोिन
हदलरुबा, एसराज, कमाइचा आहद। पंडडत सशव अचिननयम, 2003 द्वारा संथाली को भारत के
कुमार शमास प्रससद्ि संतूर वादक हैं, और उस्ताद संवविान की आठवीं अनुसूची में शासमल ककया
सुल्तान खान सारं गी के एक प्रससद्ि वादक हैं। गया, जजसमें बोडो, डोगरी और मैचथली भार्ाओं के
• युग्म 3 र्गलत है : अवनद्ि वाद्य (मेम्िानोफोन्स साथ-साथ भारत की आचिकाररक भार्ाओं को
या पक्यश
ूस न इंस्ूमें ट्स) में जानवरों की खाल से सच
ू ीबद्ि ककया गया है । इसे जोडने का अथस था कक
टकराकर ध्वनन उत्पन्न होती है जजसे समट्टी या भारत सरकार संताली भार्ा के ववकास के सलए
िातु के बतसन या लकडी के बैरल या फ्रेम में फैलाया बाध्य थी। आठवीं अनुसूची के माध्यम से, सरकार
जाता है । ऐसे यंत्रों का सबसे पहला उल्लेख वेदों में स्कूल स्तर की परीक्षाओं और सावसजननक सेवा
समलता है जहां भसू म दं ि
ु ुभी का उल्लेख समलता है । (नौकररयों) के सलए प्रवेश परीक्षाओं में बैठने वाले
• प्रकार: आकार के आिार पर इन्हें ववसभन्न वगों में छात्रों को भार्ा का उपयोग करने की अनम
ु नत दे ती
बााँटा गया है । है ।
o ऊध्वासका: जैसे तबला जोडी और चें दा।
आज तबला बजाने वाले प्रमख
ु संगीतकार ONLYIAS द्वारा ववशेर्ांक
हैं-उस्ताद आसलया राखा खान, जाककर ंताल:
हुसैन, शफात अहमद और समता प्रसाद। • संथाल समुदाय असम, ब्रत्रपुरा, ब्रबहार, छत्तीसगढ़,
o अंक्य: आज इस ककस्म में मद
ृ ं गम, पजवचम बंगाल, ओडडशा और झारखंड सहहत कई
पखावज, खोल आहद प्रमख
ु हैं। राज्यों में केंहद्रत है ।
o असलंग्या: डफ, डफली, आहद। • वे संताली भार्ा बोलते हैं जो ऑस्रो-एसशयाई भार्ाओं
o डमरू जैस:े हहमाचल प्रदे श का हुड्डा, के उप-पररवार के मुंडा समूह से संबंचित है ।
दक्षक्षणी क्षेत्र का नतसमला। • संताल रं गीन दीवारों, फशों और कलात्मक रूप से
Reference: नक्काशीदार दरवाजों के साथ अपने आवास
https://www.thehindu.com/news/national/tamil-
nadu/narasinghapettai-nagaswaram-bags-gi- वास्तुकला के सलए जाने जाते हैं।
tag/article65243440.ece • उनका प्राथसमक व्यवसाय बसे हुए कृवर् है ।

Q.95) Ans: a Reference:


Exp: https://indianexpress.com/article/india/indian-
constitution-translation-ol-chiki-7823156/
• कथन 1 ही है : ओल चचकी सलवप- ओल शब्द का
अथस है “सलखना”, और चचकी का अथस है “सलवप” –
का आववष्कार पंडडत रघन
ु ाथ मम
ु ूस ने 1925 में Q.96) Ans: D
Exp:
संताली सलखने के सलए ककया था। ओल चचकी
• कथन 1 ही है : बंगाल की खाडी बहु-क्षेत्रीय
वणसमाला है , और अन्य भारतीय सलवपयों के ककसी
तकनीकी और आचथसक सहयोग पहल (BIMSTEC)
भी शब्दांश गुण को साझा नहीं करता है । सलवप में
एक क्षेत्रीय बहुपक्षीय संगठन है । यह न केवल
30 अक्षर और पााँच बुननयादी ववशेर्क (एक ववशेर्
दक्षक्षण और दक्षक्षण पव
ू स एसशया को जोडता है , बजल्क

DPP 2023 DAY 48 63


Contact us : info@onlyias.com

OnlyIAS Nothing Else Visit : dpp.onlyias.in


Contact : +91-7007 931 912

महान हहमालय और बंगाल की खाडी की o जन दर जन संपकस


पाररजस्थनतकी को भी जोडता है । o गरीबी उपशमन
• कथन 2 सही है : इसके सदस्य बंगाल की खाडी के o काउं टर टे रररज्म एंड रांसनेशनल िाइम
तटवती और आसन्न क्षेत्रों में जस्थत हैं जो एक o जलवायु पररवतसन
ननकटवती क्षेत्रीय एकता का गठन करते हैं। पांच o व्यापार और ननवेश
दक्षक्षण एसशया से हैं- बांग्लादे श, भूटान, भारत o प्रौद्योचगकी
नेपाल, िीलंका और दो दक्षक्षण पूवस एसशया- o ऊजास
म्यांमार, थाईलैंड से हैं। इसका मुख्य उद्दे वय तीव्र o पररवहन और संचार
आचथसक ववकास के सलए अनक
ु ू ल वातावरण तैयार o पयसटन
करना है ; सामाजजक प्रगनत में तेजी लाना; और इस o मत्स्य पालन
क्षेत्र में साझा हहत के मामलों पर सहयोग को बढ़ावा
Reference:
दे ना। https://www.thehindu.com/news/national/pm-modi-
BIMSTEC के उद्दे श्य calls-for-strengthening-bimstec/article65273458.ece
o उप-क्षेत्र के तीव्र आचथसक ववकास के सलए
अनुकूल वातावरण तैयार करना। Q.97) Ans: A
o समानता और साझेदारी की भावना को Exp:
प्रोत्साहहत करना। o कथन 1 ही है : एसशयन इन्फ्रास्रक्चर

o सदस्य दे शों के साझा हहतों के क्षेत्रों में इन्वेस्टमें ट बैंक (एआईआईबी) एक

सकिय सहयोग और पारस्पररक सहायता बहुपक्षीय ववकास बैंक है जजसका समशन

को बढ़ावा दे ना एसशया में सामाजजक और आचथसक

o सशक्षा, ववज्ञान और प्रौद्योचगकी आहद के पररणामों में सुिार करना है , जनवरी 2016

क्षेत्र में एक दस में पररचालन शुरू हुआ। इसका उद्दे वय


ू रे के सलए समथसन में तेजी
लाना। ववकास को प्रोत्साहहत करना और

• कथन 3 ही है : ब्रबम्सटे क ,साकस और आससयान बनु नयादी सेवाओं तक पहुंच में सि


ु ार

सदस्यों के बीच अंतर-क्षेत्रीय सहयोग के सलए एक करना है । बुननयादी ढांचे में प्रगनत के

मंच है । यह लगभग 1.5 ब्रबसलयन लोगों का ननवास माध्यम से क्षेत्र में परस्पर संपकस और

है , जो वैजववक आबादी का लगभग 22% हहस्सा हैं। आचथसक ववकास। इसका मुख्यालय बीजजंग

2.7 हरसलयन अथसव्यवस्था के संयुक्त सकल घरे लू में स्थावपत ककया गया था। अब 100 से

उत्पाद (जीडीपी) के साथ, ब्रबम्सटे क सदस्य राज्य अचिक सदस्य हैं।

वपछले पांच वर्ों में औसतन 6.5% आचथसक ववकास o कथन 2 गलत है : बैंक में 26.61% वोहटंग

प्रक्षेपवि को बनाए रखने में सक्षम हैं। शेयरों के साथ चीन सबसे बडा शेयरिारक

• ब्रबम्सटे क ने सहयोग के सलए ननम्नसलखखत क्षेत्रों है , इसके बाद भारत (7.6%), रूस (6.01%)

की पहचान की है : और जमसनी (4.2%) का स्थान है । इससलए,

o कृवर् एसशयन इन्फ्रास्रक्चर इन्वेस्टमें ट बैंक

o सांस्कृनतक सहयोग (AIIB) में जमसनी की हहस्सेदारी सबसे कम

o पयासवरण और आपदा प्रबंिन है । क्षेत्रीय सदस्यों के पास बैंक में कुल

o सावसजननक स्वास्थ्य मतदान शजक्त का 75% हहस्सा है ।

DPP 2023 DAY 48 64


Contact us : info@onlyias.com

OnlyIAS Nothing Else Visit : dpp.onlyias.in


Contact : +91-7007 931 912

o एसशयाई अवसंरचना ननवेश बैंक के है । समझौते के अनस


ु ार, सदस्य दे श अन्य दे शों में
ववसभन्न अंग: पंजीकृत वाहनों को कुछ ननयमों और शतों के तहत
o कथन 3 गलत है : बोडस ऑफ गवनसस:स - बोडस अपने क्षेत्र में आने की अनुमनत दें गे और सीमा
ऑफ गवनससस में प्रत्येक सदस्य दे श द्वारा शुल्क और शल्
ु क संबचं ित दे शों द्वारा तय ककए
ननयुक्त एक गवनसर और एक वैकजल्पक जाएंगे और इन्हें द्ववपक्षीय और ब्रत्रपक्षीय मंचों पर
गवनसर होता है :- गवनसर और वैकजल्पक अंनतम रूप हदया जाएगा।
गवनसर ननयुजक्त सदस्य की खश
ु ी पर काम • कथन 3 गलत है: एसशयाई ववकास बैंक (एडीबी)
करते हैं। दक्षक्षण एसशया उपक्षेत्रीय आचथसक सहयोग
o ननदे शक मंडल – अननवासी ननदे शक (एसएएसईसी) कायसिम में अपनी सहायता के
मंडल बैंक के सामान्य संचालन की हदशा हहस्से के रूप में बीबीआईएन एमवीए पहल को
के सलए जजम्मेदार है , जो बोडस ऑफ गवनससस तकनीकी, सलाहकार और ववत्तीय सहायता प्रदान
द्वारा इसे सौंपे गए सभी अचिकारों का कर रहा है ।
प्रयोग करता है । • कागो प्रोटोकॉल अचिक उप-क्षेत्रीय सहयोग को
o अंतरासष्रीय सलाहकार पैनल: - बैंक की बढ़ावा दे कर बीबीआईएन दे शों के बीच व्यापार और
रणनीनतयों और नीनतयों के साथ-साथ लोगों से लोगों के बीच संपकस की पूरी क्षमता का
सामान्य पररचालन मुद्दों पर अध्यक्ष और एहसास करने में मदद करे गा।
वररष्ठ प्रबंिन का समथसन करने के सलए
बैंक ने एक अंतरासष्रीय सलाहकार पैनल
(आईएपी) की स्थापना की है ।

Reference:
https://www.thehindu.com/news/international/asian
-infrastructure-investment-bank-puts-russia-belarus-
projects-on-hold/article65187557.ece

Reference:
https://www.thehindu.com/news/national/banglades
Q.98) Ans: C h-india-nepal-move-ahead-on-motor-vehicle-
Exp: agreement-project/article65205145.ece
• कथन 1 ही है : हाल ही में , भारत, नेपाल और
बांग्लादे श ने बांग्लादे श-भूटान-भारत-नेपाल Q.99) Ans: A
(बीबीआईएन) मोटर वाहन समझौते को लागू करने Exp:
India- Australia relations:
के सलए एक समझौता ज्ञापन को अंनतम रूप हदया
• कथन 1 र्गलत है : व्यापार संबि
ं : ऑस्रे सलया भारत
है । बांग्लादे श, भूटान, भारत और नेपाल
का 17वां सबसे बडा व्यापाररक भागीदार है , जबकक
(बीबीआईएन) ने 2015 में चथम्प,ू भट
ू ान में मोटर
भारत ऑस्रे सलया का 9वां सबसे बडा भागीदार है
वाहन समझौते (एमवीए) पर हस्ताक्षर ककए थे।
और भारत ने ऑस्रे सलयाई सरकार के साथ संयक्
ु त
• कथन 2 सही है : इसका उद्दे वय न केवल लोगों से
रूप से $6 समसलयन का ननवेश करने के सलए
लोगों के बीच सहज संपकस प्रदान करना है बजल्क
प्रनतबद्ि ककया है क्योंकक यह ऑस्रे सलया में
लोगों और सामानों की सीमा पार आवाजाही को
सलचथयम और कोबाल्ट खदानों की खोज करता है ।
सुवविाजनक बनाकर आचथसक संपकस को बढ़ाना भी

DPP 2023 DAY 48 65


Contact us : info@onlyias.com

OnlyIAS Nothing Else Visit : dpp.onlyias.in


Contact : +91-7007 931 912

• कथन 2 सही है : बहुपक्षीय सहयोग: ऑस्रे सलया मख्


ु यालय रोम में है और इसकी अध्यक्षता एक राष्ट्रपनत
संयुक्त राष्र सुरक्षा पररर्द में भारत की करते हैं।
उम्मीदवारी का समथसन करता है । भारत और • आईएफएडी के उद्दे श्य तीन गुना हैं
ऑस्रे सलया क्वाड, जी-20, आईओआर-एआरसी, o गरीब लोगों की उत्पादक क्षमता में वद्
ृ चध करना।
आससयान क्षेत्रीय मंच, जलवायु और स्वच्छ ववकास o बाजार भागीदारी से उनके मलए लाभ बढाना।
पर एसशया प्रशांत साझेदारी के सदस्य हैं। o उनकी आचथवक गनतववचधयों की पयाववरणीय स्वथरता और
• कथन 3 सही है : रक्षा और ऊजास सुरक्षा: ऑस्रे सलया, जलवायु ल ीलापन को मजबत
ू करने के मलए।
औसइंडेक्स जैसे रक्षा अभ्यास सैन्य • कृवर् ववकास के मलए अिंतरावष्ट्रीय कोर् में 177 सदवय दे श
अंतःकियाशीलता को बढ़ाते हैं, अपने पारस्पररक हैं। आईएफएडी हर साल ग्रामीण ववकास ररपोटव लाता है ।
रसद समथसन समझौते के माध्यम से भारत-प्रशांत आईएफएडी की ग्रामीण ववकास ररपोटव 2021 खाद्य प्रणाली
क्षेत्र में शांनत, सुरक्षा और जस्थरता को बढ़ावा दे ते हैं। पररवतवन के सिंदभव में ग्रामीण आजीववका पर केंहद्रत है ।
2014 में, दोनों दे शों ने भारत को यूरेननयम की ब्रबिी ररपोटव ग्रामीण लोगों के मलए समान आजीववका को बढावा
का मागस प्रशस्त करते हुए असैन्य परमाणु सहयोग दे ती है , जो पोर्ण में सुधार और पयाववरण की रक्षा करने की
समझौते पर हस्ताक्षर ककए और इस प्रकार भारत आवश्यकता के साथ-साथ खाद्य प्रणामलयों को बदलने में
की ऊजास चचंताओं को दरू ककया। सबसे आगे और केंद्र हैं।
• रणनीनतक साझेदारी: व्यापक रणनीनतक साझेदारी
सीएसपी आपसी समझ, ववववास, साझा हहतों,
लोकतंत्र के साझा मूल्यों और कानून के शासन पर
आिाररत है , और COVID-19 जैसी चुनौनतयों से
ननपटने के सलए दोनों दे शों की मजबूत प्रनतबद्िता
को दशासता है ।
Reference:https://www.hindustantimes.com/india-
news/pm-modi-morrison-to-hold-second-india-
australia-virtual-summit-today-
101647824066726.html

Q.100) Ans: d
Exp:
सभी कथन सही हैं: कृवर् ववकास के मलए अिंतरावष्ट्रीय कोर्
एक ववशेर् सिंयक्
ु त राष्ट्र एजेंसी है स्जसे 1977 में बनाया गया
था जो सिंबद्ध पररयोजनाओिं के मलए कम ब्याज के साथ
अनुदान और ऋण प्रदान करने में लगी हुई थी। IFAD ग्रामीण
लोगों के साथ काम करता है स्जससे उनहें अपनी खाद्य
सरु क्षा बढाने, पोर्ण में सध
ु ार करने और अपनी आय बढाने
की अनुमनत ममलती है ।
• कृवर् ववकास के मलए अिंतरावष्ट्रीय कोर् भी लोगों को अपने
कारोबार का वववतार करने में मदद करता है । यह सिंगठन
1974 के ववश्व खाद्य सम्मेलन का पररणाम है और इसका

DPP 2023 DAY 48 66

You might also like